X



トップページ数学
1002コメント382KB
不等式への招待 第7章
レス数が1000を超えています。これ以上書き込みはできません。
0001不等式ヲタ ( ゚∀゚)垢版2013/03/09(土) 22:14:39.95
ある人は蝶を集め、ある人は切手を収集し、ある人は不等式を集める…
          ___          ----- 参考文献〔3〕 P.65 -----
    |┃三 ./  ≧ \   
    |┃   |::::  \ ./ | 
    |┃ ≡|::::: (● (● |  不等式と聞ゐちゃぁ
____.|ミ\_ヽ::::... .ワ......ノ     黙っちゃゐられねゑ!
    |┃=__    \           ハァハァ…
    |┃ ≡ )  人 \ ガラッ

まとめWiki http://wiki.livedoor.jp/loveinequality/

過去スレ
・不等式スレッド (第1章)http://science3.2ch.net/test/read.cgi/math/1072510082/
・不等式への招待 第2章 http://science6.2ch.net/test/read.cgi/math/1105911616/
・不等式への招待 第3章 http://science6.2ch.net/test/read.cgi/math/1179000000/
・不等式への招待 第4章 http://science6.2ch.net/test/read.cgi/math/1245060000/
・不等式への招待 第5章 http://uni.2ch.net/test/read.cgi/math/1287932216/
・不等式への招待 第6章 http://uni.2ch.net/test/read.cgi/math/1332950303/
・過去スレのミラー置き場 http://cid-d357afbb34f5b26f.skydrive.live.com/browse.aspx/.Public/

姉妹サイト(?)
キャスフィ 高校数学板 不等式スレ2
http://www.casphy.com/bbs/test/read.cgi/highmath/1359202700/l50
Yahoo! 掲示板 トップ > 科学 > 数学
http://messages.yahoo.co.jp/bbs?action=t&;amp;board=1835554&sid=1835554&type=r&first=1
0002不等式ヲタ ( ゚∀゚)垢版2013/03/09(土) 22:22:07.38
・不等式の和書
[1] 不等式,ハーディ・リトルウッド・ポリヤ,シュプリンガー,2003年
   http://amazon.jp/o/ASIN/4431710566
[2] 不等式,大関信雄・青木雅計,槇書店,1967年(絶版)
[3] 不等式への招待,大関信雄・大関清太,近代科学社,1987年
   http://amazon.jp/dp/4844372661
[4] 不等式入門(数学のかんどころシリーズ9),大関清太,共立出版,2012年
   http://www.kyoritsu-pub.co.jp/bookdetail/978432001...
[5] 不等式入門,渡部隆一,森北出版,2005年
   http://amazon.jp/o/ASIN/4627010494
[6] 不等式の工学への応用、海津聰、森北出版,2004年
   http://amazon.jp/o/ASIN/4627075812
[7] 不等式(モノグラフ4),染取弘,科学新興新社,1990年
   http://amazon.jp/o/ASIN/4894281740
[8] 不等式 〜 21世紀の代数的不等式論 〜,安藤哲哉,数学書房,2012年
  http://amazon.jp/dp/4903342700

・不等式の項目を含む和書
[1] 数学トレッキングツアー第3章「相加平均≧相乗平均」,東京理科大学数学教育研究所,教育出版,2006年
   http://amazon.jp/o/ASIN/4316801988
[2] 獲得金メダル! 国際数学オリンピック第1章「不等式」,小林一章,朝倉書店,2011年
   http://www.asakura.co.jp/books/isbn/978-4-254-1113...
[3] 数学オリンピック事典,数学オリンピック財団,朝倉書店,2001年
   http://amazon.jp/o/ASIN/4254110871
[4] 三角法の精選103問(シリーズ:数学オリンピックへの道 2),T.アンドレースク・Z.フェン著,朝倉書店,2010年
   http://www.asakura.co.jp/books/isbn/978-4-254-1180...
[5] 最大値と最小値の数学,P.J.ナーイン,シュプリンガー,2010年
   http://amazon.jp/dp/4621062131
[6] 最大・最小(数学one Point双書24),服部泰,共立出版,1979年
   http://amazon.jp/dp/4320012445
0003不等式ヲタ ( ゚∀゚)垢版2013/03/09(土) 22:24:39.92
・不等式の洋書
[1] The Cauchy-Schwarz Master Class: An Introduction to the Art of Mathematical Inequalities,J. M. Steele,Cambridge Univ. Pr.,2004年
   http://amazon.jp/o/ASIN/052154677X
[2] Inequalities: A Mathematical Olympiad Approach,Birkhaeuser Basel,2009年
   http://amazon.jp/dp/3034600496
[3] Inequalities: Theorems (Techniques and Selected Problems),Zdravko Cvetkovski,Springer,2012年
   http://amazon.jp/gp/product/3642237916
[4] Analytic Inequalities,Xingzhi Zhan,Dragoslav S., Dr. Mitrinovic,Springer,1970年
   http://www.amazon.co.jp/dp/3642999727
[5] Matrix Inequalities (Lecture Notes in Mathematics, No.1790),Xingzhi Zhan,Springer,2002年
   http://amazon.jp/dp/3540437983
[6] Matrix Analysis (Graduate Texts in Mathematics),Rajendra Bhatia,Springer,1996年
   http://amazon.jp/dp/0387948465

・不等式の記事
[1] 特集 「現代の不等式」 (数理科学 No.386) ,サイエンス社,1995年8月号(絶版)
[2] 特集 「不等式の世界」 (数学セミナー No.2-569) ,日本評論社,2009年2月号
   http://amazon.jp/o/ASIN/B001O9UIZ8
[3] 連載 「不等式の骨組み」 (大学への数学 vol.53,全12回,各4ページ),栗田哲也,東京出版,2009年4月号-2010年3月号
   http://www.tokyo-s.jp/index.shtml
0004不等式ヲタ ( ゚∀゚)垢版2013/03/09(土) 22:26:43.52
・不等式の埋蔵地
[1] RGMIA http://rgmia.vu.edu.au/
[2] Crux Mathematicorum Synopses http://www.journals.cms.math.ca/CRUX/synopses/
[3] Maths problems http://www.kalva.demon.co.uk/
[4] Mathematical Inequalities & Applications http://www.ele-math.com/
[5] American Mathematical Monthly http://www.maa.org/pubs/monthly.html
[6] Problems in the points contest of KöMaL http://www.komal.hu/verseny/feladatok.e.shtml
[7] IMO リンク集 http://imo.math.ca/
[9] Mathematical Olympiads Correspondence Program http://www.cms.math.ca/Competitions/MOCP/
[10] Mathematical Excalibur http://www.math.ust.hk/excalibur/
[11] MathLinks Contest http://www.mathlinks.ro/Forum/contest.html
[12] MATH PROBLEM SOLVING WEB PAGE http://www.math.northwestern.edu/~mlerma/problem_s... (要自動登録)
[13] Wolfram MathWorld http://mathworld.wolfram.com/
[14] GRA20 Problem Solving Group http://www.mat.uniroma2.it/~tauraso/GRA20/main.htm...
[15] American Mathematical Monthly Problems http://www.mat.uniroma2.it/~tauraso/AMM/amm.html
[16] Journal of Inequalities and Applications http://www.hindawi.com/journals/jia/
[17] すうじあむ http://suseum.jp/gd/all_berry_list/3504

・海外不等式ヲタの生息地
[1] Journal of Inequalities in Pure and Applied Mathematics http://jipam.vu.edu.au/
[2] MIA Journal http://www.mia-journal.com/
[3] MathLinks Math Forum http://www.mathlinks.ro/Forum/forum-55.html
0005132人目の素数さん垢版2013/03/09(土) 22:35:09.24
〔問題〕
a,b,c>0, (r=2/3 または r=1 または r≦0) のとき
 {2a/(b+c)}^r + {2b/(c+a)}^r + {2c/(a+b)}^r ≧ 3.
を示せ。

 casphy - 高校数学 - 不等式2 - 032-035, 042-043
 USAMO ?
0006KingMathematician ◆LoZDre77j4i1 垢版2013/03/10(日) 07:31:04.00
円周率をπと書く. 223/71 < π < 22/7. Archimedes の時代から知られていたらしい.
0008132人目の素数さん垢版2013/03/10(日) 23:11:29.38
>>2
追加でつ!

不等式への招待 第6章
http://uni.2ch.net/test/read.cgi/math/1332950303/901

901 名前:132人目の素数さん[sage] 投稿日:2012/12/06(木) 23:47:56.44
美しい不等式の世界 ─数学オリンピックの問題を題材として─

A5/272ページ/2013年01月25日
ISBN978-4-254-11137-8 C3041
定価3,990円(税込)
佐藤淳郎 訳

"Inequalities A Mathematical Olympicd Approach"の翻訳。
数学全般で広く使われる有名な不等式や実用的テクニックを系統立てて説明し,
数学オリンピックの問題をふんだんに使って詳しく解説。
多数の演習問題およびその解答も付す。

http://www.asakura.co.jp/books/isbn/978-4-254-11137-8/



きたか…!!

  ( ゚д゚ ) ガタッ
  .r   ヾ
__|_| / ̄ ̄ ̄/_
  \/    /
0009132人目の素数さん垢版2013/03/11(月) 12:45:08.18
>>5
そういえば、東大の入試で円周率の不等式が出て話題になったな。
簡単と思いきや、出来が悪かったそうだ。


問題.円周率 π は、3.05 より大きいことを証明せよ。
0010不等式ヲタ ( ゚∀゚)垢版2013/03/12(火) 00:25:08.56
>>2 の修正
すまなんだ
まとめWikiからコピペしたら、長いURLは ...... と略されるのを忘れていました

>>2
・不等式の和書
[4] 不等式入門(数学のかんどころシリーズ9),大関清太,共立出版,2012年
  http://www.kyoritsu-pub.co.jp/bookdetail/9784320019898

・不等式の項目を含む和書
[2] 獲得金メダル! 国際数学オリンピック第1章「不等式」,小林一章,朝倉書店,2011年
  http://www.asakura.co.jp/books/isbn/978-4-254-11132-3/
[4] 三角法の精選103問(シリーズ:数学オリンピックへの道 2),T.アンドレースク・Z.フェン著,朝倉書店,2010年
  http://www.asakura.co.jp/books/isbn/978-4-254-11808-7/

>>4
・不等式の埋蔵地
[12] MATH PROBLEM SOLVING WEB PAGE
  http://www.math.northwestern.edu/~mlerma/problem_solving/ (要自動登録)

         ∧_∧
         (´Д` )   死んでお詫びを…
         /  y/  ヽ      
    Σ(m)二フ ⊂[_ノ
        (ノノノ | | | l )
    ̄ ̄ ̄ ̄ ̄ ̄ ̄ ̄ ̄ ̄
0011132人目の素数さん垢版2013/03/12(火) 15:48:06.49
最近不等式の証明の世界を知りました高一です
学校ではn=2の場合においてのAM-GM不等式しか習いませんでした

ですが、最近とある人から数オリクラスの不等式の証明もある事を聞き興味を持ってます
まずこのスレに出てくるような問題を解くために勉強すべき事はなんでしょうか
現時点で三角関数までしかやってないです
0013132人目の素数さん垢版2013/03/12(火) 19:19:54.75
何をおいてもまずは微分積分を身に着けてから
数Vの教科書を取り寄せて勉強するべし
0014132人目の素数さん垢版2013/03/13(水) 00:24:33.98
>>9

原点(0,0)を中心とする単位円上に2点
 A (1, 0)
 B (1/√2, 1/√2)
をとる。弧AB は円周の 1/8 である。
 |AB|^2 = (1 - 1/√2)^2 + (1/√2)^2
    = 2 - √2 > 2 - 1.414775 = 0.585225 = 0.765^2,
 AB = √(2-√2) > 0.765
 π > 4・AB > 3.06
0015132人目の素数さん垢版2013/03/13(水) 01:05:14.09
>>9
2003年の東大理系の問題だったんだよね。

当時は小学校で円周率がおよそ3で済ます、ということになり、それでは数学教育として余りにも酷い、
というメッセージを世間に送る意味で出題させれたという時代背景があった。

一応日本の最高学府のしかも理系の問題でそれを提示することで、当時の数学教育に反論するのが狙い。
東大ともなると、単に難しい問題を出すだけでなく、高校数学界への影響も考慮してと意外と大変ですね。
0017132人目の素数さん垢版2013/03/13(水) 21:44:36.82
>>14
C (cosθ, sinθ)
とおくと、
 |AC|^2 = (1-cosθ)^2 + (sinθ)^2
     = 2(1-cosθ),

(例) θ=π/6 のとき
  C((√3)/2, 1/2)

 |AC|^2 = 2 - √3 = 0.268
 π > 6|AC| > 3.10

 |BC|^2 = 2 - √(3/2) - √(1/2) = 0.068
 π > 12|BC| > 3.13
0018132人目の素数さん垢版2013/03/14(木) 00:28:02.56
>>17
円周率の評価の証明で、弧度法を使ったらダメだろうが!

証明すべきこと(1周=2π)を使っているんだから、全然証明になってない。
0020132人目の素数さん垢版2013/03/14(木) 00:38:11.02
証明すべきことは
>問題.円周率 π は、3.05 より大きいことを証明せよ。
なんだが
0024132人目の素数さん垢版2013/03/14(木) 02:14:13.46
>>15
円周率が3となっていたのは日能研の電車の吊り広告での話
実際の教科書見たらそうじゃないってのはすぐにわかるのに
メディアが円周率3って大きく取り上げた
それだけのこと
0025132人目の素数さん垢版2013/03/14(木) 02:26:39.53
>>16
(1) x=1 で最小
(2) x=y で最小、
         最小値が x=y=1 のとき 0になる
おしまい
0027132人目の素数さん垢版2013/03/15(金) 04:39:07.65
>>16 (2)
重み付き相加相乗平均より、
 (a/(a+b+c))s + (b/(a+b+c))t + (c/(a+b+c))u ≧ s^(a/(a+b+c)) t^(b/(a+b+c)) u^(c/(a+b+c)).
上式に、 s = x^(a+b+c), t = y^(a+b+c), u = 1 を代入して整理。
0030132人目の素数さん垢版2013/03/16(土) 16:33:04.08
x_i(1≦i≦n+1)は正の実数でx_1+x_2+……+x_n=1, x_(n+1)=0を満たす時
Σ[k=1_n]{√(Σ[p=1_k]x_k)×√(1+Σ[q=k+1_n+1]x_q)×x_k}>π/4
0032132人目の素数さん垢版2013/03/25(月) 03:03:59.89
>>30
【審議凍結】
    ______________
   /|//              / / /|
 //|/ /         // / /  |
 | ̄ ̄ ̄ ̄ ̄ ̄ ̄ ̄ ̄ ̄ ̄ ̄ ̄ ̄|.///.|
 |/ |   .∧,,∧.  ∧,,∧./// │   .|
 |  ∧∧(´‐ω‐`)(´‐ω‐`)∧∧.  .|   .|
 | (´‐ω‐).∧∧) (∧∧ (‐ω‐`) .│///|
 | | U (´‐ω‐`)(´‐ω‐`) と ノ ./| .   |
 |  u-u (l    ) (    ノ u-u / .|/// |
 |       `u./ '/u-u'       |  /
 |//    //    //    .|/
   ̄ ̄ ̄ ̄ ̄ ̄ ̄ ̄ ̄ ̄ ̄ ̄ ̄ ̄
0033132人目の素数さん垢版2013/03/26(火) 00:14:38.70
>>30

 y_0 = 1,  
 y_k = Σ[q=k+1,n] x_q 
 y_n = 0,
とおくと、y_k は単調減少で
 (左辺) = Σ[k=1_n] √(1 - y_k) × √(1 + y_k) × {y_(k-1) - y_k}
     = Σ[k=1_n] √{1 - (y_k)^2} × {y_(k-1) - y_k}
     > Σ[k=1_n] ∫[y_k, y_(k-1)] √(1-yy) dy
     = ∫[0,1] √(1-yy) dy   (半径1の四分円 >>31)
     = π/4.
0036132人目の素数さん垢版2013/03/30(土) 21:23:53.23
〔問題〕
 √(1+xx) − |x| = F(x) とおくとき、次を示せ。

(1) |xy|≦1/2 ⇒ F(x) + F(y) ≧ 1,

(2) |xyz| ≦ (4/3)^3 ⇒ F(x) + F(y) + F(z) ≧ 1,

(3) |xy| ≧ (3/4)^2 ⇒ F(x) + F(y) ≦ 1,

 [元スレ.414, 459, 482]
 casphy - 高校数学 - 不等式スレ [1-919]
0037132人目の素数さん垢版2013/03/30(土) 23:05:12.60
>>36

(1) 分子を有利化する。
 F(x) + F(y) -1 = √(1+xx) + √(1+yy) - (1+x+y)
  = {2√(1+xx)・√(1+yy) - (-1+2x+2y+2xy)}/D1
  = {1 + 2(1-2xy)(1+2x+2y)}/(D1・D2)
  ≧ 0,    (← |xy|≦1/2)

ここに、D1 = √(1+xx) + √(1+yy) + (1+x+y) >0,
    D2 = 2√(1+xx)・√(1+yy) + (-1+2x+2y+xy) >0,

(3) 上と同様に
 1 + 2(1-2xy)(1+2x+2y)
  = 1 + 2(1-2GG)(1+2x+2y)
  ≦ 1 + 2(1-2GG)(1+4G) (← AM≧GM)
  = (3-4G)(1+2G)^2
  ≦ 0,         (← G≧3/4)
0038132人目の素数さん垢版2013/04/28(日) 07:48:43.97
実数x,y,zがx^2+y^2+z^2=1 を満たすとき
xy^2 + yz^2 + zx^2 のとり得る値の範囲はどうもとめればいいでしょyか
0040132人目の素数さん垢版2013/04/29(月) 19:19:55.62
>>38
キャスフィーの解答....

コーシーにより
 (xy^2 + yz^2 + zx^2)^2 ≦ {(xy)^2 + (yz)^2 + (zx)^2}(y^2 + z^2 + x^2),
 等号成立は x=y=z のとき,
とし、右辺に
 XY+YZ+ZX = {2(X+Y+Z)^2 -(X-Y)^2 -(Y-Z)^2 -(Z-X)^2}/6
      ≦ (1/3)(X+Y+Z)^2,
を使えばいいんじゃない?

 (xy^2 + yz^2 + zx^2)^2 ≦ (1/3)(x^2 + y^2 + z^2)^3 = 1/3,

∴取り得る値の範囲は −1/√3 〜 1/√3.
  (x=y=z=±1/√3 のとき)
0042132人目の素数さん垢版2013/04/30(火) 21:32:41.45
>>41
その問題は [第5章.288] のように、
 p = a^2 -ca +bc,
 q = b^2 -ab +ca,
 r = c^2 -bc +ab,
とおくといいらしいよ。

 p + q + r = a^2 + b^2 + c^2,
 pq + qr + rp = ab^3 + bc^3 + ca^3,
より
 (a^2 +b^2 +c^2)^2 - 3(ab^3 +bc^3 +ca^3)
  = (p+q+r)^2 - 3(pq+qr+rp)
  = (1/2){(p-q)^2 + (q-r)^2 + (r-p)^2}
  ≧ 0,

 [第5章.268-269, 284-290]
 [キャスフィー 不等式1-517, 563]
0043132人目の素数さん垢版2013/04/30(火) 22:02:48.24
〔類題〕
a,b,c ≧ 0 のとき
 (a^2 +b^2 +c^2)^2 ≧ 3{a^(4/3)・b^(8/3) + b^(4/3)・c^(8/3) + c^(4/3)・a^(8/3)},

(略解)
相加・相乗平均により
 (ab)^2 + (ab)^2 + b^4 = (a^2 +a^2 +b^2)b^2 ≧ 3a^(4/3)・b^(8/3),
 循環的にたす。
0044132人目の素数さん垢版2013/04/30(火) 22:33:15.02
第5章とはどの書物のことでしょうか
0047132人目の素数さん垢版2013/05/26(日) 20:35:53.55
〔問題〕
n∈N のとき、
 1/{2n + 4/(n+3)} < ∫[0,π/4] {tan(x)}^n dx < 1/(2n),
を示せ。(ブリジッタ)

 casphy - 高校数学 - ∫積分∫ - 046
0048132人目の素数さん垢版2013/05/31(金) 23:01:32.22
>>47
 キャスフィーの解答....

(右)
 tan(x) = t とおくと、dx = dt/(1+tt),
 1+tt > 2t, (← 相加・相乗平均)
 I_n = ∫[0,1] (t^n)/(1+tt) dt
    < ∫[0,1] t^(n-1) /2 dt
    = [ (t^n) /(2n) ](x=0,1)
    = 1/(2n),
(左)
 I_n = 1/(n+1) - I_(n+2)
   > 1/(n+1) - 1/{2(n+2)}
   = (n+3)/{2(n+1)(n+2)}
   = (n+3)/{2n(n+3) + 4}
   = 1/{2n + 4/(n+3)},
0049132人目の素数さん垢版2013/06/03(月) 22:28:46.67
>>48

 I_n > 1/(n+1) - 1/{2(n+2)}
    = ∫[0,1] (t^n)(1 - t/2) dt,
は t=1 で接線を引いて
  1/(1+tt) ≧ 1 - t/2,
としたことに相当する。
さらに
 1/(1+tt) ≦ (5-4t+tt)/4,
から、
 I_n < {5/(n+1) -4/(n+2) +1/(n+3)}/4
    = (nn+6n+10)/{2(n+1)(n+2)(n+3)}
    = (nn+6n+10)/{2n(nn+6n+10) +2(n+6)}
    = 1/{2n + 2(n+6)/[n(n+6)+10]}
    = 1/{2n + 2/[n + 10/(n+6)]}.
0050132人目の素数さん垢版2013/06/03(月) 23:11:29.81
キャスフィーから....

〔問題731〕
0 < |x| < π/2 のとき、
 sin(x)/x > cos(x/√3) > cos(x)^(1/3),
              (でえ)
0051132人目の素数さん垢版2013/06/05(水) 23:07:37.09
↑のハイパボリック版...

〔問題738〕
 sinh(x)/x > cosh(x/√3) > cosh(x)^(1/3),
               (prime_132)


cos(√t) (0<t<π^2)、cosh(√t) は下に凸らしい....
0052132人目の素数さん垢版2013/06/09(日) 21:28:40.72
>>50
 g(t) = cos(√t) は下に凸。3倍角公式から、
 cos(x/√3)^3 = {3cos(x/√3) + cos((√3)x)}/4
      = {3g(xx/3) + g(3xx)}/4
      > g(xx)   (← Jensen)
      = cos(x),
 
>51 右
 g(-t) = cosh(√t) は下に凸。3倍角公式から、
 cosh(x/√3)^3 = {3cosh(x/√3) + cosh((√3)x)}/4
      = {3g(-xx/3) + g(-3xx)}/4
      > g(-xx)   (← Jensen)
      = cosh(x),
0053132人目の素数さん垢版2013/06/09(日) 21:33:06.16
>>52

 g(t) は t≦20 で下に凸。

(略証)
・t>0 のとき
 g(t) = cos(√t),
 g '(t) = -sin(√t)/(2√t),
 g "(t) = {sin(√t) - (√t)cos(√t)}/(4t√t),
 そこで sinθ - θ・cosθ = 0, θ>0 となる最小のθを求める。
 1/θ = 1/tanθ = tan((3/2)π - θ) > (3/2)π - θ,
 1/θ + θ > (3/2)π > (20 + 1)/(√20),
 θ > √20 ≧ √t,

・t≦0 のとき
 マクローリン展開
 g(-t) = 1 + (1/2!)t + (1/4!)tt + ・・・・ + {1/(2k)!}t^k + ・・・・
 の係数がすべて正。
0055132人目の素数さん垢版2013/06/16(日) 00:09:30.45
左側はマクローリン展開。

>>50
 sin(x)/x > 1 - xx/3! + (x^4)/5! - (x^6)/7!
    = 1 - xx/3! + (x^4)/216 + (x^4)(1/270 - xx/7!)
    > 1 - xx/3! + (x^4)/216   (← xx<14)
    > cos(x/√3),

>>51
 2k+1 ≦ 3^k,
 sinh(x)/x = 納k=0,∞) (xx)^k/(2k+1)!
      > 納k=0,∞) (xx/3)^k/(2k)!
      = cosh(x/√3),
0057132人目の素数さん垢版2013/06/18(火) 22:07:49.29
〔類題〕
|xyz| ≦1 のとき、次を示せ。
 √(1+xx) + √(1+yy) + √(1+zz) -|x| -|y| -|z| ≧ 1,

[前スレ.414、459、482]
0058132人目の素数さん垢版2013/06/18(火) 22:12:24.34
>>57
キャスフィーの解答....

 √(1+xx) - |x|
   = 1/{√(1+xx) + |x|}
   ≧ 1/(1+2|x|)
   = X/{X + 2|x|^(1/3)}
   ≧ X/{X + 2/|yz|^(1/3)}   (x≦1/yz)
   ≧ X/{X + 1/|y|^(2/3) + 1/|z|^(2/3)}
   = X/(X + Y + Z),
ここに、X = 1/|x|^(2/3)、Y = 1/|y|^(2/3)、Z = 1/|z|^(2/3),
0059132人目の素数さん垢版2013/07/12(金) NY:AN:NY.AN
a, b, c>0, (1/ab)+(1/bc)+(1/ca)=1 のとき.

(a-1)(b-1)(c-1)>=2(3√3-5)を示せ。
0060132人目の素数さん垢版2013/07/17(水) NY:AN:NY.AN
>>59
例によって基本対称式を
 a+b+c = s, ab+bc+ca = t, abc = u,
とおく。題意より
 s = u,
 9/t = 9/(ab+bc+ca) ≦ 1/(ab) + 1/(bc) + 1/(ca) = 1,
 t ≧ 9,
したがって
 (4t-9)[t + 3(2√3 -3)]^2 - 4t^3 = (t-9){3(16√3 -27)t + 27(2-√3)^2} ≧0,

Schur不等式(n=-2)より
 0 ≦ F_(-2)(a,b,c)
  = abc・F_1(1/a,1/b,1/c)
  = (t^3 -4stu +9uu)/uu
  = {4t^3 - (4t-9)(s+u)^2}/(4uu) (← s=u)
  ≦ (4t-9){[t + 3(2√3 -3)]^2 - (s+u)^2}/(4uu),

∴ [t + 3(2√3 -3)] ≧ s+u,
∴ (a-1)(b-1)(c-1) = u -t +s -1 ≦ 2(3√3 -5),
0061132人目の素数さん垢版2013/07/18(木) NY:AN:NY.AN
昨日行ったファミレス。席に着くなり「ただいま○○フェアで
○○○○がお勧めです」と言うので「じゃあそれを」と頼んだら
「申し訳ありません、本日は完売となっております」って。
じゃあ勧めるなよおい。完売でもとにかく言わないといけないという
決まりでもあるのかね。
0062nobu垢版2013/07/20(土) NY:AN:NY.AN
mixi招待してください
0063132人目の素数さん垢版2013/07/20(土) NY:AN:NY.AN
>>59
キャスフィーの解答から....


a>1, b>1, c>1, で示せばいい。
附帯条件は
 0 = (a-1)(b-1)(c-1) + (a-1)(b-1) + (b-1)(c-1) + (c-1)(a-1) -2
  ≧ (a-1)(b-1)(c-1) + 3{(a-1)(b-1)(c-1)}^(2/3) -2
  = GGG +3GG -2
  = (G+1)^3 -3(G+1)
  = (G+1){(G+1)^2 -3},
ここに、 G = {(a-1)(b-1)(c-1)}^(1/3),
∴ G ≦ √3 -1,
∴ (a-1)(b-1)(c-1) = GGG ≦ (√3 -1)^3 = 2(3√3 -5),
                (じゅー)
0064132人目の素数さん垢版2013/07/25(木) NY:AN:NY.AN
log(x+√1+x^2)>sinx (x>0)
0065132人目の素数さん垢版2013/07/26(金) NY:AN:NY.AN
ピーター・フランクルの本より出題

任意の実数xについて、
sinx+sin√2x≦2-1/(100*(1+x^2))
が成立することを示せ
0067132人目の素数さん垢版2013/07/26(金) NY:AN:NY.AN
>>64
キャスフィーの解答から....

|x| < π/2 のとき、|tan(x)| > |x|

 (d/dx)log(x+√(1+x^2)) = 1/√(1+x^2),
 (d/dx)sin(x) = cos(x) = 1/√{1+tan(x)^2},
から出る。

|x| > sinh(1) = 1.1752 のとき
 (左辺) = arcsinh(x) > 1 ≧ (右辺).
0068132人目の素数さん垢版2013/08/28(水) NY:AN:NY.AN
〔問題17〕
非負値の多項式、たとえば
 f(x,y,z) = (x^4)(y^2) + (x^2)(y^4) + (z^6) - 3(xyz)^2,

 {xy(x-y)}^2 + (2|xy| + z^2)(|xy| - z^2)^2,

のように、|xy| と z^2 の多項式によって表わせますが、

x, y, z の多項式の平方の和では表わせないでしょうか?


 (参) ヒルベルト「数学の問題」 No.17
0069132人目の素数さん垢版2013/09/28(土) 18:28:22.89
自然数 n≧2 に対して次を示せ。

(1)  Σ_[k=1]^n (-1)^{k+1} n_C_k (1/n^2 )^k < 1/n

(2)  Σ_[k=1]^n n_C_k { 1/(n^2-1) }^k > 1/n

(3)  Σ_[k=1]^{2n} {2n}_C_k { 1/(n^2-1) }^k > 2/(n-1)


ただし、n_C_k = n ! /( k! (n-k)! ) は二項係数とする。
0071132人目の素数さん垢版2013/10/04(金) 06:41:24.02
最近「不等式」のレベルを格段に押しげる本が出ると聞いた
いまこそ、学問の)いち分い地一分野になれるかどうか
ともきいやた。

がんばれ不等式
俺は創業以あなた方ファンです。
0072◆yEy4lYsULH68 垢版2013/10/04(金) 07:10:46.99


○●○●○●○●○●○●○●○●○●○●○●○●○●○●○●○●○●○●○●○●○●○●○●○●○●○●○●○●○●○●○●○●○●
●○●○●○●○●○●○●○●○●○●○●○●○●○●○●○●○●○●○●○●○●○●○●○●○●○●○●○●○●○●○●○●○●○
○●○●○●○●○●○●○●○●○●○●○●○●○●○●○●○●○●○●○●○●○●○●○●○●○●○●○●○●○●○●○●○●○●
●○●○●○●○●○●○●○●○●○●○●○●○●○●○●○●○●○●○●○●○●○●○●○●○●○●○●○●○●○●○●○●○●○
○●○●○●○●○●○●○●○●○●○●○●○●○●○●○●○●○●○●○●○●○●○●○●○●○●○●○●○●○●○●○●○●○●
●○●○●○●○●○●○●○●○●○●○●○●○●○●○●○●○●○●○●○●○●○●○●○●○●○●○●○●○●○●○●○●○●○
○●○●○●○●○●○●○●○●○●○●○●○●○●○●○●○●○●○●○●○●○●○●○●○●○●○●○●○●○●○●○●○●○●
●○●○●○●○●○●○●○●○●○●○●○●○●○●○●○●○●○●○●○●○●○●○●○●○●○●○●○●○●○●○●○●○●○
○●○●○●○●○●○●○●○●○●○●○●○●○●○●○●○●○●○●○●○●○●○●○●○●○●○●○●○●○●○●○●○●○●
●○●○●○●○●○●○●○●○●○●○●○●○●○●○●○●○●○●○●○●○●○●○●○●○●○●○●○●○●○●○●○●○●○
○●○●○●○●○●○●○●○●○●○●○●○●○●○●○●○●○●○●○●○●○●○●○●○●○●○●○●○●○●○●○●○●○●
●○●○●○●○●○●○●○●○●○●○●○●○●○●○●○●○●○●○●○●○●○●○●○●○●○●○●○●○●○●○●○●○●○
0073132人目の素数さん垢版2013/10/04(金) 11:32:39.72
>>71
> 最近「不等式」のレベルを格段に押しげる本が出ると聞いた

  ノ     ∧     /) ∧
  彡  ノW \从/V  W \   ミ
  (  ノ        |      ノ \)
  ∩V      、、 |       >V7
  (eLL/ ̄ ̄\/  L/ ̄ ̄\┘/3)
  (┗(      )⌒(      )┛/
   ~| \__/ |  \__/ |~     / ̄ ̄ ̄ ̄ ̄ ̄
    爻     < |  ;     爻     < 続けたまえ
    ~爻     \_/  _, 爻~.      \______
     ~爻__/⌒ ̄ ̄ ̄~~ヽ_ 爻~
     /    ー ̄ ̄\_ ̄\
  _一‘     < ̄ ̄\\\J
<\       ー ̄ ̄ヽ_ヽJ   ̄\_
  \     _ニニニヽ )       ~\
   \  _/⌒|\ ヽ_~~ ~⌒\_
  __/~    V \_|     ~\_
0074福地 裕垢版2013/10/04(金) 12:59:17.73
1.不等式 21世紀の代数的不等式論  安藤 哲哉 著  数学書房
ソン何知ってるよたいしたことない。場合は笑って許して。でもここだけは
レベル上げといてね。他はたよりにならんから。


これは大田図書館(大田区)でみつけた。
0075福地 裕垢版2013/10/04(金) 13:05:05.59
もう一冊は楽しめそう。

美しい不等式の世界
ーーーー数学オリンピックの問題を題材としてーーーーー

砂糖 淳郎 訳  朝倉書店

残念ながらともに大田図書館所蔵です
0076福地 裕垢版2013/10/04(金) 13:07:23.30
私は真摯な数学ファンのあなた方のファンです。がんばってください、
0078132人目の素数さん垢版2013/10/07(月) 10:30:44.11
>>77 ご用ですか?
ところで,3変数4次巡回不等式
f(x,y,z) = Σx^4 + A Σx^3 y + B Σ x y^3 + C Σ x^2 y^2 + D Σ x^2 y z
が任意の非負整数 x, y, z に対して f(x,y,z) >= 0 を満たすための
A, B, C, D についての必要十分条件を求めました.
複雑でここに書けないので,以下のプレプリの
Theorem 3.5, Theorem 3.6をご覧下さい.
(2ch制限でLinkが貼れないので,
[安藤哲哉] → 論文・プレプリコーナー → 論文[9] で探して下さい)
0079132人目の素数さん垢版2013/10/07(月) 10:33:47.42
(直前の続き ---- 長すぎで2chで拒否されるので)
日本語で読みたい方は以下の正誤表の補遺(系2.3.9b, 系2.3.9c)をご覧ください.
(Linkを貼ろうとすると2chから怒られるので,
[安藤哲哉] → 「不等式」正誤表 で探して下さい)
ここで,S_4=Σx^4, S_{3,1}=Σx^3 y, S_{1,3}=Σ x y^3, S_{2,2}=Σ x^2 y^2, US_1 = Σ x^2 y z です.
0080132人目の素数さん垢版2013/10/07(月) 10:44:43.57
すいません。>78 でタイプミスしました。
> が任意の非負整数 x, y, z に対して f(x,y,z) >= 0 を満たすための
が任意の非負実数 x, y, z に対して f(x,y,z) >= 0 を満たすための
0081132人目の素数さん垢版2013/10/08(火) 18:28:27.55
証明の肝が不等式であることは実際多いんだよ。
君が何を証明したいにしてもね。
0082132人目の素数さん垢版2013/10/13(日) 19:40:49.65
〔問題〕
0 < A,B,C ≦ π/2(△ABCは鈍角△でない)とき、
 cos(A)cos(A)cos(B) + cos(B)cos(B)cos(C) + cos(C)cos(C)cos(A) ≦ 2/(3√3),
 等号成立は (cosA, cosB, cosC) = (0, √(2/3), √(1/3))またはその rotation.
を示せ。

 キャスフィ! - 高校数学 - 不等式2 - 095
0083132人目の素数さん垢版2013/10/14(月) 03:23:11.65
>>69-70
 便宜上 C[n,n+1] = 0 としておく。

(1) Σ_[k=1,n] (-1)^{k+1} C[n,k] (1/nn)^k
  = C[n,1](1/nn) - Σ_[k=1,[n/2]] {C[n,2k](1/nn)^(2k) - C[n,2k+1] (1/nn)^(2k+1)}
  = C[n,1](1/nn) - Σ_[k=1,[n/2]] C[n,2k](1/nn)^(2k) {1 - [(n-2k)/(2k+1)](1/nn)}
  < C[n,1] (1/nn) = 1/n,

(2) Σ_[k=1,n] C[n,k]{1/(nn-1)}^k > C[n,1]{1/(nn-1)} = n/(nn-1),
0084132人目の素数さん垢版2013/10/14(月) 03:26:51.02
>>69-70

(3) Σ_[k=1,2n] C[2n,k]{1/(nn-1)}^k > C[2n,1]{1/(nn-1)} + C[2n,2]{1/(nn-1)}^2 + C[2n,3]{1/(nn-1)}^3
  > C[2n,1]{1/(nn-1)} + C[2n,2]{1/(nn-1)}^2 + {6(nn-1)(n-2)/3!}{1/(nn-1)}^3
  = C[2n,1]{1/(nn-1)} + n(2n-1){1/(nn-1)}^2 + (n-2){1/(nn-1)}^2
  = C[2n,1]{1/(nn-1)} + 2(nn-1){1/(nn-1)}^2
  = 2n{1/(nn-1)} + 2{1/(nn-1)}
  = 2(n+1)/(nn-1)
  = 2/(n-1),
0085132人目の素数さん垢版2013/10/16(水) 16:39:34.30
 S_{m,n} = a^m・b^n + b^m・c^n + c^m・a^n,  >>79

[Corollary 2.6]
 f(a,b,c) = S_3 + p・S_{2,1} + q・S_{1,2} + r・U.
とおく。任意の a,b,c∈R_+ に対して f(a,b,c)≧0 が成り立つための条件は、
以下の2つの条件が成り立つことである。
(1) f(1,1,1) = 3+3p+3q+r ≧ 0.
(2) 4p^3 + 4q^3 +27 ≧ (pq)^2 +18pq or "p≧0 and q≧0"

 //www.math.s.chiba-u.ac.jp/~ando/ineq17.pdf
0086132人目の素数さん垢版2013/10/16(水) 17:27:41.96
>>78

〔定理2.3.9d.〕
 f(x,y,z) = S_4 + A・S_{3,1} + B・S_{1,3} + C・S_{2,2} + D・U・S_1,

 f(1,1,1) = 3(1+A+B+C+D) = 0,
を満たすとする。任意の非負実数 x,y,z に対して f(x,y,z)≧0 が成り立つための条件は、
以下の(1)〜(6)のいずれかが成立することである。
0087132人目の素数さん垢版2013/10/16(水) 17:30:44.20
>>78 (続き)

(1) C+2≧0, A+B≧0, A≦-2√(C+2), φ(A,B,C)≦0.
(2) C+2≧0, A+B≧0, B≦-2√(C+2), φ(A,B,C)≦0.
(3) C+2≧0, -√(C+4) ≦ A+B ≦ 0, A≧-2√(C+2), B≧-2√(C+2), φ(A,B,C)≧0.
(4) C+2≧0, A+B≧0, A≧-2√(C+2), B≧-2√(C+2).
(5) C≧0, AA+AB+BB ≦ 3C+3.
(6) C+2≦0, A+B≧0, φ(A,B,C)≦0.
ここに、φ(A,B,C) = (ABC)^2 -4(AB)^3 +18(AA+BB)ABC -4(AA+BB)C^3 -(27A^4 +6AABB +27B^4) +16C^4 -80ABCC +144(AA+BB)C -192AB -128CC +256

 //www.math.s.chiba-u.ac.jp/~ando/alg_ineq.pdf
0088132人目の素数さん垢版2013/10/19(土) 10:25:56.89
>> 87
その話で,f(1,1,1)=0という条件をはずした場合はどうなるのかというと,
実はかなり難しいのです。
「不等式」正誤表の末尾に,その話を追加しておきました。
長くて難しい話ですので,そちらをご覧下さい。
Linkは>>87の通りです。
0090132人目の素数さん垢版2013/10/20(日) 00:02:09.40
>>41
 和書[8], p.74-75 例題2.3.10 (3)


〔類題〕
a,b,c∈R のとき
 (a^2+b^2+c^2)^2 + (8/√7)(aaab+bbbc+ccca) ≧ 0,

 和書[8], p.77-78 例題2.3.13 (2)
0091132人目の素数さん垢版2013/10/21(月) 23:40:17.56
〔問題〕 次式を代数的に示せ。

(1) x≧y≧1 のとき、
 1/(1+x^n) + (n-1)/(1+y^n) ≧ n/(1+x・y^(n-1)),

(2) a_1, a_2, ・・・・, a_n ≧ 1 のとき、
 1/(1+a_1) + 1/(1+a_2) + ・・・・・・・ + 1/(1+a_n) ≧ n/(1+G),
 ここに、G = (a_1・a_2・・・・・・a_n)^(1/n),
 和書[8]、p.170 例題3.3.9 (10)
0092ななし垢版2013/10/22(火) 00:26:11.28
>>91

(1) n=1のときは明らかなので n≧2とする。
移項して分母を払うと
 {n + (n-1)x^n + y^n}{1+x・y^(n-1)} - n(1+x^n)(1+y^n)
 = {x・y^(n-1)}兩1 - 兩2
 ≧ 兩1 - 兩2   (← x・y^(n-1) ≧1)
 = (x-y)^2 Σ[k=0,n-2] (k+1){x^k・y^(n-2-k) - x^(n-2-k)・y^k}
 = (x-y)^3 Σ[k=0,n-3] (k+1)(n-2-k) x^k y^(n-3-k)
 ≧ 0,      (← x-y≧0)
ここに
 兩1 = (n-1)x^n - n・x^(n-1)・y + y^n
    = Σ[k=0,n-1] (x-y)(x^k - y^k)x^(n-1-k) ≧ 0,
 兩2 = x^n - n・x・y^(n-1) + (n-1)y^n
    = Σ[k=0,n-1] (x-y)(x^k - y^k)y^(n-1-k) ≧ 0,

(2) nについての帰納法で。

和書[8] のような解析的な方法もあるが....
0093132人目の素数さん垢版2013/10/25(金) 13:26:38.30
確率関連の不等式(Markovの不等式とか,Hoeffdingの不等式とか)が充実してる本やサイトってない?

どの本にも載ってなさげなんだが...
0095132人目の素数さん垢版2013/10/30(水) 12:23:26.57
>>64
キャスフィーの解答から....

 |t| > |tanh(t)|

 (d/dt)arcsin(t) = 1/√(1-tt),
 (d/dt)sinh(t) = cosh(t) = 1/√{1-tanh(t)^2},
から |t|≦1 のとき
 arcsin(t) > sinh(t) > 0,
が出る。
 t > sin(sinh(t)) > 0,

|t| ≧ 1 でもこの式が成立つことは明らか。

 sinh(t) = x とおくと
 arcsinh(x) > sin(x) > 0,
0096132人目の素数さん垢版2013/11/07(木) 11:41:28.21
キャスフィーから....

〔問題〕
A,B,C>0, ABC ≧1 のとき
 (A+1)/(A+B+1) + (B+1)/(B+C+1) + (C+1)/(C+A+1) ≦ 2,
 B/(A+B+1) + C/(B+C+1) + A/(C+A+1) ≧ 1,
 等号成立は A=B=C=1.

casphy - 高校数学 - 不等式2 - 028(5), 112
0097132人目の素数さん垢版2013/11/15(金) 01:38:28.47
幾何的な不等式でもよければ
幾何学大辞典にもけっこう載ってるよね
著者本人が見つけたやつもいっぱい出てるからチェックしてみるといいよ
0098ななし垢版2013/11/15(金) 15:47:56.27
キャスフィー! 不等式2 の じゅー さんへ。
 まづはWEBで....

「一般化固有値問題」(明治大)
http://www.math.meiji.ac.jp/~mk/labo/text/generalized-eigenvalue-problem/generalized-eigenvalue-problem.html

「極値としての固有値」(東京大)
http://www.misojiro.t.u-tokyo.ac.jp/~murota/lect-kisosuri/eigenmax070918.pdf

「§1固有値問題」(早稲田大)
http://www.waseda.jp/ocw/ComputerScience/17-1004345-01NumericalComputationsSpring2003/StudyMaterials/lec6.pdf

「Rayleigh商と2次形式の最大値,最小値」 - Quod Erat Demonstrandum
http://deepwave.web.fc2.com/rayleigh.pdf
0099ななし垢版2013/11/21(木) 00:38:50.46
同スレから....

(2) a,b,c >0 のとき
 (ab+bc+ca){1/(a+b)^2 + 1/(b+c)^2 + 1/(c+a)^2} ≧ 9/4,
 等号成立は a=b=c


(6) x,y,z >0 のとき
 xxx(yy+zz)^2 + yyy(zz+xx)^2 + zzz(xx+yy)^2
 - xyz{xy(x+y)^2 + yz(y+z)^2 + zx(z+x)^2} ≧ 0,

キャスフィ! - 高校数学 - 不等式2 - 126〜133
0100132人目の素数さん垢版2013/11/21(木) 01:47:49.32
ここで聞くのはスレチだとは思うがわかる人がいたら教えてほしい
以前は Live2ch で キャスフィ! を見れていたのだが
いつからか読み込めなくなった
同じ症状の人いない?
0101132人目の素数さん垢版2013/11/24(日) 14:28:07.16
>>99

〔Schur不等式の拡張〕
x,y,z≧0 で、x,y,z が△の三辺をなすとき
 x(a-b)(a-c) + y(b-c)(b-a) + z(c-a)(c-b) ≧0,
 (じゅー)

 キャスフィ! - 高校数学 - 不等式2 - 131〜132
0102132人目の素数さん垢版2013/11/24(日) 20:20:27.12
>>99

〔Schur不等式の拡張〕
x,y,z≧0 で、x,y,z が a,b,c と同順または逆順のとき
 x(a-b)(a-c) + y(b-c)(b-a) + z(c-a)(c-b) ≧0,
 (じゅー)

 キャスフィ! - 高校数学 - 不等式2 - 131〜132
0103132人目の素数さん垢版2013/11/26(火) 20:40:41.50
>>99 (6)
 yはxとzの中間にあるとしてよい。

 (左辺) - (右辺) = A(x-y)^2 + (A-B+C)(x-y)(y-z) + C(y-z)^2,
ここに
 A = yz(y^3 +z^3) + x(y-z)(y^3 -z^3) > 0,
 A-B+C = 2{(z+x)y - xz}y^3 ≧ 2{Max(x,z)min(x,z) - xz}y^3 = 0,
 C = xy(x^3 +y^3) + z(x-y)(x^3 -y^3) > 0,
より成立。

註) z+x > Max(x,z) > 0, y ≧ min(x,z) > 0, を辺々掛けた。
0104132人目の素数さん垢版2013/12/19(木) 20:54:29.76
>>99

(1) 
a,b,cは相異なる正の実数とする。
 ab・log(a/b)/(a-b) + cyc. ≦ (1/3)(√a + √b + √c)^2,
を示せ。log は自然対数です。


(8)
任意の正の実数a,b,cに対し
 a/√(a+b) + b/√(b+c) + c/√(c+a) ≦ (5/4)√(a+b+c),
 等号成立は (a,b,c) = (3,1,0) またはその rotation.

 キャスフィ! - 高校数学 - 不等式2 - 126
0106132人目の素数さん垢版2014/01/02(木) 01:31:00.06
>>104 (1)

x>0 のとき
 2Log(x)/{x -(1/x)} = (2t)/{exp(t) - exp(-t)}
 = t/sinh(t)
 ≦ 1,
より
 √(ab)・Log(a/b)/(a-b) = Log(a/b)/{√(a/b) - √(b/a)} ≦ 1,
よって
 (左辺) ≦ √(ab) + √(bc) + √(ca)
     ≦ (1/3)(√a + √b + √c)^2,
0107132人目の素数さん垢版2014/01/05(日) 07:45:41.88
↓の不等式うまい方法あるかな ?

a[i],b[i],c[i]>0および
a[i],{b[i]/c[i]}は減少列のとき
(Σa[i]b[i])/(Σa[i]c[i])≧(Σb[i])/(Σc[i])
0108132人目の素数さん垢版2014/01/05(日) 11:44:03.96
別にうまくないけど。
分数が嫌だから、b[i]/c[i]をあらためてb[i]として、式を整理すると、
(Σa[i]b[i]c[i])(Σc[i])≧(Σa[i]c[i])(Σb[i]c[i]).
これは (左辺)-(右辺)=Σ[i<j](a[i]-a[j])(b[i]-b[j])c[i]c[j]≧0 からいえる。
0109132人目の素数さん垢版2014/01/18(土) 19:41:47.51
〔問題158〕
 a,b,c >0,
 aa+bb+cc + abc = 2(ab+bc+ca),
のとき
 (ab+bc+ca) ≦ 3(a+b+c),
を示せ。

 キャスフィ! - 高校数学 - 不等式2 - 158,161
0110132人目の素数さん垢版2014/01/19(日) 12:23:09.17
〔問題163〕
0≦a,b,c≦1 のとき,
 2(a^3+b^3+c^3) ≦ 3 + aab+bbc+cca,
等号成立は (a,b,c) = (0,1,1) (1,0,1) (1,1,0) (1,1,1)

 キャスフィ! - 高校数学 - 不等式2 - 163,164
0111132人目の素数さん垢版2014/01/19(日) 20:56:00.76
>>109
 
条件式と aa+bb ≧ 2ab から
 (c+ab)c ≦ (2a+2b)c,
c>0 で割って
 ab ≦ 2a+2b -c,
循環的にたす。


>>110

 1 +aab -(aa +b) = (1-aa)(1-b) ≧ 0,
から
 (aa+bb+cc) + (a+b+c) ≦ 3 + aab+bbc+cca,
が出る。
0112132人目の素数さん垢版2014/02/04(火) 00:05:10.49
x+y=2のとき、1/(1+x^2) + 1/(1+y^2) のとりうる範囲は?

普通にやったら泥臭くて吐いた。
(通分してq=xyのみの式に直したもの=kとおき、分母を払って整理した
qの2次方程式がqのとりうる範囲内に少なくとも1つの解を持つ云々)

きれいな解法求む。
0113132人目の素数さん垢版2014/02/05(水) 21:27:47.56
>>112

 xx+yy = (x+y)^2 -2xy = 4-2q,

 1/(1+xx) + 1/(1+yy) = (2+xx+yy)/(1+xx+yy+xxyy)
 = (6-2q)/(5-2q+qq) = k,
よって
 -(√2 - 1)/2 < k ≦ (√2 + 1)/2,

 最大は q = (√2 - 1)^2 のとき
 最小は q = (√2 + 1)^2 のとき
0114132人目の素数さん垢版2014/02/05(水) 21:29:49.64
入試問題や模試や大学への数学などから持ってきますた。(じゅー)

[1]
 |z+1/2| < 1/2 のとき
 |1+z+…+z^n| < 1 を示せ。 (東工大前期)

[2]
xx+yy+zz=1 のとき
 (1) (x-y)(y-z)(z-x)
 (2) (2x-y)(2y-z)(2z-x)
の取りうる最大の値を求めよ。 (大数宿題)

[3]
a,b,c>0, a+b+c+abc=4 のとき
 a+b+c ≧ ab+bc+ca を示せ。(大学宿題)

[4]
(xx+yy)^2 = xx-yy のとき
x+y の取りうる最大の値を求めよ。(早大プレ)

キャスフィー! - 高校数学 - 不等式2 - 170
0115ななし垢版2014/02/05(水) 21:50:26.43
>>114 [3]

 a+b+c=s, ab+bc+ca=t, abc=u とおく。
 s<3 と仮定すると、相加-相乗平均で u<1 となり題意を満たさない。
∴ 3 ≦ s < 4,

 4s(s-t) = -s^3 +4ss -9u + F_1(a,b,c)
     = -s^3 +4ss -9(4-s) + F_1(a,b,c)
     = (4-s)(s-3)(s+3) + F_1(a,b,c)
     ≧ 0,
ここに
 F_1(a,b,c) = s^3 -4st +9u ≧ 0, (Schur)
0116ななし垢版2014/02/07(金) 20:38:22.91
>>114

[1] zは中心-1/2, 半径1/2 の円内にある。
  |z| ≦ |z +1/2| + 1/2 < 1,
また、|z|^2 = zz~ = (z +1/2)(z~ +1/2) -1/4 -(z+z~)/2 < -(z+z~)/2,
∴ |1 - z^(n+1)| ≦ 1 + |z|^(n+1)
  < 1 + |z|^2
  < √(1+3|z|^2)
  < √{1 -(z+z~) +zz~}
  = √{(1-z)(1-z~)}
  = |1-z|,

[2] (1) x≦y≦z とする。
 (x-y)(y-x) ≦ (1/4)(z-x)^2,
   等号は y=(x+z)/2 のとき。
 ∴ (x-z)^2 = 2{xx + [(x+z)/2]^2 + zz - (3/4)(x+z)^2}
       ≦ 2{(xx+yy+zz) - (3/4)(x+z)^2}
       = 2{1 - (3/4)(x+z)^2}
       ≦ 2,
   等号成立は (x,y,z) = (-1/√2, 0, 1/√2)
 (左辺) ≦ (1/4)(z-x)^3 ≦ 1/√2,
下限も同様に

[4] 軸を45゚回す。
 u = (x+y)/√2, v = (x-y)/√2,
与式は
 (uu+vv)^2 - 2uv = 0,
ここで、du/dv=0, とおくと、
 2v(uu+vv) -u = 0,
 u = 8v^3 = (√3)v,
 (u, v) = (±√{(3√3)/8}, ±√{(√3)/8})
 2(uu+vv) = u/v = √3,
0117ななし垢版2014/02/07(金) 21:06:57.69
>>114 [4]

連珠形とか、Jakob Bernoulli のレムニスケート(Lemniscate)
というらしい....
0118ななし垢版2014/02/09(日) 18:15:15.66
>>115

出題者によれば
”今のところ、対称性を崩さない綺麗な証明は見つかっていない。”

 Schur不等式にもそのまま言えそう...
0119115垢版2014/02/11(火) 22:14:41.65
>>118

F_1(a,b,c) = a(a-b)(a-c) + b(b-c)(c-a) + c(c-a)(c-b)
  = (ab+ca)(a-b)(a-c)/(b+c) + ・・・・・
  = ab{(a-b)(a-c)/(b+c) + (b-c)(b-a)/(c+a)} + ・・・・・・
  = {ab(aa-bb)^2 + bc(bb-cc)^2 + ca(cc-aa)^2}/{(a+b)(b+c)(c+a)}
  ≧ 0                 (じゅー)
0120132人目の素数さん垢版2014/02/11(火) 23:21:48.73
x^3+y^3+z^3=1 (x,y,z>0)の時
xxy+zzxの取りうる最大値を求めよ
(東進数学コンクール)

結局スマートな解法が思いつかないまま〆切を迎えてしまいました
0121132人目の素数さん垢版2014/02/12(水) 22:54:44.26
過去問でつが……

〔問題908〕
 正の実数 a,b,c に対して、次を示してくださいです。
 {(a+b+c)(aa+bb+cc)}^2 ≧ 27abc(a^3 +b^3 +c^3),

 2ch - 数学板 - 不等式スレ6 - 908
 キャスフィー! - 高校数学 - 不等式1 - 964
0122132人目の素数さん垢版2014/02/15(土) 12:56:22.22
120 Chebyshev kills it
We could see a Golden Section
0123132人目の素数さん垢版2014/02/16(日) 15:57:03.91
>>121

 (a+b+c)(aa+bb+cc) = S+p+q,
ここに
 S = aaa + bbb + ccc,
 p = aab + bbc + cca,
 q = abb + bcc + caa,

 (左辺) = (S+p+q)^2
   ≧ 9(Spq)^(2/3)
   ≧ 9(SS・27SU)^(1/3)    (← 補題)
   = 27Su,
ここに
  S = aaa + bbb + ccc,
  T = (ab)^3 + (bc)^3 + (ca)^3,
  U = u^3 = (abc)^3,


〔補題〕
 pq ≧ 3(3STU)^(1/3) ≧ 3√(3SU),

(略証)
 pq = (aab+bbc+cca)(abb+bcc+caa)
   = T + uS + 3uu
   ≧ 3(3STU)^(1/3)
   ≧ 3√(3SU),   {← T≧√(3SU)}
0124132人目の素数さん垢版2014/02/16(日) 16:05:17.43
>>123

〔補題〕
 pq ≧ T + 2√(3SU) ≧ 3√(3SU),

(略証)
 pq = (aab+bbc+cca)(abb+bcc+caa)
   = T + uS + 3uu
   ≧ T + 2√(3SU)
   ≧ 3√(3SU),   {← T≧√(3SU)}
0127132人目の素数さん垢版2014/02/20(木) 20:58:32.81
〔問題179〕
x,y,z>0、xyz=1 のとき、
[Easy]
 xx+yy+zz ≧ 3 + 2(x-1)(y-1)(z-1),
[Hard]
 xx+yy+zz ≧ 3 + 2(1-x)(1-y)(1-z),
を示してくださいです。
[Hard] は [Easy] と比較して難しいかなぁって感じでつ。
                  (じゅー)

 キャスフィー! - 高校数学 - 不等式2 - 179
0128ななし垢版2014/02/20(木) 21:12:45.27
>>127

Easy の方は
 (左辺)−(右辺) = (x+y+z+1)(x+y+z-3) - 2(xyz-1) ≧ 0 だが...
0129ななし垢版2014/02/21(金) 19:15:17.77
>>127

 x+y+z = s,
 xy+yz+zx = t,
 xyz = u,
とおくと Hard の方は
 (左辺)−(右辺) = (ss-2t) -3 +2(u-t+s-1)
   = (ss-4t) +2(u-1) +2s -3
   = {F_1(x,y,z) -9u}/s + 2(u-1) +2s -3
   = {F_1(x,y,z) +(2s-9)(u-1) +(s-3)(2s+3)}/s
   ≧ 0,           (天ぷら)
ここに
 F_1(x,y,z) = x(x-y)(x-z) + y(y-z)(y-x) + z(z-x)(z-y)
   = sss -4st +9u ≧ 0, (Schur)
0132132人目の素数さん垢版2014/03/01(土) 12:30:14.33
  ∧_∧
  ( ;´∀`) <興奮してきた…
  人 Y /
 ( ヽ し
 (_)_)
0133132人目の素数さん垢版2014/03/01(土) 18:40:56.44
>>131 (2014年JMO本選)

〔問題5.〕
不等式
 a/{1+9bc+4(b-c)^2} + b/{1+9ca+4(c-a)^2} + c/{1+9ab+4(a-b)^2} ≧ 1/2,
が a+b+c=1 をみたす任意の非負実数a,b,cに対して成り立つことを示せ。
0135132人目の素数さん垢版2014/03/05(水) 21:41:46.78
>>133

a+b+c=s のとき、コーシーにより、
 {a[ss+9bc+4(b-c)^2] + b[ss+9ca+4(c-a)^2] + c[ss+9ab+4(a-b)^2)]}(左辺)
  ≧ (a+b+c)^2 = ss,
よって
 (左辺) ≧ ss/{sss+27abc+4[s(ab+bc+ca)-9abc]}
  = ss/{sss +4s(ab+bc+ca) -9abc}
  ≧ ss/(2sss)  (← Schur)
  = 1/(2s),    (じゅー)
0136132人目の素数さん垢版2014/03/09(日) 23:29:23.76
この『じゅー』って今年阪大挑戦枠受かった子?
0139132人目の素数さん垢版2014/03/10(月) 08:24:33.61
阪大の合格発表見たけど
数学挑戦枠の合格者一人だけだった
去年に引き続きなかなかエグい試験だったってことだな
0141prime_132垢版2014/03/15(土) 21:42:02.66
いちょう祭が楽しみ...♪

ちなみに小生は学科違いの S53入、S59院卒 だが何か?
0145ななし垢版2014/03/24(月) 20:20:36.02
>>143

s,t,u を >>115 のようにおく。
 t<3 と仮定すると、相加-相乗平均で u<1 となり題意を満たさない。
∴ 3 ≦ t < 4,

s≧4 のときは明らか。
s<4 のとき
 (4s+9)(s-t) = -s^3 +4ss +9(s-t-u) + F_1(a,b,c)
   = -s^3 +4ss +9(s-4) + F_1(a,b,c)
   = (4-s)(s-3)(s+3) + F_1(a,b,c)
   ≧ 0,    (← s≧√(3t)≧3)
ここに
 F_1(a,b,c) = s^3 -4st +9u ≧ 0, (Schur)
0146132人目の素数さん垢版2014/03/29(土) 01:38:18.83
>>120

相加-相乗平均
 axxx + axxx + yyy ≧ (1/k)xxy,
 (1-2a)xxx + zzz/2 + zzz/2 ≧ (1/k)xzz,
を辺々たすと
 xxx+yyy+zzz ≧ (1/k)(xxy+xzz),

係数を比べて、
 aa = (1-2a)/4 = 1/(3k)^3,
aを消すと、
 k = (1/3)(1+√5)^(2/3) = 0.729273617

casphy - highmath - 不等式2 - 173-174
//twitter.com/Inequaltybot/ [181]
0147132人目の素数さん垢版2014/04/01(火) 22:21:02.29
正数x,y,zが xyz = 1 のとき
 x^3 + y^3 + z^3 + 1/x^3 +1/y^3 + 1/z^3 - 6*( x/z + y/x + z/y ) +12 ≧0


って成り立ちますか?
0149132人目の素数さん垢版2014/04/05(土) 21:17:36.82
正変数a_1, a_2, ・・・, a_n について A_n = (a_1+a_2+・・・+a_n)/n , G_n = (a_1*a_2*・・・*a_n)^(1/n) とするとき

 n(A_n-G_n) ≧ (n-1)(A_(n-1) - G_(n-1))

が成り立つそうなのですがどう示されるのでしょう
0150132人目の素数さん垢版2014/04/07(月) 20:05:40.74
(a^4+b^4+c^4)^3≧(a^3+b^3+c^3)^4ってどうやって示せばいいんだっけ
0151132人目の素数さん垢版2014/04/07(月) 20:26:10.23
頭わるそうなやり方だけどlog取って12で割って増減調べりゃいいんじゃない
0154132人目の素数さん垢版2014/04/14(月) 01:07:10.40
〔問題〕
a,b,c>0 に対して、次を示してくださいです。
 (a+b+c)^2・(a+b)(b+c)(c+a) ≧ 24abc(aa+bb+cc),

 //twitter.com/Inequalitybot/ [196]
0157132人目の素数さん垢版2014/05/04(日) 03:05:01.23
 2sinθ + tanθ > 3θ,
これは Snellius-Huygensの不等式として知られている。

この不等式で θ= π/4 - π/6 = π/12 として
 sinθ = sin(π/4 -π/6) = (√3 -1)/(2√2),
 tanθ = tan(π/4 -π/6) = 2-√3,
を使えば
 4{(√3 -1)/√2 +(2-√3)} > π,

√2 + √3 = 4{(√3 -1)/√2 +(2-√3)} + (√2 -1)^2・(2-√3)^2・(√3 -√2)
> 4{(√3 -1)/√2 +(2-√3)}
> π,
                  ぬるぽ
0158132人目の素数さん垢版2014/05/04(日) 03:24:20.18
>>149
a_n = a と略記する。
n・A_n = (n-1)A_(n-1) + a,
n・G_n = n・{G_(n-1) ^(n-1)・a}^(1/n)
 ≦ (n-1)G_(n-1) + a, (←相乗・相加平均)
辺々引く。
                ぬるぽ
0160132人目の素数さん垢版2014/05/07(水) 00:25:04.63
>>143-145

(別解)
a,b,c の2つが1以上、または2つが1以下。
a,b をその2つとすると
 4 = (a+b+c) + abc = (a+b)(1+c) + (1-a)(1-b)c ≧ (a+b)(1+c),
 (a+b+c) - (ab+bc+ca) = (a+b){4-(a+b)(1+c)}/4 +(a-b)^2・(1+c)/4 +(1-a)(1-b)c ≧0,

//www.casphy.com/bbs/highmath/ 不等式2 - 170[3] 〜172
//twitter.com/Inequalitybot/ [184]
0161132人目の素数さん垢版2014/05/12(月) 21:26:52.73
> 154
f(a,b,c) = (a+b+c)^2 (a+b)(b+c)(c+a) - 24a b c(a^2+b^2+c^2)
とおく。x ≧ 0 のとき、
f(x,1,0) = x (1 + x)^3 ≧ 0
f(x,1,1) = 2(x+1)^2(x-2) ≧ 0
よって、安藤哲哉「不等式」定理2.3.1(2)より f(a,b,c) ≧ 0.

ところで、同書の記号で
f(a,b,c) = T_{4,1} + 3T_{3,2} + 10 US_{1,1} - 18 US_2
なので、定理2.4.1が使えない。そこで、次の定理を提案する。

定理2.4.1b
f(a,b,c) = T_{4,1} + p T_{3,2} + q US_2 - (2+2p+q) US_{1,1}
とおく。任意の非負実数 a,b,c に対して f≧0 となるための必要十分条件は
次の(1)と(2)が成立することである。
(1) p ≧ -1
(2) 2p+q+4 ≧ 0 または (2p+q)^2 + 8q ≦ 0
0162132人目の素数さん垢版2014/05/12(月) 21:27:39.56
> 154
f(a,b,c) = (a+b+c)^2 (a+b)(b+c)(c+a) - 24a b c(a^2+b^2+c^2)
とおく。x ≧ 0 のとき、
f(x,1,0) = x (1 + x)^3 ≧ 0
f(x,1,1) = 2(x+1)^2(x-2) ≧ 0
よって、安藤哲哉「不等式」定理2.3.1(2)より f(a,b,c) ≧ 0.

ところで、同書の記号で
f(a,b,c) = T_{4,1} + 3T_{3,2} + 10 US_{1,1} - 18 US_2
なので、定理2.4.1が使えない。そこで、次の定理を提案する。

定理2.4.1b
f(a,b,c) = T_{4,1} + p T_{3,2} + q US_2 - (2+2p+q) US_{1,1}
とおく。任意の非負実数 a,b,c に対して f≧0 となるための必要十分条件は
次の(1)と(2)が成立することである。
(1) p ≧ -1
(2) 2p+q+4 ≧ 0 または (2p+q)^2 + 8q ≦ 0
0163132人目の素数さん垢版2014/05/13(火) 05:50:13.76
ついでに、S_5 と T_{4,1} の係数が 0 の場合は以下の通りです。

定理2.4.1c
f(a,b,c) = T_{3,2} + q US_2 - (2+2p+q) US_{1,1}
とおく。任意の非負実数 a,b,c に対して f≧0 となるための必要十分条件は
q ≧ -2

ここで(上の投稿を含めて)
T_{i,j} = Σ a^i b^j (6項対称和)
S_{i,j} = Σ a^i b^j (3項巡回和)
U = abc
0164132人目の素数さん垢版2014/05/28(水) 03:56:36.47
B4638、B4640、http://www.komal.hu/verseny/feladat.cgi?a=honap&;h=201405&t=mat&l=en

A616、B4626、B4628、http://www.komal.hu/verseny/feladat.cgi?a=honap&;h=201404&t=mat&l=en

B4620、http://www.komal.hu/verseny/feladat.cgi?a=honap&;h=201403&t=mat&l=en

A609、B4606、http://www.komal.hu/verseny/feladat.cgi?a=honap&;h=201402&t=mat&l=en

A605、http://www.komal.hu/verseny/feladat.cgi?a=honap&;h=201401&t=mat&l=en

B4585、http://www.komal.hu/verseny/feladat.cgi?a=honap&;h=201312&t=mat&l=en

A593、http://www.komal.hu/verseny/feladat.cgi?a=honap&;h=201309&t=mat&l=en

C1168、http://www.komal.hu/verseny/feladat.cgi?a=honap&;h=201304&t=mat&l=en


_| ::|_
 ̄| ::|/|           ┌──┐
  | ::|  |     .┌──┐| ∧_∧  いいな、俺たちの誰かが殉職したら・・
/|_|  |┌──┐| ∧_∧|(・ω・` )
  |文|  | | ∧_∧(    )⊂   )
  | ̄|  | | (    )⊂   ) (_Ο Ο :::
  | ::|  | | ⊂   ) (_Ο Ο わかってる、生き延びた奴が
  | ::|/ .|_ (_Ο Ο ::::::::: :::::: 不等式を収集し、証明する !
  | ::| :::::::::::::::::::::::::::::::: 俺たちゃ死んでも仲間だぜ !!
0166132人目の素数さん垢版2014/06/27(金) 23:17:34.18
>>165
ジュンク堂にあったので買ってきた。

大学入試問題から題材を取っているので、
このスレの不等式の囚人どもには目新しいものはないけど、
考え方や知識の整理にはちょうどよいかな。
おすすめ。

>>2 に追加
[9] 思考力を鍛える不等式(大学への数学・別冊)、栗田哲也、東京出版、2013
0167132人目の素数さん垢版2014/07/06(日) 07:21:38.15
> このスレの不等式の囚人どもには目新しいものはないけど
目新しいものがほしい人には、
http://www.math.s.chiba-u.ac.jp/~ando/alg_ineq.pdf
の定理2.4.1c〜命題2.4.1f はいかがですか。
なお >162, 163の定理2.4.1a, bは命題2.4.1g, hに変更して証明も変えました。
0168132人目の素数さん垢版2014/07/17(木) 02:05:03.70
質問なんだが立体の一つの頂点に集まる角度の総和が360゚未満ってどう証明したらいい?
例えば立方体だと90+90+90=270
0171132人目の素数さん垢版2014/07/17(木) 04:04:17.35
すまん、読んでみたけど…
0173132人目の素数さん垢版2014/07/17(木) 04:21:50.42
リーマンの不等式、またの名をリーマンの半分age
0174◆2VB8wsVUoo 垢版2014/07/17(木) 05:30:15.28


>6 名前:KingMathematician ◆LoZDre77j4i1 :2014/07/15(火) 20:00:03.07
> [>>1]の親は強制的に[>>1]を集団から隔離するべし.
>
>660 名前:KingMathematician ◆LoZDre77j4i1 :2014/07/15(火) 20:02:50.12
> Re:>>658 (10+a)(10+b)=100+10(a+b)+ab.
>
0175132人目の素数さん垢版2014/07/31(木) 14:46:29.19
分数の不等式とかを証明するときに、項ごとに評価してそれらを加えたらできた!ってのを見たことがあるけど、
そんなやりかたで証明するときって、どうやって気づくんだろうか?
0180132人目の素数さん垢版2014/08/10(日) 23:37:22.74
関数f(x)は導関数f’(x)および第2次導関数f’’(x)をもち,
区間0 ≦ x ≦ 1において, f(x)>0, {f’(x)}^2 ≦ f(x)f’’(x) ≦ 2 {f’(x)}^2 を満たしている
f(0)=a,f(1)=bとするとき,次の不等式を示せ.
(1)f ( 1/2 ) ≦ (a+b)/2
(2)f ( 1/3 ) ≦ (a^2b)^(1/3)
(3)f ( 1/4 ) ≧ (4ab)/(a+3b)
(4)∫[0^1]f(x) dx ≦ (a/4)+(√ab/2)+(b/4)
0182132人目の素数さん垢版2014/08/11(月) 10:35:16.82
悪代官「わしは、何より不等式が好きでの」
越後屋「あはは、不等式はかの色に限りますなあ」
悪代官「はてさて、かの色とな?」
越後屋「今回はかように取り揃えました」
悪代官「ほー、今回はまた一段と」
越後屋「お目に叶ってなによりでwwwww」
0183132人目の素数さん垢版2014/08/14(木) 02:57:28.50
x、y、z > 0 のとき、xyz/{(xy+2)(2yz+3)(3yz+1)} のとりうる値の範囲を求めよ。

( ゚∀゚)プケラッチョ!
0185132人目の素数さん垢版2014/08/14(木) 17:53:02.88
>>183,184
x、y、z > 0 のとき、xyz/{(xy+2)(2yz+3)(3zx+1)} のとりうる値の範囲を求めよ。
の書き間違えじゃないか?
0186132人目の素数さん垢版2014/08/14(木) 18:33:21.25
(xy+2)(2yz+3)(3zx+1)≧48xyz 等号はx=2/3, y=3, z=1/2の時成立
xyz→+0の時0に収束
ゆえに
0<xyz/{(xy+2)(2yz+3)(3zx+1)}≦1/48
0187132人目の素数さん垢版2014/08/14(木) 20:13:24.03
>>185
( ゚∀゚) スミマセン、ご指摘の通り、分母の3つ目はyzじゃなくてzxですた。

>>186
正解です。エレガントなやり方ありましたか?
0191132人目の素数さん垢版2014/08/16(土) 00:34:06.59
>>189
今年の滋賀医科大の入試問題らしい
0192132人目の素数さん垢版2014/08/16(土) 01:36:17.24
>>190
(a1^2 + a2^2 + … + an^2)(a2^2 + … + an^2+a1^2)≧(a1a2+a2a3+…ana1)^2
より
(a1^3/a2 + a2^3/a3 + … + an^3/a1)(a1^2 + a2^2 + … + an^2)
≧(a1^3/a2 + a2^3/a3 + … + an^3/a1)(a1a2+a2a3+…ana1)
≧(a1^2 + a2^2 + … + an^2)^2

(コーシー)
0194132人目の素数さん垢版2014/08/16(土) 20:43:09.24
>>188
f"≧0
(logf)"=(f'/f)'=(ff"-f'f')/ff≧0
(1/f)"=(-f'/ff)'=(2f'f'-ff")/fff≧0
より
(1)f(1/2)≦(1/2)(f(0)+f(1))
(2)logf(1/3)≦(2/3)logf(0)+(1/3)logf(1)
(3)4/f(4)≦3/f(0)+1/f(1)
(Jensen)
これらを整理する
(4)は凸性ゆえ
(左辺)≦(1/4)(f(0)+f(1/2))+(1/4)(f(1)+f(1/2))≦(右辺)
最後はJensenを用いた
0195132人目の素数さん垢版2014/08/20(水) 20:33:15.56
なんでニコニコ大百科にウィキペディアより詳細なシュールの不等式の記事があるんだよ
0198132人目の素数さん垢版2014/08/21(木) 22:09:46.08
こんなんja.wikipediaに書いたら
wikipediaは数学書じゃないから証明の必要性が云々とか
独自研究ガーとかいう奴が出て来て全削除とかされかねないもんな
0199132人目の素数さん垢版2014/08/24(日) 15:43:56.20
>>179
いま気づいたがこれは大数の学コンの問題の一部だな
このカス野郎が
そんなにまでして良い点とりたいか?
0200132人目の素数さん垢版2014/08/24(日) 22:30:33.72
この程度で苦労するなら良い点なんか取れないって
0203132人目の素数さん垢版2014/08/25(月) 05:39:36.69
nを定まった正の整数とし,1≦k≦nなる整数kのおのおのに,1≦r≦nなる整数rを対応させる関数r=f(k)があって
k[1]<k[2]ならばつねにf(k[1])≦f(k[2])であるとする
このとき,f(m)=mとなる整数mが存在することを証明せよ
0204132人目の素数さん垢版2014/08/25(月) 13:47:47.28
離散写像における不動点定理ですね

f(1)>1,f(n)<nの時のみ考えれば良い
f(i)>iなる最大のiをkとしてk<f(k)≦f(k+1)<k+1
これはkとk+1の間に整数があることになるため矛盾
0205132人目の素数さん垢版2014/08/26(火) 17:43:33.11
>>197
詳し過ぎて吹いたw
0209132人目の素数さん垢版2014/09/03(水) 21:33:34.29
>>208
b=dとして
c=1/aとしてa→∞とすると前者>後者
c=2としてa→∞とすると前者<後者

だから定まらなさそう
0210132人目の素数さん垢版2014/09/03(水) 21:52:25.87
ありがとうございます。
書き込んだ後、計算してみて、私も定まらないなと思いました。
何を考えていたかを説明すると…

(a^2+b^2+1)(c^2+d^2+1)≧(ac+bd+1)^2
(a^2+b^2+1)(c^2+d^2+1)≧(a-c)^2 + (b-d)^2

を眺めていて、右辺に大小関係が定まらないかなぁと考えていたら、実際に計算してみると、

(a^2+b^2+1)(c^2+d^2+1) = (a-c)^2 + (b-d)^2 + (ad-bc)^2 + (ac+bd+1)^2  ← 『不等式ヲタの等式』と命名

なので、ダメじゃんってな感じです。
0213132人目の素数さん垢版2014/09/04(木) 09:06:07.32
自力で見つけたんだからスゴイんじゃね
ちな本来の四平方和の公式はもっと一般的な
0214132人目の素数さん垢版2014/09/05(金) 00:18:44.18
実数 a、b、c、d、e、f が、次式をみたしている。
  a-b > b-c > c-d >d-e > e-f > f-a
a、b、c、d、e、f のうち最大の実数はどれか?


きたか…!!

  ( ゚д゚ ) ガタッ
  .r   ヾ
__|_| / ̄ ̄ ̄/_
  \/    /
0219132人目の素数さん垢版2014/09/10(水) 13:13:05.60
今月の東進の問題
〆切は過ぎてる

ttp://www.toshin.com/concours/img/mondai_18.jpg

高校生用としてはかなり難しいのでは?
ここの住人なら瞬殺?
0221132人目の素数さん垢版2014/09/10(水) 20:10:14.28
>>219
a=cotA, b=cotB, c=cotCとするとA,B,Cは三角形の三辺をなす
f(x)=1/(1-cosx)は凸関数のため
log(左辺)=f(A)+f(B)+f(C)≧3f(π/3)=3log2 (Jensen)
両辺の真数をとる
0227132人目の素数さん垢版2014/09/11(木) 23:12:47.63
こんなの解ける東進生がどのくらい居るのか甚だ疑問
まあ高校生に、不等式は超絶テクニックの職人の世界だぜ、
と知らしめてやるような意味はあるが
0228132人目の素数さん垢版2014/09/11(木) 23:35:51.28
若いうちに洗脳して不等式ヲタを増やすことも、我々の大事な仕事なのです。

>>221
√(a^2+1) があるので、a = tanA とか置き換えたくなるけど、 cotAとは思いつかんなぁ

いろいろな不等式を収集し証明するのは、今後も続けるとして、
そろそろ、不等式証明の定番手法や考え方を語る時期では?
0229132人目の素数さん垢版2014/09/11(木) 23:50:54.66
あ、でも値を評価するとはどういうことか、
ということを教えるのは数学的に本質的に大事だね

等式の世界と不等式の世界は全然違うからね
0230132人目の素数さん垢版2014/09/12(金) 00:15:14.25
その値を求めるのは大変だから、大体どのくらいかを評価することが必要になって…
そんな感じ?

不等式の美しさにばかり囚われていて、あまり考えたことなかったな。
0231132人目の素数さん垢版2014/09/12(金) 00:20:34.32
x、y、z >0 のとき、xyz/{(xy+2)(2yz+3)(3zx+1)} の最大値を求めよ。

これだと、『x、y、z >0 のとき』 がヒントになって、相加相乗平均でも使うんだろうなと思ったり。
確かに相加相乗平均で肩がつく (最大値は1/48) 。

この手の分数の不等式は、分割して評価した不等式を辺々かけるとか、
斉次式なので、和一定とか積一定とかの条件をつけたりするテクもあったり。

不等式スレ1章〜3章の頃に、その手のテクニックをtexでまとめていたんだけど、
パソコンが壊れたときに失われてしまったのは悲しい思い出… ('A`)
0233132人目の素数さん垢版2014/09/12(金) 18:34:10.65
a、b、c、x、y、z ∈R が、
 (a-1)^2 + (b-2)^2 + (c-3)^2 = 1、 x^2+y^2+z^2=1
をみたすとき、ax+by+czのとりうる値の範囲

CS不等式を2回使ったけど、他の解法ありますか?
0235132人目の素数さん垢版2014/09/15(月) 17:30:44.27
【結構簡単だけど、受験生40名全員が解けなかったという曰く付きの某国立大学数学科の推薦入試問題】


実n次元ベクトル X = (x_1, x_2, ... , x_n) に対して
h を非負実数として
<X>_h = √{h + (x_1)^2 + (x_2)^2 + ... + (x_n)^2} と定義する。
Y = (y_1, y_2, ... , y_n) とするとき、 任意の実n次元ベクトル X、Y に対して
<X + Y>_h ≦ <X>_h・<Y>_h
が成り立つような最小の h を求めよ。
ただし、X + Y= (x_1+y_1, x_2+y_2, ... , x_n+y_n) とする。

注) 伝聞によるので文言は全く同じではないと思われます
0236132人目の素数さん垢版2014/09/18(木) 00:25:23.04
麻呂ちゃんを救う会
┏━━━━━━━━━━┓
┃               ┃ 麻呂ちゃんは
┃...|;:;:_:;:__:;_:;_:;:l:;_;:_:_:;:_;:_:;| ┃ 生まれつき不等式収集の病気にかかり
┃|_____|_____|...┃ 一カ月以内に不等式が必要です。
┃|=|  三シノ ヾ三. ::::::.|=! ┃
┃|=| (●), 、(●)、::|=| ┃ しかし、それにはハァハァできる
┃ヾ|  ,,ノ(、_, )ヽ、,,  シノ...┃ 特殊な不等式が必要となります。
┃ |   ,;‐=‐ヽ   .:::::| ...┃
┃ \  `ニニ´  .:::/ .....┃ 麻呂ちゃんを救うために
┃ /`ー‐--‐‐―´´\. ...┃ どうか協力をよろしくお願いします。
┗━━━━━━━━━━┛
0237132人目の素数さん垢版2014/09/18(木) 00:26:03.05
| 寝ていて
| 夜 目を醒ますと          ____
| 不等式がスレにない       /;:;:;:;:;:;i;:;:;:;:;:丶
|_________/     /:;:;:;:;:;:;:;:;:i;:;:;:;:;:;:;:;:;\
                  |;:;:_:;:_:;:_:;:_;:l:;_;:_:;:_:;:_:;:_|
                  |____|____|
               // |彡  ≡    ≡  ミ|
              (  ( (6  <○)   (○> 9)
              ノ ヽ \ |    。⌒。   |          //
             イ 人 \ \   ┌-┐  ノ      //
           / λ   ヽ   ` 、/)_ ̄__ノ ̄`,    //
          ζ (     ヾ  \ ̄ ̄ ̄ ̄ ̄Uι)//
                             ̄- //

    1〜2歳の麻呂にとってそれはどんな恐怖と絶望
     なのだろう … … 麻呂は暗闇の中で泣いても
    無駄なのでただひたすらふるえていただけだった
0238132人目の素数さん垢版2014/09/18(木) 00:26:30.63
         |;;;;;;;;;;;;;;;;;;;;;;;;;;;;;;;;;;;;;;;;;;;;;;;;;;;;;;;;;;;;;;;;;;;ノ|
         |丶、 ;;; __;;;;;;;;;;;;;;;;;;;;;;;;;;;;;;;;;;;;;;_,,: ィ";;_|
         ト、;;;;;;;;;;;;;;;` ` '' ー -- ‐ '' ";;;;;;;;;,:ィ;:;!
        ,';:``' ‐ョ 、 ,_ ;;;;;;;;;;;;;;;;;;;;;;;;;;; , - '"l;:;:;:;:l   不等式を貼るだけなら三流
        l;:;:;:;:;:;:;ミ   ` ` '' ー -‐ '"    ,リ;:;:;:l
        l;:;:;:;:;:;:;:ゝ   く三)   (三シ  `ヾ;:t、
       fミ{;:;:;:;:f'´  , -−-_,, _,ィ 、_,,ィ,.-−、  };f }  言われてから不等式を貼れて二流
       l トl;:;:;:;:l  、,ィ或tュ、゙:ミ {,'ィt或アチ l:l,/
       ゙i,tヾ:;:;:!  `ヽ 二ノ   ト ` ‐''"´  l:l:f
        ヽ`ー};:l       ,r'、   ヽ      リ_)  言われる前に自分から不等式を貼れてようやく一流じゃ
         `"^l:l      ,/゙ー、  ,r'ヽ    l
           ゙i    ,ノ    `'"  丶.   ,'
             ゙l、   ′ ,, ィrェェzュ、,_ 〉 } /
            ',ヽ  ヘヾ'zェェェッ',シ' //ヽ     
             } 丶、 ` ー--‐ '"'´,/ノ:.:.:ヽ     ・・・・そなたらは一体、いつになったら
            /l   丶、      ,.イ:.:.:.:.:.:.:.:丶、、         
          ,r'"^l !    ` ー‐;オ´:.:.:.:.:.:.:.:.:.,ノ  ,}、        一流になるのでおじゃるか?
     ,. -ァ=く(:.:.:.l  l      //:.:.:.:.:.:., - '"  ,/ ヽ、
  , - '"´ / ,/`>'t、_」___,ィ'゙,ィ,.: -‐ '" ,. -‐ '"    \
0239132人目の素数さん垢版2014/09/18(木) 00:28:07.44
                l三`ー 、_;:;:;:;:;:;:j;:;:;:;:;:;:_;:;:;_;:-三三三三三l
               l三  r=ミ''‐--‐';二,_ ̄    ,三三三彡彡l_   この感じ・・・・
              lミ′   ̄    ー-'"    '=ミニ彡彡/‐、ヽ
                  l;l  ,_-‐ 、    __,,.. - 、       彡彡彳、.//  不等式か・・・・
_______∧,、_‖ `之ヽ、, i l´ _,ィ辷ァ-、、   彡彡'r ノ/_ ______
 ̄ ̄ ̄ ̄ ̄ ̄ ̄'`'` ̄ 1     ̄フ/l l::. ヽこ~ ̄     彡彳~´/  ̄ ̄ ̄ ̄ ̄ ̄
                 ヽ   ´ :l .l:::.         彡ィ-‐'′
                ゝ、  / :.  :r-、        彡′
              / ィ:ヘ  `ヽ:__,ィ='´        彡;ヽ、
          _,,..-‐'7 /:::::::ヽ   _: :_    ヽ      ィ´.}::ヽ ヽ、
      _,-‐'´    {  ヽ:::::::::ヘ `'ー===ー-- '   /ノ /::::::ヘ, ヽー、
0240132人目の素数さん垢版2014/09/18(木) 00:32:24.97
  \
:::::  \            麻呂の両腕に冷たい鉄の輪がはめられた
\:::::  \
 \::::: _ヽ __   _     外界との連絡を断ち切る契約の印だ。
  ヽ/,  /_ ヽ/、 ヽ_
   // /<  __) l -,|__) > 「刑事さん・・・、俺、どうして・・・
   || | <  __)_ゝJ_)_>    不等式なんて蒐集・・・しちゃったのかな?」
\ ||.| <  ___)_(_)_ >
  \| |  <____ノ_(_)_ )   とめどなく大粒の涙がこぼれ落ち
   ヾヽニニ/ー--'/        震える彼の掌を濡らした。
    |_|_t_|_♀__|
      9   ∂        「その答えを見つけるのは、お前自身だ。」
       6  ∂
       (9_∂          麻呂は声をあげて泣いた。
0242132人目の素数さん垢版2014/09/18(木) 14:03:46.19
0 < x < y < π/2 において、f(x) = (tanx/x)^x - (sinx/x)^x が単調増加であることを示せばよい。
これは、y=sinx、y=tanxのグラフを描けば明らか。

でも計算で示そうとして挫折… ('A`)
0244132人目の素数さん垢版2014/09/21(日) 03:12:14.29
         |;;;;;;;;;;;;;;;;;;;;;;;;;;;;;;;;;;;;;;;;;;;;;;;;;;;;;;;;;;;;;;;;;;;ノ|
         |丶、 ;;; __;;;;;;;;;;;;;;;;;;;;;;;;;;;;;;;;;;;;;;_,,: ィ";;_|
         ト、;;;;;;;;;;;;;;;` ` '' ー -- ‐ '' ";;;;;;;;;,:ィ;:;!
        ,';:``' ‐ョ 、 ,_ ;;;;;;;;;;;;;;;;;;;;;;;;;;; , - '"l;:;:;:;:l   出された不等式を証明するだけなら三流
        l;:;:;:;:;:;:;ミ   ` ` '' ー -‐ '"    ,リ;:;:;:l
        l;:;:;:;:;:;:;:ゝ   く三)   (三シ  `ヾ;:t、
       fミ{;:;:;:;:f'´  , -−-_,, _,ィ 、_,,ィ,.-−、  };f }  自分で不等式を作って証明して二流
       l トl;:;:;:;:l  、,ィ或tュ、゙:ミ {,'ィt或アチ l:l,/
       ゙i,tヾ:;:;:!  `ヽ 二ノ   ト ` ‐''"´  l:l:f
        ヽ`ー};:l       ,r'、   ヽ      リ_)  不等式を作る前に手の者に作らせて証明させてようやく一流じゃ
         `"^l:l      ,/゙ー、  ,r'ヽ    l
           ゙i    ,ノ    `'"  丶.   ,'
             ゙l、   ′ ,, ィrェェzュ、,_ 〉 } /
            ',ヽ  ヘヾ'zェェェッ',シ' //ヽ     
             } 丶、 ` ー--‐ '"'´,/ノ:.:.:ヽ     ・・・・そなたらは一体、いつになったら
            /l   丶、      ,.イ:.:.:.:.:.:.:.:丶、、         
          ,r'"^l !    ` ー‐;オ´:.:.:.:.:.:.:.:.:.,ノ  ,}、        一流になるのでおじゃるか?
     ,. -ァ=く(:.:.:.l  l      //:.:.:.:.:.:., - '"  ,/ ヽ、
0246132人目の素数さん垢版2014/09/28(日) 09:55:02.94
ガンマ関数の問題なんだが、

Γ(1/a) < π

aを求めよ。

aって求められないよね?
つか、問題が間違ってるよね?ね?
0247132人目の素数さん垢版2014/09/28(日) 14:23:20.23
aが実数または整数という条件は?
それでwikipediaのガンマ関数のグラフでも見たら。。。
0249132人目の素数さん垢版2014/10/07(火) 04:40:12.83
nを自然数、xを正の実数とするとき、n(x-1) ≦ x^n-1 ≦ n(x-1)x^n ≦ n(x-1)x^n

( ゚∀゚) プ゚ウ
ノヽノ) =3'A`)ノ ヒャー
  くく へヘノ
0250132人目の素数さん垢版2014/10/10(金) 22:50:17.76
死ねよ
0251132人目の素数さん垢版2014/10/14(火) 21:10:08.93
今日、不思議な定理を発見しました。
斉次巡回(or 対称)不等式に関するものですが、
代数多様体の商特異点の理論を経由して、
不等式のクラスがなす凸錐の構造定理を証明することによって、
古典的な方法で直接不等式の成立を示さなくても、抽象的議論だけで
間接的にいろいろな不等式の成立が証明できてしまう、という、
ちょっと気味の悪いものです。
個々の不等式を見るのではなく、ある条件を満たす不等式のクラスを、
まとめて面倒みてやるのがポイントです。
モジュライの理論でもそういう考え方が登場します。
多変数の代数不等式は、やっぱり代数幾何だったんですね。
普遍性がある議論なので、いろいろ応用が利きそうです。
そのうち、どこかで、きちんとお話します。
0254132人目の素数さん垢版2014/10/23(木) 08:25:42.36
予告編:
X を不等式 f(x_1,...,x_n)>=0 の変数が動く領域とし、
そこに有限群 G が推移的に作用していて、固定点は有限個であると仮定する。
D を G-不変な X 上の不等式全体がなす凸錐とする。
線形系 H^0(P^{n-1}, O(d))^G が定める有理写像 P^{n-1} -> P^N に
よる X の像を X_d とする。d がある程度大きいと、X_d = X/G となる。
X_d の生成する凸錐の相対凸錐が D である。
X_d の内部、境界、内部の各特異点、境界上の各特異点に対応して、
D の境界の成分が定まり、D はそれらで囲まれる。
代数的にD の境界の成分を求めると、不等式が自動的に得られる。
たぶん、学部4年程度の知識で理解できます。
0255132人目の素数さん垢版2014/10/23(木) 14:20:03.00
安藤氏がやっていることに含まれるのでは?
0256132人目の素数さん垢版2014/11/26(水) 07:51:56.64
> 255
改訂版をUPしました
http://www.math.s.chiba-u.ac.jp/~ando/ineq17.pdf
第2章が新しい部分です。
その別の応用は、また後日。
局所錐の理論も、もう少し進歩させられそうです。
0257132人目の素数さん垢版2014/11/26(水) 11:06:50.29
>>251
難しいな。これって、結構多くの不等式が一発で証明できるということなの?
ちょっと降りてきてアマチュア向けに補足を所望。できれば少し例も。
忙しいかな?
0259132人目の素数さん垢版2014/11/26(水) 14:06:16.63
        ∧∧
       ヽ(・ω・)/   ズコー
      \(.\ ノ
    、ハ,,、  ̄
     ̄
0261132人目の素数さん垢版2014/11/30(日) 21:19:05.89
> 257
代数曲面の場合でも、例えば、x^4+y^4=z^4 で定まる代数曲面を研究しようと思ったとき、
それを代数曲面の分類理論なして研究しても、あるところで行き詰まるが、
K3曲面という範疇の中で、どういう位置づけの曲面かを考えると、いろんなことが分かる。
同様に、1つの不等式を単体として証明するのとは別のアプローチとして、
不等式のある範疇の族を考え、その族の中での位置づけを考察すると新たな視点が開けるということ。
すると、個々の不等式の研究に増して、不等式の族の構造の研究も大切になってくる。
0262132人目の素数さん垢版2014/12/01(月) 09:12:04.34
>>261
説明ありがとう。でも不等式好きのアマチュアとして知りたかったのは、
このスレにも出てくるいろんな具体的な不等式たちを証明することが、
あなたの研究によってどう変わるかということだった。
分類理論もK3曲面も知らんのだ。そんな奴は相手にしてないということ
なんだろうが。
0264132人目の素数さん垢版2014/12/22(月) 09:04:18.22
A+B+C=Πのとき、
6cosA+3cosB+2cosC≦7
を示せ
0266132人目の素数さん垢版2014/12/23(火) 18:58:55.97
     *      *
  *     +  うそです
     n ∧_∧ n
 + (ヨ(* ´∀`)E)
      Y     Y    *
0267132人目の素数さん垢版2014/12/23(火) 23:15:39.09
不等式ヲタが一心不乱に不等式をいじっていたときに、ある貴婦人が訊ねた。
「そんな役にもたたないつまらないことをして何になるんですか?」
不等式ヲタはこう答えたという。
「生まれたばかりの赤ん坊が何の役にたつというのですか?」
0268 【豚】 【504円】 垢版2015/01/01(木) 00:17:51.41ID:819Tpouk
 
( ゚∀゚) プリッ
ノヽノ) =3'A`)ノ ヒャー
  くく へヘノ
0269132人目の素数さん垢版2015/01/11(日) 11:56:09.01ID:HYp86YiB
〆切過ぎたので投下

---------------------------------------------------
【大数1月号宿題】

n は3以上の自然数
x1,x2,...,xn は正の実数
S=x1+x2+...+xn
T=x1^2+x2^2+...+xn^2

とするとき

Σ[i=1,n]{xi/(S-xi)} ≦ Σ[i=1,n]{xi^2/(T-xi^2)}

が成り立つことを示せ
---------------------------------------------------
0270132人目の素数さん垢版2015/01/11(日) 15:49:08.19ID:6wGawbq2
>>114
[2](2)
2x-y, 2y-z, 2z-xがそれぞれ正正正か正負負の時のみ考えれば良い
(i)全部正の時
a=2x-y,b=2y-z,c=2z-xは全て正
7=3(aa+bb+cc)+4(ab+bc+ca)≧21GG
より
abc=G^3≦1/3√3

(ii)負が2つの時
a=2x-y,b=z-2y,c=x-2zは全て正
105=15(3(aa+bb+cc)-4a(b+c)+4bc)
=(6a-5b-5c)^2+9a^2+10(2bb+bc+2cc)
≧3(75abc)^(2/3)
これを整理して
abc≦7√35/15
等号は(a,b,c)=√35(1/3,1/5,1/5)の時成立

1/3√3<7√35/15より与式の最大値は7√35/15

>>269
(右辺)-(左辺)
=Σ[i<j]x_i*x_j*(T+x_i*x_j+(x_i+x_j)(S-x_i-x_j))(x_i-x_j)^2/((S-x_i)(S-x_j)(T-x_i^2)(T-x_j^2))
≧0
0272学術デジタルアーカイヴ院教授至高の狐独文武学者 珈琲豆SHO-GUN垢版2015/01/12(月) 17:49:50.45ID:Y1rwXzc/
現実以上。夢見饅。ツキ夜
0273132人目の素数さん垢版2015/02/06(金) 11:18:16.66ID:JF/VVn+X
n文字の基本対称式がすべて正なら、n文字は全て正は正しいなりか、キテレツ?
0274132人目の素数さん垢版2015/02/06(金) 18:33:53.81ID:D/pec2L1
f(x)=(x+a_1)…(x+a_n)=ΣS_{n-i}(a_1,…,a_n)x^i
x≧0 について f(x)>0 だから零点 -a_1,…,-a_n は全て負
0277葛厨垢版2015/02/10(火) 00:20:36.14ID:qNJQbzu4
632 :マロン名無しさん:2015/02/09(月) 23:47:58.84 ID:???
『2≧1なら普通に2=1も兼ねるだろw』

656 :マロン名無しさん:2015/02/10(火) 00:03:50.71 ID:???
『2≧1は2=1も兼ねる』

667 :マロン名無しさん:2015/02/10(火) 00:12:27.22 ID:???
『>と=の両方兼ねないと使えないのが≧ですw 』
0278葛厨垢版2015/02/10(火) 00:23:33.84ID:qNJQbzu4
679 :マロン名無しさん:2015/02/10(火) 00:21:24.07 ID:???
『「一方だけが」じゃなくて「一方もしくは両方が」だな』
0279132人目の素数さん垢版2015/02/10(火) 23:27:11.45ID:cnsbm1p4
正の整数nに対して、その正の約数の相加平均をf(n)とするとき
 √n ≦ f(n) ≦ (n+1)/2
0280132人目の素数さん垢版2015/02/11(水) 01:32:06.02ID:ufQLgWGB
>>279
a_i*b_i=nなるa_i≦b_iの組に分けられる
この時相加相乗より
2√n≦a_i+b_i
全ての正の約数について足し合わせると左側が示される

また約数は全てn以下の為
σ(n)≦Σ[1,n]k=n(n+1)/2
両辺をnで割ると右側が示される
0284132人目の素数さん垢版2015/02/13(金) 17:26:47.76ID:Eko0RVTk
過去スレで、極限が相加平均や総乗平均になる式があったと思うけど何だっけ?
0286132人目の素数さん垢版2015/03/15(日) 04:03:06.51ID:JATPX9sE
a、b、cを実定数、xを実数、f(x) = ax^2 + bx + c とする。
|x|≦1に対して、|f(x)|≦1 ならば |x^2・f(1/x)|≦2 を示せ。 ( ゚∀゚)バケラッタ!
0287132人目の素数さん垢版2015/03/21(土) 15:11:19.16ID:/GyhVn2u
実数x、yが|x|≦1、|y|≦1をみたすとき、

0 ≦ x^2 + y^2 - 2 x^2 y^2 + 2xy \sqrt{ (1-x^2)・(1-y^2) } ≦ 1

ハアハアできそうな解法ありますか?
0289132人目の素数さん垢版2015/03/21(土) 21:59:51.63ID:fODsSTst
同じようなことですが

x^2+y^2-2x^2 y^2+2xy√{(1-xx)(1-yy)}
={x√(1-yy)+y√(1-xx)}^2
=1-{xy-√((1-xx)(1-yy))}^2
Think different? by 2ch.net/bbspink.com
0291132人目の素数さん垢版2015/03/26(木) 12:00:18.74ID:xrzRkczx
p>q≧e に対して、log(log p) - log(log q) < (p-q)/e

式の形を見た瞬間にどうやって作ったかが分かってしまうので一捻りしたかったが、思いつかなかった。
誰か一捻りしてハアハアできそうな不等式を作ってくりりん。
0292132人目の素数さん垢版2015/03/30(月) 13:01:31.71ID:jdCsFSZj
正の数 a、b、c の相加平均A、相乗平均G、調和平均Hに対して、次式を証明せよ。

    (A^3)/(G^3) + (G^3)/(H^3) + 1 ≦ (3/4)・(1 + A/H)^2

       ___ 
彡     /  ≧ \    彡 ビュゥ……
  彡   |:::  \ ./ |  彡
      |:::: (● (●|    
      ヽ::::......ワ...ノ    
        人つゝ 人,,         
      Yノ人 ノ ノノゞ⌒〜ゞ    
    .  ノ /ミ|\、    ノノ ( 彡
     `⌒  .U~U`ヾ    丿
             ⌒〜⌒
0293132人目の素数さん垢版2015/03/31(火) 12:36:03.23ID:lmwv+TH5
power mean について、自分は今まで √{ (a^2 + b^2)/2 } を2乗平均って言ってたんだけど、
wiki をみたら二乗平均平方根って書いてあるんよな。

手元にある本で調べてみたら、

[1] P.1 では 『平均』 としか書かれてないし、
[2] P.110 では 『r次平均』 と書かれているから、2次平均。
[5] P.47 では 『t次の累乗平均』 で定義されているから、2次の累乗平均。

ネットで適当に検索してみたら、√なしの (a^2 + b^2)/2 を2乗平均とよんでたり、
別のところでは √{ (a^2 + b^2)/2 } を2乗平均とよんでたりするけど、


   √{ (a^2 + b^2)/2 } を2乗平均と呼んじゃダメ?
0296132人目の素数さん垢版2015/04/14(火) 08:21:42.52ID:iZ1uB03M
かしらかしら ごぞんじかしらー?

 \_●    ∠_●_
   /\|      \ し
   ▲           ▼
    ||           |
__|_|________|___
   \\         \
     \\         \
0300132人目の素数さん垢版2015/04/28(火) 11:15:01.53ID:H6i5w/Zl
>>299
グラフを描いたらたしかに
≦2となりますね。
グラフでは「示す」=「証明」という条件に
当てはまらないのでしょうね。
0311132人目の素数さん垢版2015/05/08(金) 06:21:13.84ID:UVsccB/V
a, b, c, d, p, q >0 のとき、a^p・b^q + c^p・d^q ≦1

('A` ) プウ
ノヽノ) =3'A`)ノ ヒャー
  くく へヘノ
0312132人目の素数さん垢版2015/05/09(土) 05:36:13.58ID:2Ji3jztR
-1 < x, y, z < 1 に対して、1/{(1+x^2)(1+y^2)(1+z^2)} + 1/{(1+x)^2・(1+y)^2・(1+z)^2} ≧ 2(1-x)(1-y)(1-z)
0313132人目の素数さん垢版2015/05/16(土) 09:24:02.99ID:Vp+m9OwM
面白スレより
a>0、b>0、c>0、d>0、abcd=1のとき、1/a + 1/b + 1/c + 1/d + 9/(a+b+c+d) ≧ 25/4 を証明せよ
0314132人目の素数さん垢版2015/05/18(月) 19:48:30.97ID:BC+ZL+kX
0≦x≦1 において f(x)≧0、f は連続
∫[0,1] {f(x)}^3 dx ≧ 4・(∫[0,1] x^2・f(x) dx)・(∫[0,1] x・{f(x)}^2 dx)
0315132人目の素数さん垢版2015/05/18(月) 19:51:18.42ID:BC+ZL+kX
x, y, z >0、x+y+z=3 のとき、(x^4+x^2+1)/(x^2+x+1) + (y^4+y^2+1)/(y^2+y+1) + (z^4+z^2+1)/(z^2+z+1) ≧ 3xyz
0316132人目の素数さん垢版2015/06/12(金) 00:35:57.46ID:J8A12ugu
For a, b, c > 0
(1+a+ab)(1+b+bc)(1+c+ca) ≦ (1+a+a^2)(1+b+b^2)(1+c+c^2) ≦ (1+a+b^2)(1+b+c^2)(1+c+a^2)
0318132人目の素数さん垢版2015/06/23(火) 23:00:44.46ID:dLrMTxVf
>>317
abc=t^3かつt<2 として
a=tx/y, b=ty/z, c=tz/y (x,y,z>0)とおける
この時

1/(a+1)+1/(b+1)+1/(c+1)
=y/(tx+y)+z/(ty+z)+x/(tz+x)
≧(x+y+z)^2/(xx+yy+zz+t(xy+yz+zx))
>(x+y+z)^2/(xx+yy+zz+2(xy+yz+zx)
=1
0320132人目の素数さん垢版2015/07/19(日) 22:35:11.49ID:5fmvhM43
【面白スレ223より】
> 任意の2以上の整数nに対して,
> 不等式 tan(π/(2n))≦2/((n-1)*n^(1/(n-1)))
> が成り立つことを示せ.
0322132人目の素数さん垢版2015/08/16(日) 19:42:02.94ID:7Zcz05hp
大学入試問題なんだが
(1)
x[1]+x[2]=1のときx[1]^2+x[2]^2≧1/2を示せ
(2)
x[1]+x[2]+x[3]=1のときx[1]^2+x[2]^2+x[3]^2≧1/3を示せ
(3)
x[1]+x[2]+…+x[n]=1のときx[1]^2+x[2]^2+…+x[n]^2≧1/nを示せ

(3)はコーシーシュワルツで一発なんだが(1)(2)の誘導使って解く方法ないかな
0323132人目の素数さん垢版2015/08/16(日) 22:43:29.67ID:+QPM3wJI
x[1]+x[2]+…+x[k+1]=1 を x[1]/(1-x[k+1])+x[2]/(1-x[k+1])+…+x[k]/(1-x[k+1])=1 として帰納法、でいいんじゃないかな
誘導の意図が帰納法を使えということだとしてだけど
0325132人目の素数さん垢版2015/11/06(金) 20:29:53.93ID:OdHYjEZu
一応、真面目な代数的不等式論の話です。
Hilbertの第17問題を実閉体を使ってArtinが解決したとき、
Artinは実代数多様体への一般化を考えていた。
でも、実代数多様体はカテゴリー(圏)を形成しないので、うまくいかない。
しかし、半代数的集合を一般化した概念をうまく作ると、
それと正則写像によって圏が形成できて、
その上の線形系の中で代数的不等式錐を考えると、
スキーム論のような、綺麗な理論体系ができるみたい。
長年眠っていた順序体の理論も、実スペクトル(Spec_r)理論でよみがえる。
永田先生の可換体論の順序体(実閉体)の章も、久々に読み返してみたら、
いいことが書いてあった。
0327132人目の素数さん垢版2015/11/18(水) 18:39:42.97ID:Qfx2C9OA
>>325
実閉体の話はTarskiあたりを参考に書いたのでしょうかねぇ。
教科書に出典が書いてなくて。
0328132人目の素数さん垢版2015/11/24(火) 16:46:32.16ID:C6eyGVmW
> 327
そのうち、プレプリを書いて公開します。
一番参考になりそうな文献は
J. Bochnak, M.Coste, M-F.Roy, Real Algebraic Geometry
もちろん、そこには書いてないけど。
0329132人目の素数さん垢版2015/11/24(火) 21:24:31.12ID:Bfc3KxAd
ちょっとわかりにくいことを書いたので、初心者向けに例題で説明します。
[例題] 6次斉次巡回多項式
f = (x^6+y^6+z^6) + a_1(x^5y + y^5z + z^5x) + ... + a_9 x^2y^2z^2
を考える。
任意の実数 x,y,z に対して f(x,y,z)≧0 が成り立つための必要十分条件を
a_1,...,a_9 の式で表せ。
[考え方]
Hilbertが証明したように、そういう f は多項式の2乗の和に表せるとは限らない。
SOS methodより、そういう f 全体のなす半代数的凸錐の実代数幾何的特徴を
攻めたほうが早い。
この場合、その凸錐のザリスキー閉包は既約な代数多様体になる。
ただし、4次の場合ですら複雑だったから、6次はもっと複雑だと予想できる。
そこで、まず、代数多様体としての特徴を考察してみましょう、
とかいうのが応用例。
0331132人目の素数さん垢版2015/11/30(月) 01:52:51.35ID:5m9vQyAq
A : 算術平均
G : 幾何平均
H : 調和平均
(A/G)^3 + (G/H)^3 + 1 ≦ (3/4)*(1 + A/H)^2

(1) この不等式の証明はどうやるんでしょうか?
(2) この不等式には名前がついていますか?
0332132人目の素数さん垢版2015/12/02(水) 09:06:16.61ID:eDabyYFA
重みつき相加平均・相乗平均の関係の積分形って、どっかに載ってないかな?
0335132人目の素数さん垢版2015/12/05(土) 09:00:01.06ID:0Ltl1TAM
a^(1/3)b^(1/3)c^(1/3)
=exp((1/3)log(a)+(1/3)log(b)+(1/3)log(c)).

exp(∫log(f(x))dx/∫dx)≦∫f(x)dx/∫dx.
0340132人目の素数さん垢版2016/01/08(金) 19:26:23.02ID:QUdG1pIk
すみまんせん

 実数xに対して、x^16 + x + 1 > 0 を示せ

というのはどうすればいいでしょうか。
0341132人目の素数さん垢版2016/01/08(金) 21:32:00.51ID:tLrcJQrP
|x|≧1の時は、x^16≧|x|≧0 なので自明
|x|<1の時は、x^16+x+1≧x^16-|x|+1>1-|x|≧0
0343132人目の素数さん垢版2016/01/08(金) 21:58:52.85ID:5Mf1q4fA
任意の実数 x に対して、次式を示せ。
x^16 + x + 1 > 0
x^16 - x + 1 > 0

4平方和になるよな。
0347132人目の素数さん垢版2016/01/08(金) 23:30:11.43ID:QUdG1pIk
平方和だなんて
なんでこんな変態的な解法がそんなに簡単に思い浮かぶのですかあ
0350132人目の素数さん垢版2016/01/09(土) 12:04:31.42ID:/ERKXySg
マジレスすると、代数的不等式の常套手段だからとりあえず平方完成を試みただけだと思う
0351132人目の素数さん垢版2016/01/09(土) 14:02:20.94ID:Mkv80jBo
>>350
きみは実に面白くない人だな。
普段から言わなくてもいい一言で場を凍りつかせていないか?
0352132人目の素数さん垢版2016/01/09(土) 14:20:20.82ID:/ERKXySg
いや、ちょっと気持ち悪いノリが続いたからさ
単発で留めておけば気にならなかったのに
0353132人目の素数さん垢版2016/01/09(土) 14:21:37.85ID:ytPTCyVz
ここって昔からキモさ全開じゃん
0355132人目の素数さん垢版2016/01/15(金) 18:47:00.28ID:A1LDyDSu
俺も>>344は変態的ではないと思うな。
>>344レベルで変態的と思うようでは、不等式ヲタとしては勉強不足だと折れは思うわ。
0359132人目の素数さん垢版2016/02/18(木) 20:11:00.29ID:1twGteVq
a、b、c を正の実数とする。
a^3・b^6 + b^3・c^6 + c^3・a^6 + 3・a^3・b^3・c^3 ≧ abc・(a^3・b^3 + b^3・c^3 + c^3・a^3) + a^2・b^2・c^2・(a^3 + b^3 + c^3)
0360132人目の素数さん垢版2016/03/14(月) 20:18:24.48ID:rpo94kst
大学への数学の3月号宿題ですがお知恵を貸して下さい
特に高校範囲の回答が未だ出てません
高校範囲外の回答も大歓迎です!

---------------------------------------------------------------------------------
(z1,z2,...,zn)∈C^n,
(z1,z2,...,zn)≠(0,0,...,0),
F=(|z1−z2|^2+...+|z(n-1)−zn|^2+|zn−z1|^2)/(|z1|^2+|z2|^2+...+|zn|^2)

とする

(1) m=1,...,n-1 の各場合のおいて

zk=cos(2kmπ/n) +i sin(2kmπ/n) (1≦k≦n)

のとき F の値を求めよ

(2)さらに z1+z2+...+zn=0 をみたすとき F の最大値と最小値を求めよ
--------------------------------------------------------------------------------
0361132人目の素数さん垢版2016/03/14(月) 20:22:55.03ID:rpo94kst
参考スレッド

大学受験_sc
★【月刊大学への数学】学力コンテスト・宿題23
0363132人目の素数さん垢版2016/03/17(木) 12:40:29.21ID:rjgplTGC
>>340 実数xに対して、x^16 + x + 1 > 0 を示せ

任意の実数xと任意の自然数nに対して、x^2n + x + 1 > 0  を示せ

ここまで一般化できるよね それだけ
0365132人目の素数さん垢版2016/03/21(月) 23:28:07.28ID:1HYPEF6D
a,b,c,d≧0
a≦1, a+b≦5, a+b+c≦14, a+b+c+d≦30
のとき
 sqrt(a)+sqrt(b)+sqrt(c)+sqrt(d)≦10
を示せ。
0366132人目の素数さん垢版2016/03/23(水) 05:14:47.27ID:cBAcmLOd
a + 4*(b/4) + 9*(c/9) + 16*(d/16) ≦ 30 に、
f(x)=√xとして、凸不等式 (1/n)Σ[f(x_i)] ≦ f((Σx_i)/n) を用いると
(1/30)*{√a + 4*√(b/4) + 9*√(c/9) + 16*√(d/16)} ≦ √[{a+4*(b/4)+9*(c/9)+16*(d/16)}/30]≦1
整理すると 
a + 2√b + 3√c + 4√d ≦ 30  ・・・(1)
を得る。同様に、
a + 2√b + 3√c ≦ 14  ・・・(2)
a + 2√b + ≦ 5  ・・・(3)
a ≦ 1  ・・・(4)
{(1)*3 + (2) + (3)*2 + (4)*6}/12 を計算すると目的の式を得る
0368132人目の素数さん垢版2016/03/23(水) 14:46:27.16ID:pTWUy74g
狂気すら感じる思考回路だわw
0370132人目の素数さん垢版2016/03/24(木) 07:45:57.15ID:rxJME1YD
x,y,a,bが非負のとき sqrt((x+a)(y+b)) ≧ sqrt(xy) + sqrt(ab)

両辺を平方して同値変形してもすぐに示せるのですが
コーシーとかイエンセンとかでもっと華麗に示す方法はないでしょうか。
0371132人目の素数さん垢版2016/03/24(木) 11:43:02.90ID:QnvnOXRV
BF恒等式
(A^2+B^2)(X^2+Y^2)=(AX+BY)^2+(AY-BX)^2
BとXを入れ替えて
(A^2+X^2)(B^2+Y^2)=(AB+XY)^2+(AY-BX)^2
A^2→a (a≧0) 等と置き換えると
(a+x)(b+y)=(√(ab)+√(xy))^2+(√(ay)-√(bx))^2
(a+x)(b+y)≧(√(ab)+√(xy))^2
0372132人目の素数さん垢版2016/03/24(木) 16:18:34.64ID:qfgEsb+P
ベクトル (sqrt(x), sqrt(a)) と (sqrt(y), sqrt(b)) についてコーシーの不等式そのまんまやん
0373132人目の素数さん垢版2016/03/24(木) 22:43:49.57ID:rxJME1YD
そういわれてみるとそうでした
0377132人目の素数さん垢版2016/04/04(月) 17:33:43.64ID:GEiBx8Vr
x≧0において √(x+1)+√(3x+9) ≦ √(x+4)+√(3x+4)
0378132人目の素数さん垢版2016/04/05(火) 23:04:02.63ID:nBcOqMq6
正の数a,b,c及び0≦x≦y≦zが
 a≦x かつ a+b≦x+y かつ a+b+c≦x+y+z
を満たすとき
 √a +√b +√c ≦ √x +√y +√z
0379132人目の素数さん垢版2016/04/06(水) 07:32:06.00ID:TK/STlbk
検索しても2種類の数列の場合しか出てきませんが、並べ替え不等式は、一般化できますか?
0382132人目の素数さん垢版2016/04/07(木) 11:57:36.82ID:MmBSjqX/
>>381
384ページに書いてあるでしょ
君の言うとおり2種類の数列以外の場合に一般化できるってこと
各数列を降順(昇順)ソートして対応する要素をかけて足した方が大きい(小さい)
=の可能性もあるけど
0383132人目の素数さん垢版2016/04/08(金) 06:40:00.72ID:fwk06iXt
>>377
√(x+1)+√(3x+9) ≦ √(x+4)+√(3x+4) において x→12x^2と置き換えると
√(4+12x^2)+√(4+36x^2) ≧ √(1+12x^2) + √(9+36x^2) となる。この式を証明することにする。
なお、x=0の時成立するのは明白なので、以後、x>0とする。

O(0,0) , A(2,(2√3)x) , B(1,(2√3)x) , P(4,(6+2√3)x) とすると この式は
OA + AP ≧ OB + BP 

三角形OAPの面積は、△OAP=(1/2)*|2*(6+2√3)-4*(2√3)|x=2√3(√3-1)x
三角形OBPの面積は、△OBP=(1/2)*|1*(6+2√3)-4*(2√3)|x=3(√3-1)x

OP=√(16+(48+24√3)x^2) に注意して |OA-AP|/OP および |OB-BP|/OP を評価すると
0 ≦|OA-AP|/OP < 2-√3 、2-√3 <  |OB-BP|/OP ≦ 1/2 

ところで、辺長が2a,2b,2cの三角形の面積をSとすると、S^2=(a+b+c)(a+b-c)(a-b+c)(-a+b+c) なので
(a+b)^2 = c^2 +S^2*r/c^2  ,ただしr=1/(1-((a-b)/c)^2) という関係がある。

三角形OAPにおいて、OA=a、AP=b、OP=c として、上の関係式を使うと
(OA+AP)^2 = OP^2 + (△OAP/OP)^2 * r_a
r_a=1/(1-(|OA-AP|/OP)^2) ≧1 なので
(OA+AP)^2 ≧ OP^2 + (2√3(√3-1)x)^2/OP^2 *1 = OP^2 + 12(4-2√3)x^2/OP^2

同様に三角形OBPにおいて、OB=a、BP=b、OP=c とすると
(OB+BP)^2 = OP^2 + (△OBP/OP)^2 * r_b
r_b=1/(1-(|OB-BP|/OP)^2) ≦4/3 なので (∵ |OB-BP|/OP ≦ 1/2)
(OB+BP)^2 ≦ OP^2 + (3(√3-1)x)^2/OP^2 * (4/3)= OP^2 + 12(4-2√3)x^2/OP^2

最右辺が一致しているので (OA+AP)^2 ≧ OP^2 + 12(4-2√3)x^2/OP^2 ≧(OB+BP)^2  が示され、目的の式が得られる
0387132人目の素数さん垢版2016/04/15(金) 14:22:58.26ID:NGXVqzKa
実数 a, b, c に対して、2(a^4 + b^4 + c^4) + (71+17√17)/2 ≧ 4abc + a^2b^2 + c^2a^2 + 3b^2c^2
0388prime132垢版2016/04/20(水) 21:59:27.66ID:cUe+Ipvo
>>267
ファラデー(電磁誘導の研究で)
0389132人目の素数さん垢版2016/04/21(木) 10:17:03.84ID:/+LZXjJ0
>>267
貴婦人は不等式ヲタをしげしげと眺め
「あなた、生まれたばかりの赤ん坊の時代から随分年をとったけど
いまだに誰の役にもたってないじゃん」
といったので
「うっせーババア」
と言うしかなかったという。
0390prime132垢版2016/04/21(木) 16:26:16.73ID:iheZo7AQ
>>340 >>343
相加-相乗平均
x^16−16(a^15)x+15(a^16)=(x^8-a^8)^2+2(a^8)(x^4-a^4)^2+4(a^12)(xx-aa)^2+8(a^14)(x-a)^2,
で、16(a^15)=−1とおくと
a=−0.8312379
x^16+x+1≧1+(15/16)a=0.220714
0391132人目の素数さん垢版2016/04/21(木) 16:54:58.77ID:QoWZFuzY
>>389

 _|_ \.       `ヽ、`ヽ、`ヽ、              ____\\
  _|_.        `ヽ、`   ヽ、                  /
/ |  ヽ         `ヽ、`ヽ、 ∧∧              (
\ノ   ノ           `ヽ、 (  ).               \
                 `ヽ、/ 三つ
              ハ,,ハ  /// `ヽ、
             (゚ω゚)/`ヽ、  | | ←貴婦人
           ,..‐''"ii "  ./ | `ヽ、`ヽ`ヽ、
 lヽ│/ /    | ┝ ||-┨/  | `ヽ、`ヽ   `ヽ、
 !ー┼‐ |‐┬   / ∧ ||   |  `ヽ、∪ヽ、`ヽ、`ヽ、`ヽ、    __|_
 |./│ヽ l  |   ヽ,''i|口=彡'i   | ||`ヽ、   `ヽ、`ヽ、      _|
.└── .l  |    ヽ' ||   'i  `ヽ、|`ヽ、`ヽ、`ヽ、       (_|
            |ノ||   'i  |`ヽ、   `ヽ、           ノ
            | ||  `ヽ`ヽ|`ヽ|`ヽ、
            'i ||   `ヽ|, ∪∪
             'i ̄|i``'''‐`ヽ、
 l   |         'i  | 'i`ヽ、`ヽ、`ヽ、`ヽ、`ヽ、         ┏┓┏┓
 レ  |          |_._| 'i_ 'i_`ヽ、`ヽ、   `ヽ、`ヽ、      ┃┃┃┃
      l         /__,.|  | ̄\ `ヽ、`ヽ、`.ヽ、        ┗┛┗┛
   _ノ.          ̄     ̄ ̄                   ┏┓┏┓
0392prime132垢版2016/04/21(木) 18:26:19.65ID:iheZo7AQ
>>387
等号成立は
a=(3+√17)/2,
b=c=±√{(19+5√17)/2},
abc=(71+17√17)/2,
のとき。
0393prime132垢版2016/04/21(木) 19:05:20.42ID:iheZo7AQ
>>365
コーシーで
(√a+√b+√c+√d)^2≦(1+2+3+4){a+(b/2)+(c/3)+(d/4)},
=10・{(1-1/2)a+(1/2-1/3)(a+b)+(1/3-1/4)(a+b+c)+(1/4)(a+b+c+d)}
≦100.
ぐらいしか思いつかぬ…
0394prime132垢版2016/04/21(木) 19:35:06.66ID:iheZo7AQ
>>363
相加-相乗平均
x^(2n)−2n・a^(2n-1)x+(2n-1)a^(2n)={x^(2n-2)+2a・x^(2n-3)+…+(2n-2)a^(2n-3)x+(2n-1)a^(2n-2)}(x-a)^2,
で、2n・a^(2n-1)=−1とおく。
a=−(1/2n)^[1/(2n-1)],
x^(2n)+x+1≧1−{(2n-1)/2n}|a|.

だよね。ただそれだけ
0396prime132垢版2016/04/21(木) 23:10:05.41ID:iheZo7AQ
>>377 >>385
では単に2乗して考える。
{(右辺)^2−(左辺)^2}/2=√{(x+4)(3x+4)}−√{(x+1)(3x+9)}−1
=(4x+7)/[√{(x+4)(3x+4)}+√{(x+1)(3x+9)}] −1
≧(4x+7)/[(2x+4)+(2x+3)] −1
=0,
0398132人目の素数さん垢版2016/04/22(金) 04:24:48.44ID:in8T91Zq
>>387
f = 2(a^4 + b^4 + c^4) - ( 4abc + a^2b^2 + c^2a^2 + 3b^2c^2 )
= (a^2+2a-bc)^2 + 2(b^2-c^2)^2 - a^2(b-c)^2 + a^4 - 4a^3 - 4a^2
fはbとcの入れ替えに対し不変なので、b=c上で極値を持つ
従って、g(a) = a^4 - 4a^3 - 4a^2 の最小値が、fの最小値になる
(最小値を取るときのaの値を用いて、b=c=±√(a^2+2a)の時)
g'(a) = 4a^3-12a^2-8a = 4a(a^2-3a-2) 等から、最小値を取るときのaは a=(3+√17)/2
g(a) = a^4 - 4a^3 - 4a^2 = (a^2-3a-2)(a^2-a-5) -17a-10
g((3+√17)/2) = -17 * (3+√17)/2 -10 = (-71-17√17)/2
0399132人目の素数さん垢版2016/04/22(金) 10:07:06.42ID:KRm4h22p
絶対値が1未満の任意の複素数α,β,γに対して、以下が成立するための正の実数λの最小値を求めよ。
・α+β+γ=0ならば、|αβ+βγ+γα|^2+|αβγ|^3<λが成立する
0400prime132垢版2016/04/22(金) 18:28:52.31ID:RdqVAiN1
>>398

f=g(a)+{(aa+2a)−(bb+cc)/2}^2+2a(b-c)^2+(7/4)(bb-cc)^2≧g(a).

等号成立は b=c=±√(aa+2a) のとき。
0401132人目の素数さん垢版2016/04/23(土) 00:58:07.85ID:lKzrVXQh
>>400
f_(b=1,c=0)= 2 a^4 -a^2 +2 = f_10(a) とすると、
f_10(-2)=2*16-4+2=30
g(-2)=16-4*(-8)-4*4=32
f≧g が常に成り立つわけではない。
0402132人目の素数さん垢版2016/04/23(土) 07:32:00.59ID:iIo4nVsk
rは実数、a>1、f(r) = [ (a^(r+1)-1) / { (r+1)(a-1) } ]^(1/r) が単調増加することを証明するにはどうすればいいですか?
0403132人目の素数さん垢版2016/04/23(土) 07:46:12.96ID:iIo4nVsk
>>400
> f=g(a)+{(aa+2a)−(bb+cc)/2}^2+2a(b-c)^2+(7/4)(bb-cc)^2≧g(a).

a<0 のとき 2a(b-c)^2 < 0 だから、上の不等式は常には成り立たんよな。
0404132人目の素数さん垢版2016/04/23(土) 09:21:29.81ID:lKzrVXQh
>>403
a<0の時、2a(b-c)^2 ≦0となるのは、一目瞭然。
しかし、他の項との兼ね合いで、全体で結果的には不等式が成立する という可能性は残されていて、
それを否定するために、具体例を挙げた。
0406prime132垢版2016/04/23(土) 18:09:51.16ID:5r1UlAr6
>>401 >>403-405

2|a|(bb+cc)−4abc=(|a|+a)(b-c)^2+(|a|-a)(b+c)^2≧0,

f=g(|a|)+{(aa+2|a|)−(bb+cc)/2}^2+2|a|(bb+cc)−4abc+(7/4)(bb-cc)^2≧g(|a|)
0407prime132垢版2016/04/23(土) 18:20:18.97ID:5r1UlAr6
>>401 >>403-405

2|a|(bb+cc)−4abc≧4(|abc|-abc)≧0,

g(a)+(71+17√17)/2={[a+(-1+√17)/2]^2+√17−1}{a−(3+√17)/2}^2≧0,

等号成立は a=(3+√17)/2.
0408132人目の素数さん垢版2016/04/23(土) 19:03:41.02ID:lKzrVXQh
なるほど、aの符号に注目するのなら、
f = 2(a^4 + b^4 + c^4) - ( 4abc + a^2b^2 + c^2a^2 + 3b^2c^2 )
≧ 2(a^4 + b^4 + c^4) - ( 4|abc| + a^2b^2 + c^2a^2 + 3b^2c^2 )
= 2(|a|^4 + |b|^4 + |c|^4) - ( 4|a|*|b|*|c| + |a|^2*|b|^2 + |c|^2*|a|^2 + 3|b|^2*|c|^2 )
となるので、fの最小値を調べるときは、a,b,c が非負の範囲だけを調べれば十分
として、>>400へ至るのもありですね
0409prime132垢版2016/04/23(土) 19:45:23.68ID:5r1UlAr6
>>359
abb=x、bcc=y、caa=zとおくと、
(左辺)−(右辺)=(x^3+y^3+z^3)+3xyz−(zxx+xyy+yzz)−(zzx+xxy+yyz)
=x(x-y)(x-z)+y(y-z)(y-x)+z(z-x)(z-y)
=F_1(x y z)
≧0.  (Schur)
0410prime132垢版2016/04/24(日) 18:45:52.58ID:qRA+Gwsm
>>291
f(x)=log(x)/xはx>eで単調減少。
log(p)/p<log(q)/q≦1/e,
∴log{log(p)}−log{log(q)}<log(p)-log(q)<(p-q)log(q)/q≦(p-q)/e.
0411prime132垢版2016/04/24(日) 19:50:25.33ID:qRA+Gwsm
>>297
 a^4+b^4+c^4+d^4−4|abcd|={(aa-bb)^2+(aa-cc)^2+(aa-dd)^2+(bb-cc)^2+(bb-dd)^2+(cc-dd)^2+2(|ab|-|cd|)^2+2(|ac|-|bd|)^2+2(|ad|-|bc|)^2}/3≧0,

>>299 >>302
 |sin(x)^5|≦sin(x)^4≦sin(x)^2、|cos(x)^5|≦cos(x)^2.

>>305
{1+sin(x)}{1+cos(x)}=1+{sin(x)+cos(x)}+sin(x)cos(x)
=1+(√2)C+(CC−1/2)
=(C+1/√2)^2,
ここに C=cos(x-π/4).

>>307
 (1+a)(1+b)=(1+ab)+(a+b)
 {(1+a)(1+b)}^2≧4(1+ab)(a+b)
 {(1+a)(1+b)}^4≧16(4ab)(a+b)^2

>>308
(左辺)=a/a^(1-p)+b/b^(1-p)+c/c^(1-p)
≧a/(a+b+c)^(1-p)+b/(a+b+c)^(1-p)+c/(a+b+c)^(1-p)
=(a+b+c)/(a+b+c)^(1-p)
=(右辺).
0412prime132垢版2016/04/24(日) 20:06:11.29ID:qRA+Gwsm
>>309
左辺を2乗して >>287-290 を参照.

>>312
1/{(1+xx)(1+yy)(1+zz)}^2+1/{(1+x)(1+y)(1+z)}^2
≧{(1-xx)(1-yy)(1-zz)}^2+1/{(1+x)(1+y)(1+z)}^2
≧2(1-x)(1-y)(1-z)    (相加-相乗平均)

>>314
0≦{f(x)・y+x・f(y)}{f(x)・y−x・f(y)}^2
=f(x)^3・y^3−xf(x)^2・yyf(y)−xxf(x)・yf(y)^2+x^3・f(y)^3,
これを 0<x<1、0<y<1 で積分する。

>>315
x+y+z=sとして
左辺=(xx-x+1)+(yy-y+1)+(zz-z+1)
={ss+(x-y)^2+(y-z)^2+(z-x)^2}/3−s+3
≧ss/3−s+3
=sss/9+(1+ss/9)(3-s)
≧sss/9
≧3xyz.

>>316
中辺−左辺=(ss-3t)+(aab+bbc+cca-3abc)+(tt-3su)≧0
右辺−中辺=(aaa+bbb+ccc-abb-bcc-caa)+(aaab+bbbc+ccca-abcs)+(aabbb+bbccc+ccaaa-abct)≧0

>>317
1/(a+1)=A、1/(b+1)=B、1/(c+1)=Cとおくと A+B+C=1,
abc=(1-A)/A・(1-B)/B・(1-C)/C
=(B+C)/A・(C+A)/B・(A+B)/C≧8
 佐藤(訳)、例1.7.1
0413prime132垢版2016/04/24(日) 21:16:30.41ID:qRA+Gwsm
>>297
a^4+b^4+c^4+d^4−4|abcd|=(aa-bb)^2+(cc-dd)^2+2(|ab|-|cd|)^2≧0
で十分だ....
0414prime132垢版2016/04/24(日) 23:41:46.77ID:qRA+Gwsm
>>313
これはムズイが、未定乗数を使わないでやるなら、

√(ab)=g≦c+d として{a,b,c,d}と{g,g,c,d}を比べる。
1/a+1/b+9/(a+b+c+d)−2/g−9/(2g+c+d)={1/ab−9/[(a+b+c+d)(2g+c+d)]}(√a−√b)^2≧{1/gg−9/(2g+c+d)^2}(√a−√b)^2≧0。{←(2g+c+d)≧3g}
∴{g,g,c,d}の方が左辺は小さい。
∴左辺が最小のときは、小さい方の3つは等しいはず。
{a、a、a、1/a^3}のとき、
{左辺−25/4}a(3a^4+1)=3a^8−(75/4)a^5+19a^4−(25/4)a+3
=g(a)(1-a)^2≧0,
ここに、g(a)=3a^6+6a^5+9a^4−(27/4)a^3−(7/2)a^2−(1/4)a+3,
極小値:g(0.567317)=1.88411147556
とでもするか、う〜む。

[35] CGMO-2011 問4
0416132人目の素数さん垢版2016/04/25(月) 02:17:12.36ID:1zw10KMQ
>>313
1/a + 1/b + 1/c + 1/d + 9/(a+b+c+d)
= 4*1/(4a) + 4*1/(4b) + 4*1/(4c) + 4*1/(4d) + 9*1/(a+b+c+d)
≧25*1/{{(4*4a+4*4b+4*4c+4*4d+9*(a+b+c+d)}/25}=25/(a+b+c+d)
≧25/(4*[4]√(abcd))=25/4
第一の不等式は、1/xの凸不等式、第二の不等式は相加相乗平均の関係
0418132人目の素数さん垢版2016/04/25(月) 18:02:36.24ID:V9CJZU3J
対数平均を一般化したものを2つ。
(1) r(b^{r+1} - a^{r+1}) / {(r+1)(b^r - a^r)}、r≠0,-1 が単調増加することを示せ。
(2) [(b^{r+1} - a^{r+1}) / {(r+1)(b-a)}]^{1/r}、r≠0,-1 が単調増加することを示せ。
0419prime132垢版2016/04/25(月) 19:38:27.89ID:9bCodAUH
>>286
|f(0)|=|c|≦1,

|f(-1)|=|a-b+c|≦1より
 -1≦a-b+c≦1,
|f(1)|=|a+b+c|≦1より
 -1≦a+b+c≦1

∴ |a+c|+|b|≦1,

また、|a|≦|a+c|+|c|≦2,

|xxf(1/x)|=|a+bx+cxx|=…
0420prime132垢版2016/04/25(月) 19:45:19.89ID:9bCodAUH
0<p≦x_i≦qとし、{x_1、x_2、……、x_n}の相加平均をA、調和平均をHとすると

 1≦A/H≦1+(1/4pq)(q-p)^2,
0421prime132垢版2016/04/25(月) 19:55:19.98ID:9bCodAUH
【Johnstonの不等式】
0<p≦x_i≦qとし、{x_1、x_2、……、x_n}の相加平均をA、相乗平均をGとすると

A≧G≧{p^(q-A)・q^(A-p)}^[1/(q-p)],

(略証)
y=log(x)は上に凸だから、
 log(x_i)≧{(q-x_i)log(p)+(x_i-p)log(q)}/(q-p)
i=1〜nでたしてnで割ると、
 log((1/n)Σx)≦(1/n)log(Πx)≧{(q-A)log(p)+(A-p)log(q)}/(q-p),
0422prime132垢版2016/04/26(火) 23:08:45.36ID:XSjTtKsA
>>414
g(a)=3(x-0.567317455622392)^2・{(x+1.18748893643)^2+2.74219081934}{(x+0.37982851920)^2+0.134057954257}+1.8841114755587
≧1.8841114755587
0423prime132垢版2016/04/26(火) 23:53:39.43ID:XSjTtKsA
>>414
a≧0に限定すれば
g(a)=3a^6+6a^5+(27/16)(3a^4−4a^3+1)+(63/16)(aa−1/2)^2+(3/16)a^2+(1/4)(a−1/2)^2+17/64
>17/64,
0424prime132垢版2016/04/27(水) 22:15:30.43ID:iO9YGMzp
>>414
a〜1/√3 の辺りに極小があるので…

g(a)=3(a-1/√3)^2・{aa(a+1+1/√3)^2+(2/3)(4+√3)(a−a1)^2+c}+[(7√3-11)/3]a+11(1+√3)/18
≧3c(a-1/√3)^2+[(7√3-11)/3]a+11(1+√3)/18
=3c(a−a2)^2+d
≧d.
 
a1=−(361√3-300)/624=−0.521266573
a2=(9883-4788√3)/(3108+731√3)=0.363487378
c=(3108+731√3)/4992=0.8762277925
d=(40898191√3−63973937)/3718422=1.845892626
0425prime132垢版2016/04/27(水) 23:35:27.38ID:iO9YGMzp
>>190
相加-相乗平均で(n-1)a^n+b^n≧na^(n-1)・b,
∴(a^n)/b≧{n・a^(n-1)−b^(n-1)}/(n-1),

>>210-213
Lagrangeさんもびっくり!ですね。


>>279
dがnの約数ならn/dも約数で(d、n/d)のペアになっている。(√n以外は)
f(n)=√n ぢゃね?

>>310
右側は >>156-157
0426prime132垢版2016/04/28(木) 17:41:13.28ID:+XWwkT1H
>>210
(ad-bc)^2+(ac+bd)^2=(aa+bb)(cc+dd),
もよく使うよ。

>>264
6cos(A)+3cos(B)+2cos(C)
=4cos(A)+cos(B)+2{cos(A)+cos(B)+cos(C)}
=4cos(A)+cos(B)+2+8sin(A/2)sin(B/2)sin(C/2) (←補題)
≦4cos(A)+cos(B)+2+8{sin(A/2)}^2+2{sin(B/2)}^2
=4cos(A)+cos(B)+2+4{1−cos(A)}+{1−cos(B)}
=7.

>>279
間違いますた...

>>310
左側は >>156-157
0427prime132垢版2016/04/28(木) 17:52:13.57ID:+XWwkT1H
>>264
【補題】
A+B+C=πのとき、
cos(A)+cos(B)+cos(C)=1+4sin(A/2)sin(B/2)sin(C/2).

(略証)
A/2=α、B/2=β、C/2=γとおくと、cos(α+β)−sinγ=0,
(左辺)−(右辺)=cos(2α)+cos(2β)+cos(2γ)−1−4sinα・sinβ・sinγ
=2cos(α+β)cos(α-β)−2(sinγ)^2−4sinα・sinβ・sinγ
=2{cos(α+β)−sinγ}{cos(α-β)+sinγ}
=0.
0428prime132垢版2016/04/28(木) 19:48:40.92ID:+XWwkT1H
>>320
n=2、3については等号成立。
y=tan(x)は下に凸だから、n≧4のとき
tan(π/2n)≦(4/n)tan(π/8)
=4(√2−1)/n
<1.671811536/n
≦2/{(n-1)・n^[1/(n-1)]},
0429prime132垢版2016/04/29(金) 19:32:34.67ID:aknhMslS
〔第17問題の多項式版〕
fは実係数、n変数、d次斉次の多項式(n≧2、d≧2)でかつ半正値とする。
fが次のいずれかである場合は、fは何個かの多項式の2乗の和として表わせる。(Hilbert)
・2変数
・2次式
・3変数かつ4次式
ここで半正値とは、
任意の実数x1,x2,……,xnに対して、f(x1,x2,……,xn)≧0.

>>251-256
0430prime132垢版2016/04/29(金) 19:44:38.17ID:aknhMslS
>>429
・2次式の場合は、線型代数学の教科書を参照。

//www.amazon.co.jp/%E7%B7%9A%E5%9E%8B%E4%BB%A3%E6%95%B0%E5%AD%A6-%E6%96%B0%E8%A3%85%E7%89%88-%E6%95%B0%E5%AD%A6%E9%81%B8%E6%9B%B8-%E4%BD%90%E6%AD%A6-%E4%B8%80%E9%83%8E/dp/4785313161
//www.shokabo.co.jp/mybooks/ISBN978-4-7853-1301-2.htm
0431prime132垢版2016/04/30(土) 22:57:13.70ID:F01tFwBb
>>326
x+p/2=X,a+p/2=a',b+p/2=b',q-pp/4=q' とおくと、
∫[a〜b] sin(xx+px+q)dx=∫[a'〜b'] sin(XX+q')dX
=sin(q')∫[a'〜b'] cos(XX)dX+cos(q')∫[a'〜b'] sin(XX)dX
=sin(q'){C(b')−C(a')}+cos(q'){S(b')−S(a')}.
ここに
C(x)=∫[0〜x] cos(tt)dt、S(x)=∫[0〜x] sin(tt)dt
(フレネル積分と云うらしい。)
ここで次を使う。
|sin(q')|≦1,
|cos(q')|≦1,
|C(x)|≦|C(√(π/2))|=0.977451424291
|S(x)|≦|C(√π)|=0.894831469484

ぬるぽ
0432prime132垢版2016/05/01(日) 18:05:44.62ID:6BDe1+0Z
続き
=sin(q')・僂+cos(q')・儡
=sin(q'+α)√[|僂|^2+|儡|^2}
≦√[|僂|^2+|儡|^2]
≦2√[max{C(x)^2+S(x)^2}]
=2√[0.889016^2+0.790241^2]
=2.37893
(等号はa'=b'=1.51573)
0433132人目の素数さん垢版2016/05/02(月) 14:49:32.33ID:GTe7iXRO
x≧0、y≧0、x^2+y^2=1 のとき、xy/(x+y) ≦ 1/(2√2) をエレガントに証明できないかな?
0436132人目の素数さん垢版2016/05/02(月) 18:47:38.37ID:GTe7iXRO
では、x≧0、y≧0、x^2+y^2=1 のとき、3/32 ≦ x^{10} +(xy)^5 + y^{10} ≦1 もエレガントに証明できたりしますか? ワクワク…
0437prime132垢版2016/05/02(月) 23:23:04.09ID:YMu89qmg
>>436
左は
x^5+y^5=xx・x^3+yy・y^3≦(xx+yy)・(xx+yy)^(3/2)=(xx+yy)^(5/2),
x^10+(xy)^5+y^10≦(x^5+y^5)^2≦(xx+yy)^5,
0438132人目の素数さん垢版2016/05/03(火) 01:05:52.50ID:uimsCZ8A
>>436
f(p)=((x^p+y^p)/2)^(1/p) は単調増加
f(10)≧f(2), f(5)≧f(2)
より
x^10+(xy)^5+y^10=(x^10+y^10)/2+(x^5+y^5)^2/2≧1/32+1/16=3/32
0441132人目の素数さん垢版2016/05/03(火) 03:27:39.65ID:JxMIUNyU
検索中に exponencial mean というものを見つけたけど、使い道とか、AM,GMとの大小比較とかできるのだろうか?
0442132人目の素数さん垢版2016/05/03(火) 15:48:32.19ID:JxMIUNyU
a、b、c >0 のとき、5/3 < (b+3a)/(a+3b) + (c+3b)/(b+3c) + (a+3c)/(c+3a) < 7 を証明せよ。

エレガントな方法ありますか?
対称式じゃなくて巡回式だから、a≦b≦c、a≦c≦b のときで、上限下限が変わるよね? ( ゚∀゚) テヘッ
0443prime132垢版2016/05/03(火) 17:19:20.71ID:+kp+oyUm
>>438
【補題】
f(p)={(x^p+y^p)/2}^(1/p),
g(p)=(1/2)(x^p+y^p)^(1/p),
とおくと
p>q ⇒ f(p)>f(q)>…>f(1)=g(1)>…>g(q)>g(p)>…

(略証)
p/q=rとおく。

f(p)={(x^p+y^p)/2}^(1/p)
={(X^r+Y^r)/2}^(1/p)  (←x^q=X, y^q=Y)
>{[(X+Y)/2]^r}^(1/p)  (←r>1)
=[(x^q+y^q)/2]^(1/q)
=f(q),

2g(p)=(x^p+y^p)^(1/p)
=(X^r+Y^r)^(1/p)  (←x^q=X, y^q=Y)
={X・X^(r-1)+Y・Y^(r-1)}^(1/p)
<{(X+Y)(X+Y)^(r-1)}^(1/p)  (←r>1)
={(X+Y)^r}^(1/p)
=(x^q+y^q)^(1/q)
=2g(q),

ぬるぽ
0444132人目の素数さん垢版2016/05/03(火) 19:41:13.45ID:+kp+oyUm
>>442

3≦(b+3a)/(a+3b)+(c+3b)/(b+3c)+(a+3c)/(c+3a)<19/3,

上限は、a≪b≪cのとき11/3、a≪c≪bのとき19/3,
最小値は a=b=cのとき,
の希ガス
0445prime132垢版2016/05/03(火) 20:59:30.42ID:+kp+oyUm
>>438
【系】
f(p)={(x^p+y^p)/2}^(1/p),
とおくと
p>q ⇒ 2^(1/q-1/p)・f(q)>f(p)>f(q).
0447132人目の素数さん垢版2016/05/04(水) 02:42:19.93ID:AtoK4P1C
>>446
月刊誌「大学への数学2014年1月号、東京出版」、P.P.56-59 「いろいろな平均とその不等式」、蓑田恭秀

その記事内で、L_pが単調増加することの証明がどうも理解できなかった。
最初のページに M_p(a,b) = {(a^p+b^p)/2}^(1/p) が単調増加であることを証明してるんだけど、それも?
0448132人目の素数さん垢版2016/05/04(水) 02:53:37.23ID:AtoK4P1C
その記事内の Power mean が単調増加することの証明は以下の通り。

 a、bは正の実数、pは実数で、f(p) = log{(a^p + b^p)/2} とおく。
 f''(p) = {a^p b^p (log b -log a)^2} / (a^p+b^p)^2 > 0 より、f(p)は下に凸。

ここまでは分かる。

 y=f(p)のグラフ上の2点(0,0)と(p,f(p))を結ぶ線分の傾き f(p)/p (= log M_p)は、
⇒ p<0とp>0で単調増加、
 したがってM_pも単調増加となる

⇒のところは明らかなのかな?
0449132人目の素数さん垢版2016/05/07(土) 17:48:47.52ID:kYYUhR/a
>>442
aab+bbc+cca=p, abb+bcc+caa=q, abc=u,
とおくと、
p-3u≧0, q-3u≧0,
与式=3+{10(p-3u)+6(q-3u)}/(3p+9q+28u)
  =19/3−(24q+141.333u)/(3p+9q+28u)
  ≦19/3,

a<b<cのときは
q-p=(a-b)(b-c)(c-a)>0,
与式=13/3−{6(q-p)+85.333u}/(3p+9q+28u)
  ≦13/3.
0452132人目の素数さん垢版2016/05/10(火) 21:13:39.92ID:7+TJpR/l
機中の経済学者に「テロ」容疑?=「謎の暗号」は微分方程式−米
http://www.jiji.com/jc/article?k=2016050900019&;g=int


  \
:::::  \            不等式ヲタの両腕に冷たい鉄の輪がはめられた
\:::::  \             
 \::::: _ヽ __   _     外界との連絡を断ち切る契約の印だ。
  ヽ/,  /_ ヽ/、 ヽ_  
   // /<  __) l -,|__) > 「刑事さん・・・、俺、どうして・・・
   || | <  __)_ゝJ_)_>    機内で不等式にハァハァしちゃったのかな?」
\ ||.| <  ___)_(_)_ >
  \| |  <____ノ_(_)_ )   とめどなく大粒の涙がこぼれ落ち
   ヾヽニニ/ー--'/        震える彼の掌を濡らした。
    |_|_t_|_♀__|        
      9   ∂        「その答えを見つけるのは、お前自身だ。」
       6  ∂
       (9_∂          不等式ヲタは声をあげて泣いた。
0453132人目の素数さん垢版2016/05/15(日) 23:39:00.95ID:bcOloU+W
-1 < x、y、z < 1 のとき、1/{(1+x^2)(1+y^2)(1+z^2)} + 1/{(1+x)(1+y)(1+z)}^2 ≧ 2(1-x)(1-y)(1-z) を示せ。
0454132人目の素数さん垢版2016/05/16(月) 18:32:13.74ID:/cRJoxc5
>>453
相加-相乗平均で
(左辺)≧2{1/(1+xx)(1+x)}{1/(1+yy)(1+y)}{1/(1+zz)(1+z)}
=2{(1-x)/(1-x^4)}{(1-y)/(1-y^4)}{(1-z)/(1-z^4)}
≧(右辺).
0455prime132垢版2016/05/16(月) 19:16:07.91ID:/cRJoxc5
>>286
|xxf(1/x)|=|a+bx+cxx|≦|a+cxx|+|bx|≦max{|a+c|,|a|}+|b|.

|f(0)|=|c|≦1,

|f(-1)|=|a-b+c|≦1より
 -1≦a-b+c≦1,
|f(1)|=|a+b+c|≦1より
 -1≦a+b+c≦1
∴ |a+c|+|b|≦1,

・|a+c|≧|a| のときは
|xxf(1/x)|≦|a+c|+|b|≦1,

・|a+c|≦|a|のときは
|xxf(1/x)|≦|a|+|b|≦|c|+(|a+c|+|b|)≦1+1,
0456132人目の素数さん垢版2016/05/17(火) 13:17:49.45ID:veMWuJCB
【不等式を見かけたときの反応で分かる不等式オタク度チェック】

(発病初期)
うれしくなる、勃起する、キタ━┌(_Д_┌ )┐━!!ーと叫ぶ

(軽度だが治療不能)
収集せずにはいられない、メモやコピーを取り、出典を記録する

(全身に転移)
収集済のコレクションにないか確認する、検索して類題を探す

(末期症状)
証明せずにはいられない、緩い不等式は改良せずにはいられない
証明済でも別解を考える、不等式を貼って感染者を増やす
0457132人目の素数さん垢版2016/05/17(火) 13:25:17.40ID:veMWuJCB
a、b、c は正の実数で、a≠bc、b≠ca、c≠ab かつ a^2 + b^2 + c^2 = 1 をみたすとき、

   1/(a^2 - b^2 c^2) + 1/(b^2 - c^2 a^2) + 1/(c^2 - a^2 b^2) ≧ 27/2

たくさん集めても、証明する力が全く身につかないのは何故か (ノ∀`)アチャー
0459132人目の素数さん垢版2016/05/18(水) 00:05:36.13ID:Xe8Zc5En
まとめページの『重み付き累乗平均に関するチェビシェフの不等式』の証明はどうやるんですか?
0462132人目の素数さん垢版2016/05/20(金) 22:35:07.22ID:pIbdaXLN
>>461
測度が重みそのもの。
0463132人目の素数さん垢版2016/05/21(土) 19:22:06.59ID:AUtpbrg4
x、y≧0 に対して、2 √{(x+1)(y+1)} ≧ √(xy+2x+1) + √(xy+2y+1) ≧ √(xy) + √{(x+2)(y+2)}
0464132人目の素数さん垢版2016/05/22(日) 22:20:47.22ID:+Q7R9DuS
今月号(2016.06)のエレガントに不等式の問題がある。一本道なので面白みはないがな。

「内接円(半径r)と外接円(半径R)をもつ四角形があって、R≧(√2)*r を示せ。」

締め切り前なので答えを書かないように。
0465132人目の素数さん垢版2016/05/24(火) 23:13:06.44ID:cY07kHAV
Popoviciu

って何て発音するんですか
0466◆2VB8wsVUoo 垢版2016/05/24(火) 23:14:09.87ID:98ujCcPg


>性犯罪者の増田哲也(50歳・東京都足立区千住寿町)が
>8月4日にJR牟岐線の列車内で、午後4時20分ごろから約50分にわたり、
>徳島県内の女性(21歳・専門学校生)の胸や太ももなどを触った疑いで、
>8月5日未明、県迷惑行為防止条例違反(痴漢行為)容疑で徳島県警阿南署に
>逮捕されました。
>
>性犯罪者 増田哲也の供述
>「夏休み期間に、講演活動を兼ねて旅行していた。好みの女性だったのでムラムラした」
>
0467◆2VB8wsVUoo 垢版2016/05/24(火) 23:14:28.00ID:98ujCcPg


>性犯罪者の増田哲也(50歳・東京都足立区千住寿町)が
>8月4日にJR牟岐線の列車内で、午後4時20分ごろから約50分にわたり、
>徳島県内の女性(21歳・専門学校生)の胸や太ももなどを触った疑いで、
>8月5日未明、県迷惑行為防止条例違反(痴漢行為)容疑で徳島県警阿南署に
>逮捕されました。
>
>性犯罪者 増田哲也の供述
>「夏休み期間に、講演活動を兼ねて旅行していた。好みの女性だったのでムラムラした」
>
0468◆2VB8wsVUoo 垢版2016/05/24(火) 23:14:44.94ID:98ujCcPg


>性犯罪者の増田哲也(50歳・東京都足立区千住寿町)が
>8月4日にJR牟岐線の列車内で、午後4時20分ごろから約50分にわたり、
>徳島県内の女性(21歳・専門学校生)の胸や太ももなどを触った疑いで、
>8月5日未明、県迷惑行為防止条例違反(痴漢行為)容疑で徳島県警阿南署に
>逮捕されました。
>
>性犯罪者 増田哲也の供述
>「夏休み期間に、講演活動を兼ねて旅行していた。好みの女性だったのでムラムラした」
>
0469◆2VB8wsVUoo 垢版2016/05/24(火) 23:15:00.95ID:98ujCcPg


>性犯罪者の増田哲也(50歳・東京都足立区千住寿町)が
>8月4日にJR牟岐線の列車内で、午後4時20分ごろから約50分にわたり、
>徳島県内の女性(21歳・専門学校生)の胸や太ももなどを触った疑いで、
>8月5日未明、県迷惑行為防止条例違反(痴漢行為)容疑で徳島県警阿南署に
>逮捕されました。
>
>性犯罪者 増田哲也の供述
>「夏休み期間に、講演活動を兼ねて旅行していた。好みの女性だったのでムラムラした」
>
0470◆2VB8wsVUoo 垢版2016/05/24(火) 23:15:19.36ID:98ujCcPg


>性犯罪者の増田哲也(50歳・東京都足立区千住寿町)が
>8月4日にJR牟岐線の列車内で、午後4時20分ごろから約50分にわたり、
>徳島県内の女性(21歳・専門学校生)の胸や太ももなどを触った疑いで、
>8月5日未明、県迷惑行為防止条例違反(痴漢行為)容疑で徳島県警阿南署に
>逮捕されました。
>
>性犯罪者 増田哲也の供述
>「夏休み期間に、講演活動を兼ねて旅行していた。好みの女性だったのでムラムラした」
>
0471◆2VB8wsVUoo 垢版2016/05/24(火) 23:15:36.64ID:98ujCcPg


>性犯罪者の増田哲也(50歳・東京都足立区千住寿町)が
>8月4日にJR牟岐線の列車内で、午後4時20分ごろから約50分にわたり、
>徳島県内の女性(21歳・専門学校生)の胸や太ももなどを触った疑いで、
>8月5日未明、県迷惑行為防止条例違反(痴漢行為)容疑で徳島県警阿南署に
>逮捕されました。
>
>性犯罪者 増田哲也の供述
>「夏休み期間に、講演活動を兼ねて旅行していた。好みの女性だったのでムラムラした」
>
0472◆2VB8wsVUoo 垢版2016/05/24(火) 23:16:09.94ID:98ujCcPg


>性犯罪者の増田哲也(50歳・東京都足立区千住寿町)が
>8月4日にJR牟岐線の列車内で、午後4時20分ごろから約50分にわたり、
>徳島県内の女性(21歳・専門学校生)の胸や太ももなどを触った疑いで、
>8月5日未明、県迷惑行為防止条例違反(痴漢行為)容疑で徳島県警阿南署に
>逮捕されました。
>
>性犯罪者 増田哲也の供述
>「夏休み期間に、講演活動を兼ねて旅行していた。好みの女性だったのでムラムラした」
>
0473◆2VB8wsVUoo 垢版2016/05/24(火) 23:16:25.82ID:98ujCcPg


>性犯罪者の増田哲也(50歳・東京都足立区千住寿町)が
>8月4日にJR牟岐線の列車内で、午後4時20分ごろから約50分にわたり、
>徳島県内の女性(21歳・専門学校生)の胸や太ももなどを触った疑いで、
>8月5日未明、県迷惑行為防止条例違反(痴漢行為)容疑で徳島県警阿南署に
>逮捕されました。
>
>性犯罪者 増田哲也の供述
>「夏休み期間に、講演活動を兼ねて旅行していた。好みの女性だったのでムラムラした」
>
0474◆2VB8wsVUoo 垢版2016/05/24(火) 23:16:42.78ID:98ujCcPg


>性犯罪者の増田哲也(50歳・東京都足立区千住寿町)が
>8月4日にJR牟岐線の列車内で、午後4時20分ごろから約50分にわたり、
>徳島県内の女性(21歳・専門学校生)の胸や太ももなどを触った疑いで、
>8月5日未明、県迷惑行為防止条例違反(痴漢行為)容疑で徳島県警阿南署に
>逮捕されました。
>
>性犯罪者 増田哲也の供述
>「夏休み期間に、講演活動を兼ねて旅行していた。好みの女性だったのでムラムラした」
>
0475◆2VB8wsVUoo 垢版2016/05/24(火) 23:16:59.52ID:98ujCcPg


>性犯罪者の増田哲也(50歳・東京都足立区千住寿町)が
>8月4日にJR牟岐線の列車内で、午後4時20分ごろから約50分にわたり、
>徳島県内の女性(21歳・専門学校生)の胸や太ももなどを触った疑いで、
>8月5日未明、県迷惑行為防止条例違反(痴漢行為)容疑で徳島県警阿南署に
>逮捕されました。
>
>性犯罪者 増田哲也の供述
>「夏休み期間に、講演活動を兼ねて旅行していた。好みの女性だったのでムラムラした」
>
0476prime132垢版2016/05/26(木) 19:13:00.93ID:FOvQsbPl
>>463
左側は凸性から出る。
右側
x(y+2)+1=a、(x+2)y+1=b、xy=c、(x+2)(y+2)=d とおくと
a+b-c-d=-2,   …(*)
ab-cd=ab-(a-1)(b-1)=a+b-1,  …(**)
∴(√a+√b+√c+√d)(√a+√b-√c-√d)
=(√a+√b)^2−(√c+√d)^2
=(a+b-c-d)+2√ab−2√cd
=2√ab−2√cd−2    ←(*)
=2(ab-cd)/(√ab+√cd)−2
=2(a+b-1)/{√ab+√[(a-1)(b-1)]}−2  ←(**)
≧4(a+b-1)}/{(a+b)+(a-1)+(b-1)}−2
=0,

ぬるぽ
0477prime132垢版2016/05/26(木) 19:49:26.86ID:FOvQsbPl
>>476
後半は
cd=(a-1)(b-1)=ab-(a+b)+1≦{√(ab)−1}^2,
∴√(ab)−√(cd)−1≧0  (←ab≧1)
の方が早いな…
0479132人目の素数さん垢版2016/05/27(金) 11:41:31.00ID:/KLM3aoL
a、b、c、d、e、f、g ≧0 かつ a+b+c+d+e+f+g=1 のとき、
max{a+b+c、b+c+d、c+d+e、d+e+f、e+f+g} ≧?
0481132人目の素数さん垢版2016/05/27(金) 14:01:05.69ID:Lks76rr8
>>477
〔補題〕
 a = a1 + a2 + … + an, (aj≧0)
 b = b1 + b2 + … + bn, (bk≧0)
のとき、コーシーにより
 √(a・b) ≧ √(a1・b1) + √(a2・b2) + …… + √(an・bn),
0482132人目の素数さん垢版2016/05/30(月) 19:19:54.08ID:1GyiVnWU
〔加比の理〕
a1/b1 > c1/d1 >0,
a2/b2 > c2/d2 >0,
のとき次を示せ。
(a1+a2)/(b1+b2) > (c1+c2)/(d1+d2),
0483132人目の素数さん垢版2016/05/30(月) 19:32:26.46ID:1GyiVnWU
>>482
0 < |d| < |e| < A, B のとき
(A-d)/(A-e) > (A+e)/(A+d),
(B-d)/(B+e) > (B-e)/(B+d),
よって
(C-2d)/C > C/(C+2d) ???
(但し、C=A+B)

(シンプソンのパラドックス)
0485132人目の素数さん垢版2016/06/01(水) 17:02:22.35ID:4y470XDY
Anthony B. Atkinson: "Inequality"

「21世紀の不平等 − 格差をあきらめない15の方法」
山形浩生/森本正史(訳)
東洋経済新報社
2015/12/11
464p.
3888円

「不平等研究の新しい基本書。日本の格差対策にも本書の15の提案は必読だ。」
−−−大竹文雄(大阪大学教授)
0487132人目の素数さん垢版2016/06/02(木) 21:43:19.02ID:nY0di/bv
>>2の[6] P.16 問1.3(b) について。
n≧2、0≦a_k<1 に対して、Π[k=1 to n] (1-a_k) > 1 - Σ[k=1 to n] a_k

これって、不等号は> じゃなくて ≧ じゃないの?
等号成立条件は、n個のa_kのうちn-1個以上が0のときでok?
0490132人目の素数さん垢版2016/06/13(月) 00:57:49.88ID:4lqZeqj9
(1)
正の数 a, b, c が abc=1 をみたすとき、
\sqrt{(a+1)/(a^2-a+1)} + \sqrt{(b+1)/(b^2-b+1)} + \sqrt{(c+1)/(c^2-c+1)} ≦ 3*\sqrt{2}

(2)
実関数 f は C^2[0,1] 級、f(1/2)=0 とする
∫[0,1] {f''(x)}^2 dx ≧ 320*(∫[0,1] f(x) dx)^2
0491132人目の素数さん垢版2016/06/16(木) 10:48:44.76ID:bzeKqK4U
z、wを複素数、cを正の実数とするとき、|z+w|^2 ≦ (1+c)*|z|^2 + (1+ 1/c)*|w|^2

  ('A`) ,..;:〜''"
 ノ( ヘヘ ,,.、;;:〜'''
0494132人目の素数さん垢版2016/06/20(月) 18:29:42.26ID:5rd60K6N
>>464
締め切り後だからいいな。次の補題を使った例。

〔補題〕
△の2辺の長さをp,qとすると、(△の面積)≦pq/2.

∵pを底辺と見れば、高さはq以下
(△の面積) = (底辺)(高さ)/2≦pq/2.
0495132人目の素数さん垢版2016/06/20(月) 18:40:29.04ID:5rd60K6N
>>464
6月号の解答例

◇ABCD=Sとおく。

S=△ABC+△CDA≦(AB・BC+CD・DA)/2,
S=△BCD+△DAB≦(BC・CD+DA・AB)/2,
平均すると
S≦(AB+CD)(BC+DA)/4≦(L/4)^2 … (1)
ここに、周長L = (AB+CD)+(BC+DA) とおいた。

また、対角線AC、BDの交点をXとすると、
S=△ABX+△BCX+△CDX+△DAX
≦(AX+XC)(BX+XD)/2
=AC・BD/2
≦2RR … (2)
∵ AC≦2R、BD≦2R

(1)×(2)より
S≦L(R/√2)/2 … (3)


一方、内接円の中心をIとすると、
S=△ABI+△BCI+△CDI+△DAI
=(AB+BC+CD+DA)・r/2
=Lr/2 … (4)
これと(3)を比べる。
0496132人目の素数さん垢版2016/06/21(火) 06:40:01.55ID:Q7b/QKg5
適当に検索して見かけた不等式で、スポポビッチみたいな名前のやつ何だっけ?
ふと思い出して検索したけど見つけられん…
0497132人目の素数さん垢版2016/06/21(火) 06:43:16.84ID:Q7b/QKg5
>>464
私は、S、R、r をすべて4辺の長さ a、b、c、d で表して、相加平均・相乗平均の不等式を用いた。
0498132人目の素数さん垢版2016/06/21(火) 09:15:06.44ID:ARKwvWwv
popoviciu's inequality ではなくて?
0499132人目の素数さん垢版2016/06/21(火) 09:15:32.38ID:ARKwvWwv
wvWwv
0501132人目の素数さん垢版2016/06/21(火) 18:51:48.07ID:Q7b/QKg5
>>498
( ゚∀゚) ソレダ!( ゚∀゚) スポポビッチ、ヌポポビッチ、ウホホビッチ!

>>500
四角形をABCDとし、AB=a、BC=b、CD=d、DA=d、内接円の中心をOとおく。

(1) R、r を S で表す。面積公式と、正弦定理より、

S = △ABC + △ADC
  = (1/2)*ab*sinC + (1/2)*cd*sin(π-B)
  = AC(ab+cd)/4R

S = △OAB + △OBC + △OCD + △ODA
  = r(a+b+c+d)/2

∴ R/r = (a+b+c+d)(ab+cd)AC/(8*S^2)  ……(a)


(2) 邪魔な AC、Sを そげぶ(ブチ殺す)。 余弦定理を用いて、

△ABCに対して、AC^2 = a^2 +b^2 - 2ab*cosB
△ADCに対して、AC^2 = c^2 +d^2 + 2cd*cosB

∴ AC = √{(ac+bd)(ad+bc)/(ab+cd)}  ……(b)

一方、cosB = (a^2+b^2-c^2-d^2)/{2*(ab+cd)} より、
sinB = {√(b+c+d-a)(c+d+a-b)(d+a+b-c)(a+b+c-d)}/{2*(ab+cd)}

S = △ABC + △ADC
 = (1/2)*(ab+cd)*sinB
 = √(b+c+d-a)(c+d+a-b)(d+a+b-c)(a+b+c-d)

四角形ABCDが円に外接するので a+c = b+d をみたす。
これより b+c+d-a = 2c などを上式に代入する。

∴ S = √(abcd)  ……(c)

(3) 相加平均・相乗平均で息の根を止める。
(a)に(b)(c)を代入し、分子の四つの括弧にそれぞれAM-GM。

R/r = (a+b+c+d)*√{(ac+bd)(ad+bc)/(ab+cd)}/(8*abcd) ≧ √2 
0502132人目の素数さん垢版2016/06/21(火) 19:11:05.41ID:olhvI6Km
こういう文体で送っても真面目に採点してくれるの?
0504132人目の素数さん垢版2016/06/21(火) 19:19:37.72ID:Q7b/QKg5
検索すると出題者は数オリ出場者みたいだから、この手の問題が好きなんだろうなあ。

それより出題者は、エレガントな解答を用意してるんだろうか?
計算の羅列で解説を終わったら、オナラかましたくなるな。

( ゚∀゚) プゥ
ノヽノ) =3'A`)ノ ヒャー
  くく へヘノ ← 出題者
0506132人目の素数さん垢版2016/06/24(金) 03:28:52.14ID:W/Uj9dP4
はい
http://www.dotup.org/uploda/www.dotup.org915431.pdf

わざわざ書いたけど,初等的な不等式I とほとんど同じ証明法なんだよな
しかもこっちだと重み付きの一般変数の場合もあるし
0508132人目の素数さん垢版2016/06/24(金) 03:50:44.16ID:W/Uj9dP4
人いたのか

>>507
a*はaの要素を大きい順に並べたベクトルね
0509132人目の素数さん垢版2016/06/24(金) 04:07:55.28ID:W/Uj9dP4
これでかなりの数の初等不等式を証明できるよね
問題は使いこなせるかどうかだけど

【不等式類】
Bernoulli の不等式,Cauthy の不等式,Chebyshev の不等式,Holder の不等式,Jensen の不等
Karamata の不等式,Minkovski の不等式,Muirhead の不等式,Nesbitt の不等式,PowerMean 不等式
QM-AM-GM-HM 不等式,Radon の不等式(次数1),Rearrangement 不等式,Schur の不等式

【メソッド】
ABC,CD3,CDN,CRT,isolated fudging,normalization
Lagrange Multipliers,SD3,SMV,SOS,UMV,symmetric separation,uvw,置換
0510132人目の素数さん垢版2016/06/24(金) 04:28:46.11ID:JzDlPDy9
>>509
Radon の不等式(次数1)は知らないです…。

ヌポポビッチを改造した。 改造せずにはいられない! 不等式ヲタ-ヌポポビッチの不等式 と名づけよう!

凸関数 f に対して、
{ f(a) + f(b) + f(c) + f((a+b+c)/3) }/4 ≧ [ {f(a) + f(b) + f(c)}/3 + f((a+b+c)/3) ]/2 ≧ { f((a+b)/2) + f((b+c)/2) + f((c+a)/2) }/3

左側は Jensenの不等式 {f(a) + f(b) + f(c)}/3 ≧ f((a+b+c)/3) を変形しただけ。
右側は ヌッポビッチそのまま。
0511132人目の素数さん垢版2016/06/24(金) 07:23:09.91ID:JzDlPDy9
もう少し改造。ぬるぽビッチの不等式と命名。

【ぬるぽビッチの不等式】

凸関数 f に対して、
{ f(a) + f(b) + f(c) - f((a+b+c)/3) }/2
≧ { f(a) + f(b) + f(c) + f((a+b+c)/3) }/4
≧ [ {f(a) + f(b) + f(c)}/3 + f((a+b+c)/3) ]/2
≧ { f((a+b)/2) + f((b+c)/2) + f((c+a)/2) }/3

          ___         
    |┃三 ./  ≧ \   
    |┃   |::::  \ ./ | 
    |┃ ≡|::::: (● (● |  不等式と聞ゐちゃぁ
____.|ミ\_ヽ::::... .ワ......ノ     黙っちゃゐられねゑ!
    |┃=__    \           ハァハァ…
    |┃ ≡ )  人 \ ガラッ
0512132人目の素数さん垢版2016/06/24(金) 12:40:59.46ID:W/Uj9dP4
>>510
Radon の不等式って言ってるけど,Cauthy を変形したものだよ
証明も Cauthy で済ませてると思う
一次の場合は Bergstom's inequality とか,>>8 だと Engel 型の Cauthy の不等式って書いてある
0513132人目の素数さん垢版2016/06/24(金) 13:04:01.41ID:W/Uj9dP4
もしかして徹夜でぬるぽビッチやってたのか

何か実例に当てはめられないかね
0514132人目の素数さん垢版2016/06/24(金) 13:52:35.40ID:JzDlPDy9
>>512
なるほど、これですか。 この形にする必要があるのかと小一時間…。


【Engel 型の Cauthy の不等式】

x、y、z > 0 のとき、a^2/x + b^2/y + c^2/z ≧ (a+b+c)^2/(x+y+z)
0515132人目の素数さん垢版2016/06/24(金) 13:57:31.91ID:JzDlPDy9
【ぬるぽビッチの不等式・改】

凸関数 f に対して、
{ f(a) + f(b) + f(c) - f((a+b+c)/3) }/2
≧ { f(a) + f(b) + f(c) + f((a+b+c)/3) }/4
≧ [ {f(a) + f(b) + f(c)}/3 + f((a+b+c)/3) ]/2
≧ { f((a+b)/2) + f((b+c)/2) + f((c+a)/2) }/3
≧ f((a+b+c)/3)


1番目と2番目の不等号は、Jensenの不等式 {f(a) + f(b) + f(c)}/3 ≧ f((a+b+c)/3) を変形しただけ。
3番目がヌポポビッチ。
4番目の不等号は、Jensenの不等式のa、b、cに代入しただけ。
     ___
   /|_   |
   ||. (゚∀゚ ) | ぬるぽと聞いて・・・
   ||oと.  U|
   || |(__)J|
   ||/彡 ̄ ガチャ
0516132人目の素数さん垢版2016/06/24(金) 17:40:22.34ID:W/Uj9dP4
>>514
まあ見やすいしそのまま不等式に適用できるしいいじゃん
一般の Radon の不等式は Holder の不等式と同じだけど,たぶん別々に発見されたんだろうな
詳しくは知らないけど
0517132人目の素数さん垢版2016/06/24(金) 17:41:35.44ID:W/Uj9dP4
>>490
(1)
a≧b≧c として一般性を失わない。
f(x) = \sqrt[(x+1)/(x^2-x+1) とおく。

・a ≦ 32.8295 のとき
g(x) = (-1/(2\sqrt[2]) * log[x] + \sqrt[2]) とおくと f(x) ≦ g(x) が成り立つ。よって
Σ[cyc] f(a) ≦Σ[cyc] g(a) = 3\sqrt[2]
となる。

・a ≧ 32.8295 のとき
a > 32.8295,0 < c < 1/\sqrt[32.8295] = 0.1745 である。よって
f(a) ≦ f(32.8295) = 0.1798
f(b) ≦ f(0.7320) = 1.4678
f(c) ≦ f(0.1745)= 1.1714
つまり
f(a) + f(b) + f(c) ≦ 0.1798 + 1.4678 + 1.1714 = 2.8190 ≦ 3\sqrt[2]
となる。

どや
0518132人目の素数さん垢版2016/06/24(金) 17:50:54.05ID:W/Uj9dP4
普通に g(x) = (-1/(2\sqrt[2])) * x + \sqrt[2] + 1/(2\sqrt[2]) で上から抑えればよかったな
0519132人目の素数さん垢版2016/06/24(金) 17:54:33.54ID:W/Uj9dP4
ああ違うかなんでもない
0520132人目の素数さん垢版2016/06/24(金) 19:14:35.39ID:JzDlPDy9
>>517
いつもながら流石ですな。

ラドンは検索しても殆ど出てこなくて挫けそうになった。
http://www.icstm.ro/DOCS/josa/josa_2008_1/a.10_GENERALIZATIONS_AND_REFINEMENTS_FOR_BERGSTROM_AND_RADONS_INEQUALITIES.pdf

P.P.1-2 Radon’s inequality
P. 3 for refinement of Radon’s inequality
P.P.3-4 refinement of Bergstr¨om’s inequality
P. 4 the generalization of Radon’s inequality
P. 5 the generalization of Radon’s inequality - a variant

また一つコレクションが増えたな。
0521132人目の素数さん垢版2016/06/24(金) 20:03:05.61ID:RC5mmS3F
>>505-506
weighted Popoviciuの略証
(px+qy)/(p+q)=X,
(px+rz)/(p+r)=Y,
(qy+rz)/(q+r)=Z,
(px+qy+rz)/(p+q+r)=m とおく。

(i) x,y<m<z のとき
X<m<Y,Z
p・f(x) + q・f(y)≧(p+q)f(X),
r・f(z) + (p+q+r)f(m)≧(p+r)f(Y) + (q+r)f(Z),
辺々たす。

(ii) x<m<y,z のとき
X,Y<m<Z
p・f(x) + (p+q+r)f(m)≧(p+q)f(X) + (p+r)f(Y),
q・f(y) + r・f(z)≧(q+r)f(Z),
辺々たす。

佐藤(訳)「美しい不等式の世界」朝倉書店、演習問題1.89
0522132人目の素数さん垢版2016/06/24(金) 20:13:17.88ID:W/Uj9dP4
>>457
a≧b≧c として一般性を失わない。
斉次化して整理すれば不等式は
27Σa^4b^4(a^4+b^4) + 54Σa^6b^6 ≧ 46a^2b^2c^2Σa^6 + 30a^2b^2c^2Σa^2b^2(a^2+b^2) + 6a^4b^4c^4
と同値。b = a + p, c = a + q (p, q≧0) を代入すれば不等式は
(27*p^4*q^8+54*p^6*q^6+27*p^8*q^4)
+a*(108*p^3*q^8+216*p^4*q^7+324*p^5*q^6+324*p^6*q^5+216*p^7*q^4+108*p^8*q^3)
+a^2*(116*p^2*q^8+864*p^3*q^7+1536*p^4*q^6+1944*p^5*q^5+1536*p^6*q^4+864*p^7*q^3+116*p^8*q^2)
+a^3*(16*p*q^8+928*p^2*q^7+3984*p^3*q^6+6192*p^4*q^5+6192*p^5*q^4+3984*p^6*q^3+928*p^7*q^2+16*p^8*q)
+a^4*(8*q^8+128*p*q^7+3848*p^2*q^6+11808*p^3*q^5+15024*p^4*q^4+11808*p^5*q^3+3848*p^6*q^2+128*p^7*q+8*p^8)
+a^5*(64*q^7+592*p*q^6+10096*p^2*q^5+22848*p^3*q^4+22848*p^4*q^3+10096*p^5*q^2+592*p^6*q+64*p^7)
+a^6*(272*q^6+1760*p*q^5+17360*p^2*q^4+28800*p^3*q^3+17360*p^4*q^2+1760*p^5*q+272*p^6)
+a^7*(736*q^5+3232*p*q^4+19456*p^2*q^3+19456*p^3*q^2+3232*p^4*q+736*p^5)
+a^8*(1268*q^4+3584*p*q^3+11724*p^2*q^2+3584*p^3*q+1268*p^4)
+a^9*(1360*q^3+1560*p*q^2+1560*p^2*q+1360*p^3)
+a^10*(720*q^2-720*p*q+720*p^2)
≧0
となるが,これは明らかに成り立つ。
0523132人目の素数さん垢版2016/06/24(金) 20:13:57.50ID:W/Uj9dP4
明後日 TOEIC があるというのに何をやっとるんだ!
0524132人目の素数さん垢版2016/06/24(金) 20:19:41.98ID:W/Uj9dP4
こんな解き方(?)じゃ証明力つかなくてもしょうがないな…
0525132人目の素数さん垢版2016/06/24(金) 20:22:13.49ID:JzDlPDy9
5月にTOEICを初めて受けた。
リスニングは全く聞き取れず、リーディングは1問だけ分かった。残りは適当に埋めた。
Lis130、Read170、合計300点。
4択をランダムに埋めても990÷4≒250なのに、300点。
1問しか分からないのに300点とは、いかに検定試験が当てにならないものかよく分かった。

>>521
( ゚∀゚)キタコレ
0526132人目の素数さん垢版2016/06/24(金) 20:41:33.98ID:W/Uj9dP4
>>521
こういうスマートな解き方できるのはいいね
0527132人目の素数さん垢版2016/06/24(金) 21:49:56.15ID:W/Uj9dP4
>>522
符号気にしないで分母払ったから間違えてるな
0529132人目の素数さん垢版2016/06/25(土) 03:47:35.78ID:bGJ+ot70
>>521
> (i) x,y<m<z のとき
> X<m<Y,Z

> r・f(z) + (p+q+r)f(m)≧(p+r)f(Y) + (q+r)f(Z),

ちょっと分からないので、教えてください。

疑問(1)
x、y<m<z のとき、X<mは分かるけど、m<Y、Zはどうやって示すのですか?

疑問(2)
r・f(z) + (p+q+r)f(m)≧(p+r)f(Y) + (q+r)f(Z) はどこから?
0530132人目の素数さん垢版2016/06/25(土) 05:34:40.62ID:loQiTtfh
>>528
それも面白みがないじゃん
0531132人目の素数さん垢版2016/06/25(土) 05:41:14.30ID:loQiTtfh
[2]の数オリ本でも書いてあるけどバンチという言葉って使われてるの?
Schurheadの方が多い気がするけど…
0532521垢版2016/06/25(土) 18:12:23.22ID:nJzcBttQ
>>529

回答(1)
(pX+qX+rz)/(p+q+r) = m, より m-X 〜 z-m,
(pY+qy+rY)/(p+q+r) = m, より Y-m 〜 m-y,
(px+qZ+rZ)/(p+q+r) = m, より Z-m 〜 m-x,
(〜は同符号の意味)

回答(2)
{(Y-m)/(z-m)}f(z) + {(z-Y)/(z-m)}f(m) ≧ f(Y),
{(Z-m)/(z-m)}f(z) + {(z-Z)/(z-m)}f(m) ≧ f(Z),
(上式)・(p+r)+(下式)・(q+r) から。
0533521垢版2016/06/25(土) 18:17:57.53ID:nJzcBttQ
>>529

回答(1)
{(p+q)X+rz}/{(p+q)+r} = m, より m-X 〜 z-m,
{(p+r)Y+qy}/{(p+r)+q} = m, より Y-m 〜 m-y,
{px+(q+r)Z}/{p+(q+r)} = m, より Z-m 〜 m-x,
(〜は同符号の意味)

変わり映えしない…
0534132人目の素数さん垢版2016/06/26(日) 00:44:47.35ID:HxtdGT1C
正の実数 a, b, c が a+b+c=3 を満たすとき,次の不等式を示せ。
(4a+5)/(a^2+2a+3) + (4b+5)/(b^2+2b+3) + (4c+5)/(c^2+2c+3) ≦ 9/2

まったくもって綺麗じゃないけど
0535132人目の素数さん垢版2016/06/26(日) 01:00:55.95ID:HxtdGT1C
ちなみに >>534 は a+b+c=3 じゃなく abc=1 でも成り立ちますね
0536132人目の素数さん垢版2016/06/26(日) 01:30:42.22ID:HxtdGT1C
すいません正に限る必要なかったですね書き直します。

(1) 実数 a, b, c が a + b + c = 3 を満たすとき,次の不等式を示せ。
(4a+5)/(a^2+2a+3) + (4b+5)/(b^2+2b+3) + (4c+5)/(c^2+2c+3) ≦ 9/2

(2) 正の実数 a, b, c が abc = 1 を満たすとき,上の不等式を示せ。
0538132人目の素数さん垢版2016/06/26(日) 11:01:58.80ID:l4WYJ2nA
実数 a、b、x、y に対して、(a^2 + ab + b^2)(x^2 + xy + y^2) ≧ {ax + (ay+bx)/2 + by}^2
( ゚∀゚) ウヒョッ!
0539132人目の素数さん垢版2016/06/26(日) 17:57:03.72ID:HJ99ghG/
>>538
(左辺)
= {3(a+b)^2 +(a-b)^2} {3(x+y)^2 +(x-y)^2}/16
≧{3|a+b||x+y| + |a-b||x-y|}^2 /16
≧{3(a+b)(x+y) + (a-b)(x-y)}^2 /16
=(右辺),

ハァハァ…
0542132人目の素数さん垢版2016/06/27(月) 08:01:32.78ID:WslfN4nO
正の数 a、b、c に対して、a^6 + b^6 + c^6 -2a^3*b^3 - 2b^3*c^3 - 2c^3*a^3 + 3a^2*b^2*c^2 ≧0
( ゚∀゚) ヌルポ!
0545132人目の素数さん垢版2016/06/27(月) 16:17:35.34ID:YK4TWhlE
>>542
f(t)=exp(6t) は下に凸。
∴ぬるぽビッチにより
exp(6x)+exp(6y)+exp(6z)-2exp(3(x+y))-2exp(3(y+z))-2exp(3(z+x))+3exp(2(x+y+z))≧0,
exp(x)=a、exp(y)=b、exp(z)=cとおく。
0546132人目の素数さん垢版2016/06/27(月) 16:56:07.79ID:WslfN4nO
>>545
ヌルポビッチから作ったのがバレバレでしたね。

>>520のP.2のRadonの不等式の証明で、
d_{n+1} - d_n の計算過程において、n=2のRadonの不等式を使っているけど、
自分の証明に自分を使ってるような…
0547132人目の素数さん垢版2016/06/27(月) 21:36:37.50ID:sKl6oXWf
おい何でぬるぽビッチが定着してるんだ
0548132人目の素数さん垢版2016/06/27(月) 23:39:17.06ID:sKl6oXWf
>>546
n=2 は別途証明してるからその部分はいいんじゃないの
参考文献 [6] を参照しろって書いてあるじゃん(元の論文は読んでないけど)
それより
n = 2, p = 3,
x = (1, 1, 2),
a = (2, 2, 4)
とすると
d[n + 1] - d[n] = -1/2 < 0
にならない?
0549132人目の素数さん垢版2016/06/28(火) 00:38:13.33ID:dgtzi8wj
>>542
(左辺)=(a^2 + b^2 + c^2 + ab + bc + ca)(Σa^4 + abcΣa - Σab(a^2 + b^2)) ≧ 0 = (右辺)
後ろの不等式はシューアから

ぬるぽビッチの方が綺麗だね
0550132人目の素数さん垢版2016/06/28(火) 01:27:33.35ID:uqZezJ5q
>>548
n = 2, p = 3、x = (1, 1, 2)、a = (2, 2, 4) とすると、
d[n + 1] - d[n]
 = (x_3)^4/(a_3)^3 + (x_1 + x_2 + x_3)^4/(a_1 + a_2 + a_3)^3 - (x_1 + x_2)^4/(a_1 + a_2)^3
 = 1/4 + 1/2 - 1/4
 = 1/2 > 0
大丈夫っぽい。

>>549
因数分解できるのか…。 すげえな!
ぬるぽビッチって便利だな。
0551132人目の素数さん垢版2016/06/28(火) 01:55:56.39ID:dgtzi8wj
>>550
勘違いしてた二乗じゃなく p + 1 乗だよな
何でこんなミスに気づかなかったんだ…
0555132人目の素数さん垢版2016/06/28(火) 21:17:01.28ID:uqZezJ5q
>>520
4ページ目の定理8「ラドンの一般化」の等号成立条件は、
power meanを使っている時点で、a_1 = … = a_n が要るから、
a_1/b_1 = … = a_n/b_n じゃなくて、a_1 = … = a_n かつ b_1 = … = b_n じゃない?
0556132人目の素数さん垢版2016/06/28(火) 21:19:23.83ID:uqZezJ5q
>>555
記号がおかしかったな。考えている途中で記号が置き換わってたわ。
x_1/a_1 = … = x_n/a_n じゃなくて、x_1 = … = x_n かつ a_1 = … = a_n じゃない?
0557132人目の素数さん垢版2016/06/28(火) 21:22:29.12ID:dgtzi8wj
ヘルダーが出来るのは p が整数の場合だけでしょ
0558132人目の素数さん垢版2016/06/28(火) 23:37:55.65ID:dgtzi8wj
そうだね等号成立は正しくは次のいずれかが成り立つ時だね
・q=1 かつ x[1]/a[1] = … = x[n]/a[n]
・x[1] = … = x[n] かつ a[1] = … = a[n]
0560132人目の素数さん垢版2016/06/29(水) 06:56:55.95ID:hgoh04AG
>>558
試しに x_1/a_1 = … = x_n/a_n = c とおいて、拡張Radonの不等式に代入したら、

(左辺) = c^{p+q} Σ(a_k)^q
(右辺) = (c^{p+q}/n^{q-1})*(Σa_k)^q

∴(左辺) - (右辺) = n*c^{p+q}*{(1/n)Σ(a_k)^q - ((1/n)Σa_k)^q}

となって、p乗平均 と 算術平均のp乗 の差なので、これだけでは等号は成立しないよね。
0562132人目の素数さん垢版2016/06/29(水) 10:59:13.06ID:SxjNJqQ2
>>559
重みが実数ならいいんだよね
数列の個数が整数個か
何か頭ごちゃごちゃになってきた
0578132人目の素数さん垢版2016/06/29(水) 18:17:05.21ID:xnYT2GDL
>>549
変数が実数の場合も成立ですね。

もっとも負になり得る項は
-2(xy)^3≧-2|xy|^3
だけなので、正の場合に成り立つことが(ぬるぽビッチ等で)分かれば
実数の場合も成り立ちまつが。
0582132人目の素数さん垢版2016/06/30(木) 16:52:25.09ID:sy+V6Hgj
>>547

 (゚∀゚) ソレダ! (゚∀゚) スポポビッチ、ヌポポビッチ、ウホホビッチ!

から派生したらしい…
0583132人目の素数さん垢版2016/06/30(木) 18:20:26.57ID:sy+V6Hgj
>>501
◇ABCDの面積をSとする。

◇が外接円(半径R)をもつとき、
S = √{(-a+b+c+d)(a-b+c+d)(a+b-c+d)(a+b+c-d)},
R^2 = (ac+bd)(ad+bc)(ab+cd)/(4S)^2 = {uu + [(a+b+c+d)^2-4t]v}/(4S)^2,
ここに、t=ab+ac+ad+bc+bd+cd、u=abc+abd+acd+bcd、v=abcd.

◇が内接円(半径r)をもつとき、
a+c=b+d,
-a+b+c+d=2c, a-b+c+d=2d, a+b-c+d=2a, a+b+c-d=2b,
r = 2S/(a+b+c+d) = 2(√v)/(a+b+c+d),

ここで
G(a b c d) = {16abcd - (-a+b+c+d)(a-b+c+d)(a+b-c+d)(a+b+c-d)}/(a+b+c+d)
= (a-b)(a-c)(a-d) + (b-a)(b-c)(b-d) + (c-a)(c-b)(c-d) + (d-a)(d-b)(d-c)
= (a+b-c-d)(a-b+c-d)(a-b-c+d)
= (a+b+c+d)^3 -4(a+b+c+d)t +8u,
とおく。
内接円をもつとき G=0 ゆえ
R^2 = {uu + (G-8u)v/(a+b+c+d)}/{16v-(a+b+c+d)G},
= {uu -8uv/(a+b+c+d)}/(16v),
よって
R^2 - 2r^2 = {uu -8uv/(a+b+c+d)}/(16v) - 8v/(a+b+c+d)^2
= {u -16v/(a+b+c+d)}{u +8v/(a+b+c+d)}/(16v)
≧0,
0584132人目の素数さん垢版2016/07/02(土) 17:56:03.82ID:N5gz9LFw
6√2・√(4 -√6 -√2)<π<2(√3 -1)(√3+√2 -1),

 3.132628613         3.142349131

(略証)
左は >>17
右は >>157
0585132人目の素数さん垢版2016/07/02(土) 21:28:35.90ID:vbadd4D+
随分汚い不等式だな
0586132人目の素数さん垢版2016/07/03(日) 00:17:00.07ID:LZWlka8j
H(x, y) で x, y の調和平均を表す。
正の実数 a[1], …, a[n] および b[1], … b[n] に対して次の不等式を示せ。
H(a[1], b[1]) + … + H(a[n], b[n]) ≦ H(a[1] + … + a[n], b[1] + b[n])
0587132人目の素数さん垢版2016/07/03(日) 00:18:03.58ID:LZWlka8j
なんか途中で書き込みしてしまった…

H(x, y) で x, y の調和平均を表す。
正の実数 a[1], …, a[n] および b[1], … b[n] に対して次の不等式を示せ。

H(a[1], b[1]) + … + H(a[n], b[n]) ≦ H(a[1] + … + a[n], b[1] + … + b[n])
0588132人目の素数さん垢版2016/07/03(日) 00:29:42.54ID:LZWlka8j
正の実数 a[1][1], … a[n][n] の対して次の不等式を示せ。
ΣH(a[1][i], …, a[n][i]) ≦ H(Σa[1][i], …, Σa[n][i])
0589132人目の素数さん垢版2016/07/03(日) 00:40:46.84ID:LZWlka8j
間違えた
こっちのほうがスッキリしてるね

ベクトル x に対して H(x) は各要素の調和平均を表すとする。
各要素が正である n 個の m 項ベクトル a[1], …, a[n] に対し,次の不等式を示せ。
  H(a[1]) + … + H(a[n]) ≦ H(a[1] + … + a[n])
0590132人目の素数さん垢版2016/07/03(日) 01:06:47.24ID:LZWlka8j
駄目だ興奮してまた間違えてる
まだ違ってたら察してくれ

n×m 行列 X = [a[1] … a[n]]t = [[b[1] … b[m]] に対し,次の不等式を示せ。
  H(a[1] + … + a[n]) ≧ H(b[1]) + … + H(b[m])

累乗平均や和ではなくまた別の平均に置き換えるとどうなるだろうか
0591132人目の素数さん垢版2016/07/03(日) 01:41:26.49ID:LZWlka8j
はぁ…>>589が正しい…これ以降は察して…
0592132人目の素数さん垢版2016/07/03(日) 03:51:06.05ID:LZWlka8j
スレ汚してしまって申し訳ない>>590がやはり正しい

次数 r の一般の累乗平均 M に対して
  M(a[1] + … + a[n]) - (M(b[1]) + … + M(b[m]))
の符号はどうなるのだろうか
・r > 1のとき:負
・r = 1 のとき:0
・r < 1 のとき:正
な気がする
r = 1 は明らかで r = 0 はヘルダーになる

それとももう結果は知られてるのか
0593132人目の素数さん垢版2016/07/03(日) 22:56:42.54ID:sQrzmkLN
>>587

H(a,b) = 2ab/(a+b) = (a+b)/2 - |a-b|^2 /{2(a+b)},
なので、本題は
Σ[i=1,n] (a_i-b_i)^2 /(a_i+b_i) ≧ {Σ[j=1,n] (a_j-b_j)}^2 /{Σ[k=1,n] (a_k+b_k)},
に帰着するが、これはコーシーで簡単に出そう。
0594132人目の素数さん垢版2016/07/03(日) 23:36:50.30ID:LZWlka8j
ちなみに
・n=2, m=2 : Poland 1993
・n-2, m=3 : KMO Weekend Program 2007
です
0595132人目の素数さん垢版2016/07/04(月) 00:35:44.47ID:nl9wXVG2
>>592
n×m 行列 X = [a[1] … a[n]]t = [[b[1] … b[m]] に対し,次の不等式を示せ。
  H(b[1] + … + b[m]) ≧ H(b[1]) + … + H(b[m])
0596132人目の素数さん垢版2016/07/04(月) 17:40:15.09ID:10Q9AITI
>>589
a[1]+a[2]+…+a[n]=sとおくと、
H(s) - H(a[1]) - H(a[2]) - … - H(a[n])
= ……
= Σ[1≦i<j≦m] {H(s_i, s_j) - H(a[1]_i,a[1]_j) - … - H(a[n]_i,a[n]_j)}
= Σ[1≦i<j≦m] h(b[i],b[j]),

つまりm項ベクトルの場合も、実は2項ベクトルの調和平均差の総和に過ぎない。 ←これ重要

h(x,y) = H(Σ[k=1,n] x_k, Σ[k=1,n] y_k) - H(x_1,y_1) - H(x_2,y_2) - …… - H(x_n,y_n)
≧0, (>>593)
0598132人目の素数さん垢版2016/07/04(月) 21:18:40.30ID:gsq/KsKg
>>597
a^6 + b^6 + c^6 -2(aa-bc)(bb-ca)(cc-ab) + (abc)^2
= (a^3 + b^3 + c^3 - abc)^2
≧0

ぬるぽビッチにどうやって当てはめるのだらうか?
0599132人目の素数さん垢版2016/07/04(月) 22:04:41.76ID:10Q9AITI
>>598
無理ぽビッチみたい…

(p+1)(a^4+b^4+c^4) -p(a+b+c)(a^3+b^3+c^3) +(pp-1){(ab)^2+(bc)^2+(ca)^2} +p(2-p)abc(a+b+c)≧0,

A=aa+pbc, B=bb+pca, C=cc+pab とおくと?
0601132人目の素数さん垢版2016/07/05(火) 00:32:20.90ID:vlIg6IAS
累乗平均 M の次数を r とすると
・r>1 のとき:M(a[1]) + … + M(a[n]) ≧ M(a[1] + … + a[n])
・r=1 のとき:M(a[1]) + … + M(a[n]) = M(a[1] + … + a[n])
・r<1 のとき:M(a[1]) + … + M(a[n]) ≦ M(a[1] + … + a[n])
どっかで見たことあると思ったらポリアの不等式の本に書いてありました
0603132人目の素数さん垢版2016/07/05(火) 13:24:55.90ID:vlIg6IAS
>>602
http://mathtrain.jp/crt

分数和を下から抑えるときに使うテクニック
分母にAM≧GMを適用させると与不等式とは不等号が逆になるから,分子を無理やり分母で割ってあまりの部分の分数を符号を反転させる
0604132人目の素数さん垢版2016/07/05(火) 13:35:43.21ID:vlIg6IAS
・Romania 1997 xyz=1, x, y, z>0

(x^9+y^9)/(x^6+x^3y^3+y^6) + (y^9+z^9)/(y^6+y^3z^3+z^6) + (z^9+x^3)/(z^6+z^3x^3+x^6) ≧ 2


・Lithuania 1987 x, y, z>0

x^3/(x^2+xy+y^2) + y^3/(y^2+yz+z^2) + z^3/(z^2+zx+x^2) ≧ (x+y+z)/3
0605132人目の素数さん垢版2016/07/05(火) 19:37:08.65ID:zGQUvoYm
>>604
・上
x^3 = X, y^3 = Y, z^3 = Z とおく。
(X^3 + Y^3)/(XX+XY+YY)
=(X+Y)(XX-XY+YY)/(XX+XY+YY)
=(X+Y){1 + 2(X-Y)^2/(XX+XY+YY)}/3
≧ (X+Y)/3,

・下
x^3/(xx+xy+yy)
= (2x-y)/3 + (x+y)(x-y)^2/{3(xx+xy+yy)}
≧ (2x-y)/3,
0606132人目の素数さん垢版2016/07/05(火) 21:41:03.01ID:vlIg6IAS
CRTの話題が出てきたのでその練習問題として出したんだけど
0607132人目の素数さん垢版2016/07/05(火) 22:42:14.15ID:zGQUvoYm
>>589
mについての帰納法で…

・m=2のとき
>>587 >>593

・m>2のとき
H(a)=H(a_1, a_2,…,a_(m-1),a_m)
H0(a)=H(a_1, a_2,…,a_(m-1))
s=a[1] + a[2] + …… + a[n],
とおく。
m-1について成立したとする。(帰納法の仮定)
H1 = Σ[k=1,n] H0(a[k]) ≦ H0(s),

さて
H(a) = H(a_1,a_2,・・・・・・,a_(m-1),a_m)
= H(H0(a),・・・・・・,H0(a),a_m)
= m・a_m・H0(a)/{(m-1)a_m + H0(a)}
= {a_m + (m-1)H0(a)}/m - ((m-1)/m)Σ[k=1,n] {a_m - H0(a)}^2 /{(m-1)a_m + H0(a)},
なので、
Σ[k=1,n] H(a[k]) = {s_m+ (m-1)H1}/m - ((m-1)/m)Σ[k=1,n] {a[k]_m - H0(a[k])}^2 /{(m-1)a[k]_m + H0(a[k])}
≦ {s_m + (m-1)H1}/m - ((m-1)/m)(s_m - H1)^2 /{(m-1)s_m + H1} (←コーシー)
= m・s_m・H1/{(m-1)s_m + H1}
≦ m・s_m・H0(s)/{(m-1)s_m + H0(s)}  (← H1≦H0(s))
= H(s),

>>596 は違うっぽい…
0608132人目の素数さん垢版2016/07/05(火) 22:51:44.60ID:vlIg6IAS
何かみんな難しく考えてない?

>>589
n 以下で成り立つと仮定すると
H(a[1] + … + a[n] + a[n+1])
≧H(a[1] + … + a[n]) + H(a[n+1])
≧H(a[1]) + … + H(a[n]) + H(a[n+1])
0609132人目の素数さん垢版2016/07/05(火) 23:02:59.95ID:vlIg6IAS
>>589 において,n=2 としたものが >>587
つまりベクトルの項数ではなく個数が 2 個
>>593 は n=2 で正しく証明してるから,帰納法を使うなら m ではなく n についてやらなきゃダメ
それが >>608
0611132人目の素数さん垢版2016/07/06(水) 20:08:21.77ID:714K2b8V
>>608
n=2 の場合を示せば十分だな。

やっぱり、mについての帰納法になるのかな。
H(a) + H(b)
= H(H0(a),……,H0(a),a') + H(H0(b),……,H0(b),b')
= m・a'・H0(a)/[(m-1)a' + H0(a)] + m・b'・H0(b)/[(m-1)b' + H0(b)]
= m(a'+b'){H0(a)+H0(b)}/[(m-1)(a'+b')+H0(a)+H0(b)]−{(m-1)/m}[a'・H0(b)−b'・H0(a)]^2 {H(a)/[a'・H0(a)]}{H(b)/[b'・H0(b)]}/[(m-1)(a'+b')+H0(a)+H0(b)]
≦ m(a'+b'){H0(a)+H0(b)}/[(m-1)(a'+b')+H0(a)+H0(b)]
≦ m(a'+b')H0(a+b)/{(m-1)(a'+b')+H0(a+b)}  {← H0(a)+H0(b)≦H0(a+b)}
= H(H0(a+b),……,H0(a+b), a'+b')
= H(a+b),

>>609 は意味不明…
0612132人目の素数さん垢版2016/07/07(木) 17:17:18.77ID:qeaoYgad
>>604 >>606
CRT(ブラウン管)を使うと、

(XX-XY+YY)/(XX+XY+YY)
= 1 - 2XY/(XX+XY+YY)
≧ 1 - 2XY/(3XY)
= 1/3,

x^3/(xx+xy+yy)
= x - xy(x+y)/(xx+xy+yy)
≧ x - xy(x+y)/(3xy)
= x - (x+y)/3
= (2x-y)/3,
0614132人目の素数さん垢版2016/07/07(木) 18:56:39.91ID:PHUaneap
>>613
a^6 +b^6 +c^6 +2(aa+bc)(bb+ca)(cc+ab) -3(abc)^2
= (a^3 + b^3 + c^3 + abc)^2
≧0

ぬるぽビッチの性能とやらを見せてもらおうか?
0615132人目の素数さん垢版2016/07/07(木) 20:32:24.67ID:o/iOEGvs
Let a, b, c be three positive real numbers such that a+b+c=1.
Find max of k∈R satisfies (a+b)(b+c)(c+a)≧k√(abc).
0616132人目の素数さん垢版2016/07/08(金) 04:26:06.83ID:uwPT+eQg
>>147
左辺は (x, y, z) = (1, (√5-1)/2, (√5+1)/2) 及びこの巡回置換の時に最小値 11-5√5 = -0.18 を取ります
よって不等式は成り立ちません
0617132人目の素数さん垢版2016/07/08(金) 18:04:18.17ID:+GqLshVo
>>615
let a+b+c=s, then
(a+b)(b+c)(c+a) ≧ (8/9)s(ab+bc+ca) ≧ (4s/3)^(3/2)・√(abc),

Left:
(a+b)(b+c)(c+a) - (8/9)s(ab+bc+ca)
= (1/9){(a+b+c)(ab+bc+ca) - 9abc}
= (1/18){a(b-c)^2 + b(c-a)^2 + c(a-b)^2}
≧0

Right:
(ab+bc+ca)^2 - 3s(abc)
= (ab+bc+ca)^2 - 3(ab・bc + bc・ca + ca・ab)
= (1/2){aa(b-c)^2 + bb(c-a)^2 + cc(a-b)^2}
≧0,

∴k=(4/3)^(3/2).
0618132人目の素数さん垢版2016/07/08(金) 21:28:17.08ID:uwPT+eQg
>>617
これはぬるぽビッチの性能を見せつけるための問題ですよ
0621132人目の素数さん垢版2016/07/09(土) 19:20:08.37ID:UxNa1hu5
>>615 >>618
Soko made iwaretan-ja shooganee na…

y=log(x) は上に凸

log(LHS)
= log(a+b) + log(b+c) + log(c+a)
= 3log(2) + log[(a+b)/2] + log[(b+c)/2] + log[(c+a)/2]
≧ 3log(2) + (3/2)log[(a+b+c)/3] + (1/2){log(a)+log(b)+log(c)}
= (3/2)log(4) + (3/2)log(1/3) + (1/2)log(abc)
= (3/2)log(4/3) + (1/2)log(abc)
= log(RHS),

∴k=(4/3)^(3/2).

equality: a=b=c=1/3
0623132人目の素数さん垢版2016/07/12(火) 20:37:55.08ID:wu5v/x5V
>>622

〔問題147"〕
正数X,Y,Z について次を示せ。
(X + Y + Z)(1/X + 1/Y + 1/Z - 6/G) + (-2+5√5) ≧0,
ここに、G=(XYZ)^(1/3).
0624132人目の素数さん垢版2016/07/13(水) 03:54:57.94ID:16ioO3Ne
〔問題147'''〕
k を実数とする。正の数 x, y, z が xyz=1 を満たすとき
(x^3 + y^3 + z^3) + (1/x^3 + 1/y^3 + 1/z^3) + k(x/z + y/x + z/y)
の最小値を求めよ。

実際には最小値を与える x, y, z の条件を求めることになるけど…
k の値にもよるけどこの不等式自体あまり綺麗じゃないね
0625◆2VB8wsVUoo 垢版2016/07/13(水) 03:56:24.51ID:K/H8KF/H
>>437
私は『父親から被害に遭った』ので、だから彼とは事情が違います。それ
にあんな偉い数学者と一緒にしないで下さいまし。彼は歴史に残る偉大な
数学者であり、私みたいな小魚とは違うので。

彼みたいな有能な人でも「人間関係が邪魔になる」んだから、私みたいな
小魚はもっと大変ですわ。かなり有能でなければ、大学に勤務してたらば
『自分が擦り減るだけ』なので。かつて邪魔する芳雄を倒すだけでも大変
な苦労をしたので。

0626◆2VB8wsVUoo 垢版2016/07/13(水) 03:59:24.61ID:K/H8KF/H
日本人の躾けは『大人の都合』、その目的は威厳に屈服させる為:
ある父親:クマが出没する山林に息子を放置、しかも嘘を吐いて保身。
別の父親:勉強の邪魔をして進路を妨害し、学歴を砕く。出世を強要。
ソレでも「親の行為は子供の為」という傲慢な常識を振り回す世間、しかも
「親を尊敬して大切に扱え」という無根拠な思想を押し付ける儒教文化。

お父さん、お母さんを大切にしましょう!!!ソレが世間体というモノ!

ケケケ¥

政治家も、お教授も、権力を振り回すのが大好きな低能人種:
ある男:ボクは都民の為に湯河原で休んでるんだ。知事が信じられんのかっ!
別の男:オレは哲也の為に指導してやってるんだ。父親が信じられんのかっ!
上から目線で強弁すれば、自分の言い分は何でも通る国があるらしい…

ああ、素晴らしき日本文化よ。キミ達も国会議員を見習い給え。何せ多数決で選
ばれた『皆の代表』なので。だからある男も別の男もエラいんだよォ〜〜〜んw

コココ¥

終わり良ければ全てヨシ。途中経過はどうでもヨシ。
大学:学生の知能なんてどうでもヨシ。カネが儲かる教室を巧みに運営シロ。
狸研:研究の詳細なんてどうでもヨシ。世間が驚く大論文を外国に発表シロ。
芳雄:学問の中身なんてどうでもヨシ。安易に教授になれる分野を専攻シロ。

学問なんて所詮は出世の道具。周囲に秀才っぽく見せ掛けられたらソレでヨシ。
社会的に高い地位、そして豪華で贅沢な暮らし。世間が羨む大学教授のポスト。
ソレさえ手に入れば学問そのものなんて洋梨よォ〜〜〜ん。

よよよ、よ〜〜〜しお。そやしノ〜ベル賞が欲しいよォ〜〜〜んんんwww

シシシ¥
0628132人目の素数さん垢版2016/07/13(水) 19:34:08.54ID:2uLWSWwd
〔問題〕
a^4 +b^4 +c^4 - p(aaab+bbbc+ccca) - p'(abbb+bccc+caaa) + {(pp+pp'+p'p')/3 -1}(aabb+bbcc+ccaa) + {p+p'-(pp+pp'+p'p')/3}abc(a+b+c)≧0,
を示せ。

x=aa+p'bc-cc-pbc, y=bb+p'ca-aa-pbc, z=cc+p'ab-bb-pca とおくと…

(参考)
安藤:「不等式」数学書房 (2012) p.59-61 §2.3.2 定理2.3.3(1)
0629132人目の素数さん垢版2016/07/14(木) 00:45:31.55ID:JfFkOvJ6
以前に見たような気がするけど、過去ログ調べても出てこなかった。
もっときつい評価(≦)を証明してくれような気がするんだけど、記憶が曖昧模糊ビッチ。

a, b, c >0のとき、5/3 < (3a+b)/(3b+a) + (3b+c)/(3c+b) + (3c+a)/(3a+c) < 7
0630132人目の素数さん垢版2016/07/14(木) 01:12:06.98ID:EvL3WsYI
>>442 じゃなくて?
0631◆2VB8wsVUoo 垢版2016/07/14(木) 01:16:04.23ID:W/q5qfz4
日本人の躾けは『大人の都合』、その目的は威厳に屈服させる為:
ある父親:クマが出没する山林に息子を放置、しかも嘘を吐いて保身。
別の父親:勉強の邪魔をして進路を妨害し、学歴を砕く。出世を強要。
ソレでも「親の行為は子供の為」という傲慢な常識を振り回す世間、しかも
「親を尊敬して大切に扱え」という無根拠な思想を押し付ける儒教文化。

お父さん、お母さんを大切にしましょう!!!ソレが世間体というモノ!

ケケケ¥

政治家も、お教授も、権力を振り回すのが大好きな低能人種:
ある男:ボクは都民の為に湯河原で休んでるんだ。知事が信じられんのかっ!
別の男:オレは哲也の為に指導してやってるんだ。父親が信じられんのかっ!
上から目線で強弁すれば、自分の言い分は何でも通る国があるらしい…

ああ、素晴らしき日本文化よ。キミ達も国会議員を見習い給え。何せ多数決で選
ばれた『皆の代表』なので。だからある男も別の男もエラいんだよォ〜〜〜んw

コココ¥

終わり良ければ全てヨシ。途中経過はどうでもヨシ。
大学:学生の知能なんてどうでもヨシ。カネが儲かる教室を巧みに運営シロ。
狸研:研究の詳細なんてどうでもヨシ。世間が驚く大論文を外国に発表シロ。
芳雄:学問の中身なんてどうでもヨシ。安易に教授になれる分野を専攻シロ。

学問なんて所詮は出世の道具。周囲に秀才っぽく見せ掛けられたらソレでヨシ。
社会的に高い地位、そして豪華で贅沢な暮らし。世間が羨む大学教授のポスト。
ソレさえ手に入れば学問そのものなんて洋梨よォ〜〜〜ん。

よよよ、よ〜〜〜しお。そやしノ〜ベル賞が欲しいよォ〜〜〜んんんwww

シシシ¥
0633132人目の素数さん垢版2016/07/14(木) 14:32:01.86ID:JfFkOvJ6
実数x、yに対して、(x^2+12xy+9y^2)^2 ≧ 4y(2x+3y)^3

( ゚∀゚) プゥ
ノヽノ) =3'A`)ノ ヒャー
  くく へヘノ
0634132人目の素数さん垢版2016/07/14(木) 15:24:01.43ID:EvL3WsYI
>>633
例えば (x, y) = (1, 1) だと?
0635132人目の素数さん垢版2016/07/14(木) 15:28:55.94ID:JfFkOvJ6
>>633
すまん、忘れてくれ…


   シュボッ
       ., ∧_∧ 
      []() (・ω・` )      l二ヽ
       □と    ) ̄⊃     ) )
      ⊂ (_(_つ   ̄⊃  / ̄ ̄ ̄ヽ
       ⊂_      ._⊃   | (\/) |
         ⊂__⊃.      |  > <  |
                     | (/\). |
                     ヽ___/
0636◆2VB8wsVUoo 垢版2016/07/14(木) 15:33:42.95ID:W/q5qfz4


>232 :132人目の素数さん:2016/07/01(金) 13:34:39.39 ID:zLVRVGit
> >>217 たんなる京大とプロ数学者じゃ全然話が違うだろ
> 同列に書くあたり、ほんと、どうしようもないクソ京大コンプだな、じじい
>
>246 名前:132人目の素数さん :2016/07/01(金) 18:07:16.21 ID:/KsaK/zz
> >>217
> >解ってると思うが、悪質なネット民は絶対に許さんのでナ。低能は低能だ
> けで遊べや。ほんでや、頭の悪いアホが京大とかプロの数学者とか、そう
> いうモンを話題にすんなや。解りもセンくせにいい加減な事を言うてや、
> ほんでプロに迷惑なんて掛けるなや。許さんのでナ。
>
>
> 本当のエリートは有象無象の言うことなど、ハナから眼中にない。
> アンタが有象無象の言うことが癇に障ってしかたがないのは、アンタ自身が
> (アンタがヘドがでるほど嫌悪する)有象無象の一人に過ぎない証拠。
>

>> 217 :¥ ◆2VB8wsVUoo :2016/07/01(金) 11:07:44.51 ID:Hb6rl5wG
>> 解ってると思うが、悪質なネット民は絶対に許さんのでナ。低能は低能だ
>> けで遊べや。ほんでや、頭の悪いアホが京大とかプロの数学者とか、そう
>> いうモンを話題にすんなや。解りもセンくせにいい加減な事を言うてや、
>> ほんでプロに迷惑なんて掛けるなや。許さんのでナ。
>>
>> そもそも他人のプライバシーなんかに興味を持つんじゃねェんだよ。こう
>> いう匿名無責任糞板はケシカラン連中が跋扈してるやろ。そやし壊滅する
>> まで焼くさかいナ。エエな。馬鹿は馬鹿だけで閉じて遊べや。京大を話題
>> になんてスナ。焼き払ってやる。アホは絶対に許さんのでナ。糞野郎共め。
>>
>> ¥
>>
0637132人目の素数さん垢版2016/07/14(木) 15:53:11.16ID:EvL3WsYI
a[1], …, a[n]; b[1], …, b[n] を正の実数とし,M[1], m[1] をそれぞれ a[1], …, a[n] の最大値,最小値,M[2], m[2] をそれぞれ b[1], …, b[n] の最大値,最小値とする。
  (a[1]^2+…+a[n]^2)(b[1]^2+…+b[n]^2) / (a[1]b[1]+…+a[n]b[n])^2
の最大値および等号成立条件を求めよ。

【THE USER OLYMPIAD PROBLEM BOOK, problem 299, pp.70-71.】
0638◆2VB8wsVUoo 垢版2016/07/14(木) 16:13:56.03ID:W/q5qfz4


>232 :132人目の素数さん:2016/07/01(金) 13:34:39.39 ID:zLVRVGit
> >>217 たんなる京大とプロ数学者じゃ全然話が違うだろ
> 同列に書くあたり、ほんと、どうしようもないクソ京大コンプだな、じじい
>
>246 名前:132人目の素数さん :2016/07/01(金) 18:07:16.21 ID:/KsaK/zz
> >>217
> >解ってると思うが、悪質なネット民は絶対に許さんのでナ。低能は低能だ
> けで遊べや。ほんでや、頭の悪いアホが京大とかプロの数学者とか、そう
> いうモンを話題にすんなや。解りもセンくせにいい加減な事を言うてや、
> ほんでプロに迷惑なんて掛けるなや。許さんのでナ。
>
>
> 本当のエリートは有象無象の言うことなど、ハナから眼中にない。
> アンタが有象無象の言うことが癇に障ってしかたがないのは、アンタ自身が
> (アンタがヘドがでるほど嫌悪する)有象無象の一人に過ぎない証拠。
>

>> 217 :¥ ◆2VB8wsVUoo :2016/07/01(金) 11:07:44.51 ID:Hb6rl5wG
>> 解ってると思うが、悪質なネット民は絶対に許さんのでナ。低能は低能だ
>> けで遊べや。ほんでや、頭の悪いアホが京大とかプロの数学者とか、そう
>> いうモンを話題にすんなや。解りもセンくせにいい加減な事を言うてや、
>> ほんでプロに迷惑なんて掛けるなや。許さんのでナ。
>>
>> そもそも他人のプライバシーなんかに興味を持つんじゃねェんだよ。こう
>> いう匿名無責任糞板はケシカラン連中が跋扈してるやろ。そやし壊滅する
>> まで焼くさかいナ。エエな。馬鹿は馬鹿だけで閉じて遊べや。京大を話題
>> になんてスナ。焼き払ってやる。アホは絶対に許さんのでナ。糞野郎共め。
>>
>> ¥
>>
0640132人目の素数さん垢版2016/07/15(金) 16:38:30.80ID:402uibiN
>>233
> a、b、c、x、y、z ∈R が、
>  (a-1)^2 + (b-2)^2 + (c-3)^2 = 1
>   x^2+y^2+z^2=1
> をみたすとき、ax+by+czのとりうる値の範囲
>
> CS不等式を2回使ったけど、他の解法ありますか?


( ゚∀゚) ワクワク ワクワク …
0641132人目の素数さん垢版2016/07/15(金) 19:04:34.16ID:qKj/pzjF
>>640
OP=(a,b,c) OX=(x,y,z) とおくと OX=1 だから、
|ax+by+cz| = OP・OX |cosφ| ≦ OP,
P=(a,b,c) は中心 C=(1,2,3) 半径1の球面上の点だから、△不等式より
OP ≦ OC +1 = √14 +1,
0642132人目の素数さん垢版2016/07/15(金) 20:27:28.25ID:qKj/pzjF
>>292 >>331

(A/G)^3 = {(a+b+c)/3}^3 /(abc) = s^3/(27u),
(G/H)^3 = abc{(1/a+1/b+1/c)/3}^3 = t^3/(27uu),
(A/H) = (a+b+c)(1/a+1/b+1/c)/9 = st/(9u),
ここに、s=a+b+c, t=ab+bc+ca, u=abc.

∴ (右辺)−(左辺)
= (3/4)(1 + st/9u)^2 - s^3/(27u) - t^3/(27uu) -1
= {(9u+st)^2 -4s^3・u -4t^3}/(108uu) -1
= 刧/(108uu)
≧0,
ここに = (a-b)(b-c)(c-a), …差積
0643132人目の素数さん垢版2016/07/16(土) 21:55:48.73ID:PABwwLAI
>>231
aa=xy/2, bb=2yz/3, cc=3zx,
とか置くと、
x=2ca/(3b), y=3ab/c, z=bc/(2a), xyz=abc,
よって
(与式) = abc/{6(aa+1)(bb+1)(cc+1)},
(aa+1)-2a =(a-1)^2 ≧0.


>>628
(左辺) = (1/6){(x-y)^2 + (y-z)^2 + (z-x)^2}
≧0,


>>637
ラグランジュさんの恒等式
(a[1]^2+…+a[n]^2)(b[1]^2+…+b[n]^2)-(a[1]b[1]+…+a[n]b[n])^2=Σ[1≦i<j≦n](a[i]b[j]-b[i]a[j])^2
が最大になるのは…
0647132人目の素数さん垢版2016/07/17(日) 23:42:09.72ID:0qSKZ1Xz
>>624

(x^3 + y^3 + z^3) + (1/x^3 + 1/y^3 + 1/z^3) + k(x/z + y/x + z/y)
= {x^3 + 1/z^3 + (-k/3)^3 + kx/z}
+ {y^3 + 1/x^3 + (-k/3)^3 + ky/x}
+ {z^3 + 1/y^3 + (-k/3)^3 + kz/y}
+ kkk/9
= (x +1/z -k/3){ … } + (y +1/x -k/3){ … } + (z +1/y -k/3){ … } + kkk/9
≧ kkk/9,

綺麗でもねぇし、(k≠-3では)最小値も出ねぇし…
0648132人目の素数さん垢版2016/07/20(水) 21:56:45.67ID:tgs7YrRF
〔演習問題1.96〕
x,y,zを非負実数とするとき、次を示せ。
x^3 +y^3 +z^3 -3xyz ≧ k|(x-y)(y-z)(z-x)|,

(a) k = 4
(b) k = √(9+6√3) = 4.403669475

ルーマニア 2007年(改)
佐藤(訳)§1.6 演習問題1.96
0649132人目の素数さん垢版2016/07/20(水) 22:37:49.58ID:tgs7YrRF
>>648
(左辺) = (x+y+z)(xx+yy+zz-xy-yz-zx),

yはxとzの中間にあるとしてよい。
xx+yy+zz-xy-yz-zx = (x-y)^2 + |x-y||y-z| + (y-z)^2

(a)の方は
x+y+z ≧ |x-y| + |y-z| + min{|x-y|, |y-z|}

(左辺) ≧ (|x-y|+|y-z|)^3 = |x-z|^3 ≧ 4|(x-y)(y-z)(z-x)|,
0650132人目の素数さん垢版2016/07/21(木) 04:37:28.80ID:pItodR5J
>>648
(b)
LHS - RHS の最小値は (x, y, z) = (0, c*(3+sqrt(6(sqrt(3)-9)), c*(3-sqrt(6(sqrt(3)-9))) (c は任意の実数) およびこの任意の置換で起こるってとこだね
0651132人目の素数さん垢版2016/07/21(木) 04:39:33.36ID:pItodR5J
>>648
(b) LHS- RHS の最小値は
  (x, y, z) = (0, c*(3+sqrt(6sqrt(3)-9)), c*(3-sqrt(6sqrt(3)-9))) (c は任意の実数)
およびこの任意の置換で起こる
0652132人目の素数さん垢版2016/07/21(木) 15:41:03.72ID:pItodR5J
>>651
c は実数ではなく任意の非負実数っていうのと,最小値は x = y = z でも起こることを付け足しておきます

【問題648'】
非負実数 x, y, z に対し,次の最小値を求めよ
  (x^3+y^3+z^3-3xyz) - abs(x*(x-y)*(x-z))
0654132人目の素数さん垢版2016/07/21(木) 16:10:58.48ID:pItodR5J
解いてはないけどあってると思う

【問題648''】
非負実数 x, y, z に対し,次の不等式を示せ
  ・x^3+y^3+z^3+xyz ≧ 1/2 (x+y)(y+z)(z+x)
  ・x^3+y^3+z^3+1/4 (x+y)(y+z)(z+x) ≧ xyz
0655132人目の素数さん垢版2016/07/21(木) 16:12:09.88ID:pItodR5J
>>653
絶対値です
texやmaximaなど,色々なところで使えます
0656648-649垢版2016/07/21(木) 16:49:03.75ID:NpEdeMjc
>>652
正解です! 
(x,y,z) = (0, C*(k+√3), C*(k-√3))  C≧0
でもいい…

>>654
x^3 +y^3 +z^3 -(x+y)(y+z)(z+x) +5xyz = x(x-y)(x-z) + y(y-z)(y-x) + z(z-x)(z-y) = F1(x,y,z) ≧0,
(Schur) と
(x+y)(y+z)(z+x) - 8xyz = x(y-z)^2 + y(z-x)^2 + z(x-y)^2 ≧0,
を使えば出そう…
0657132人目の素数さん垢版2016/07/23(土) 22:53:41.28ID:vBFJLUiR
もう変形問題は十分かな?

【問題648'''】
λを実数とする。任意の非負実数 x, y, z に対し次の不等式が成り立つ実定数 k が存在するような λ を求め,その時の k の最大値を求めよ。
  x^3 +y^3 +z^3 +λxyz ≧ k|(x-y)(y-z)(z-x)|,
0658132人目の素数さん垢版2016/07/23(土) 23:35:47.80ID:vBFJLUiR
ところでみんなはどんな感じに不等式コレクションしてるの?
0659132人目の素数さん垢版2016/07/24(日) 00:49:56.31ID:KzbjbpQs
とりあえずTeXで問題と証明をまとめて、不等式フォルダにブチ込む。
時間のあるときに覗いてハァハァしながら、適当に分類する。
昔は出典(入試問題なら出題年と大学名、webならURL)を記録していなかったことを後悔。
一度HDDが壊れて失われたことがあるので、今は殆ど残っていない。
最近は時間がないので、TeXでまとめずに、urlだけメモって終わり。
0660132人目の素数さん垢版2016/07/24(日) 14:35:35.94ID:fbilIAGJ
>>659
そのフォルダ見たいぜ!

任意の実数 x, y, z に対し,次の不等式を示せ。
  x^4 + y^4 + z^4 + 2xyz(x + y + z) ≧ x^3y + y^3z + z^3x
【VASILE CIRTOAJE】
0671132人目の素数さん垢版2016/07/24(日) 21:49:10.16ID:vPY226Dk
>>660

>>628 で p=1、p'=0 とすると、
X = xx-zz-pxy,
Y = yy-xx-pyz,
Z = zz-yy-pzx,
により
(左辺)−(右辺)={(X+Y)^2+(Y+Z)^2+(Z+X)^2}/2 ≧0,

等号成立は
(x,y,z) = (1, (0.8019377358), 1/(-2.2469796037)) とその rotation.
t^3 +2tt -t-1=0 の根。


【問題660'】
任意の実数x,y,zに対し
x^4 + y^4 + z^4 + (√3 -1)xyz(x+y+z) ≧ (√3)(x^3・y + y^3・z + z^3・x),
は成り立つか?
0682132人目の素数さん垢版2016/07/25(月) 19:42:43.05ID:DgzBjBOv
>>628
(1/6)(p-p')^2・(ab+bc+ca)^2 ≧0 を足すと…

〔系〕
a^4 +b^4 +c^4 - p(aaab+bbbc+ccca) - p'(abbb+bccc+caaa) + {(pp+p'p')/2 -1}(aabb+bbcc+ccaa) + (p+p'-pp')abc(a+b+c)≧0,
0683132人目の素数さん垢版2016/07/25(月) 20:02:06.87ID:DgzBjBOv
>>628 >>682

x = aa-cc-pab+p'bc,
y = bb-aa-pbc+p'ca,
z = cc-bb-pca+p'ab,
とおくと、
(628の左辺) = {(x-y)^2 + (y-z)^2 + (z-x)^2}/6 ≧0,
(682の左辺) = (xx+yy+zz)/2 ≧0,
0684132人目の素数さん垢版2016/07/27(水) 22:29:50.52ID:iPJXlDFo
f(x) をn次多項式
M = max |f(x)| (1≦x≦1)
m = max |f’(x)| (1≦x≦1)
とおくとき、m’≦(n^2)・M
0685132人目の素数さん垢版2016/07/28(木) 04:53:00.74ID:yd5KkHjM
そういや昔、日本の高校教師がAM-GM inequalityの新証明を発見したよ(ドヤ顔)ってニュースがあって、
数セミにも記事が載ってたけど、どの辺が新証明なのか分からなかった。サルにも分かるように説明キボンヌ!
0686132人目の素数さん垢版2016/07/28(木) 05:08:15.85ID:yd5KkHjM
>>685
不等式への招待 第5章

536 名前:132人目の素数さん[sage] 投稿日:2011/08/25(木) 16:54:26.26
そういや3年位前に、高校の先生が相加相乗平均の新証明の記事があったけど、いまさらながら、その論文のリンクを貼っておく
http://www.emis.de/journals/JIPAM/images/080_08_JIPAM/080_08.pdf
並べ替え不等式を使うのか…

541
その方法と 全 く 同 じ 方 法 で、色々な不等式(もちろん相加相乗平均も)を証明した記事が、数学セミナーに掲載されている。

数学セミナー 2004.2
ttp://www.nippyo.co.jp/magazine/4352.html
>対称性を有する不等式の統一的証明について 仁平政一 52
↑この記事。2004年だから、例の高校の先生より早い。

545
記事名をキーワードにググってみたら、数研通信とかいうサイトに まるごと載ってるじゃねーか(^o^)

数研通信 47号2003年8月
不等式の証明の統一的方法(仁平政一)
ttp://www.chart.co.jp/subject/sugaku/suken_tsushin/47/47-5.pdf

>541と若干タイトルが違うが、著者は同じ。で、こっちの方がさらに年月が古く、2003年8月となっている。
>541のやつは、この記事の加筆修正なのかもしれん(俺の手元に数セミが無いので、確認できない^o^)。

546
数蝉の年2回のNOTEは、コピーしてファイルしてるので見たけど、数検通信の記事から抜粋したものですな
で、この方法は >>2 参考文献[1] P.25の方法と同じな希ガス…

547
ということは、並べ替え不等式を使う方法はずっと昔から知られていたと。

548
所謂, Rearrangememt Inequalityですな。

583
 x_(n-1) ≦ G ≦ x_n,
を仮定して
 x_(n-1) + x_n - {x_(n-1)・x_n /G + G} = (x_n - G){G - x_(n-1)}/G ≧ 0,
 x_(n-1) + x_n ≧ {x_(n-1)・x_n /G} + G,
を導いています。

584
つまり既出の証明でも専門誌に発表できるということですね

585
対称性に注目って不等式考える上では突飛なアイデアじゃないよね
ってか定跡やん。これを「新証明」と主張することに不安は感じなかったのだろうか。
0687132人目の素数さん垢版2016/07/29(金) 06:20:06.49ID:qniiitz9
論文って、民間人が投稿できるの?数学者かつ、その雑誌の会員になってないとダメなんじゃないん?
しかも掲載してもらうのに金がかかるのでは?
0688132人目の素数さん垢版2016/07/30(土) 16:41:14.11ID:Dzi09bPm
>>682 に (1/2)(p-p')^2・(ab+bc+ca)^2 = (1/2)(x+y+z)^2 ≧0 を足すと…

〔系〕
a^4 +b^4 +c^4 -p(aaab+bbbc+ccca) -p'(abbb+bccc+caaa) +(pp-pp'+p'p'-1)(aabb+bbcc+ccaa) +(p+p'+pp-3pp'+p'p')abc(a+b+c)
= {(x+y)^2 +(y+z)^2 +(z+x)^2}/2
≧0,

p=1、p'=0 とおけば >>660


(略証)
(x+y)^2 + (y+z)^2 + (z+x)^2 = x^2 + y^2 + z^2 + (x+y+z)^2,
3(x^2 + y^2 + z^2) = (x-y)^2 + (y-z)^2 + (z-x)^2 + (x+y+z)^2,
より
0689◆2VB8wsVUoo 垢版2016/08/01(月) 00:28:09.98ID:qfoqfHkv
日本人ってホンマに『人を舐めてる』よね。こういう糞みたいな奴ばっか
りだから国家も腐るし、そして学問もダメになる。だからとにかく馬鹿板
は焼きます。こういう場所でカキコする低能が苦悩する様に。



>331 名前 :132人目の素数さん:2016/07/31(日) 12:42:35.54 ID:eoIQzfwB
>  反論できないってことは読んでないんだなw
>  なのに数学談義大好きw馬鹿の考えてることはよくわからんw
>
0690◆2VB8wsVUoo 垢版2016/08/01(月) 16:15:46.52ID:qfoqfHkv
日本人ってホンマに『人を舐めてる』よね。こういう糞みたいな奴ばっか
りだから国家も腐るし、そして学問もダメになる。だからとにかく馬鹿板
は焼きます。こういう場所でカキコする低能が苦悩する様に。



>331 名前 :132人目の素数さん:2016/07/31(日) 12:42:35.54 ID:eoIQzfwB
>  反論できないってことは読んでないんだなw
>  なのに数学談義大好きw馬鹿の考えてることはよくわからんw
>
0691◆2VB8wsVUoo 垢版2016/08/01(月) 16:21:17.67ID:qfoqfHkv
日本人ってホンマに『人を舐めてる』よね。こういう糞みたいな奴ばっか
りだから国家も腐るし、そして学問もダメになる。だからとにかく馬鹿板
は焼きます。こういう場所でカキコする低能が苦悩する様に。



>331 名前 :132人目の素数さん:2016/07/31(日) 12:42:35.54 ID:eoIQzfwB
>  反論できないってことは読んでないんだなw
>  なのに数学談義大好きw馬鹿の考えてることはよくわからんw
>
0692◆2VB8wsVUoo 垢版2016/08/01(月) 16:21:41.72ID:qfoqfHkv
日本人ってホンマに『人を舐めてる』よね。こういう糞みたいな奴ばっか
りだから国家も腐るし、そして学問もダメになる。だからとにかく馬鹿板
は焼きます。こういう場所でカキコする低能が苦悩する様に。



>331 名前 :132人目の素数さん:2016/07/31(日) 12:42:35.54 ID:eoIQzfwB
>  反論できないってことは読んでないんだなw
>  なのに数学談義大好きw馬鹿の考えてることはよくわからんw
>
0693132人目の素数さん垢版2016/08/01(月) 18:18:26.53ID:CDqeLcE8
任意の実数 x, y, z に対し,次の不等式を示せ。
  (x^2 + y^2 + z^2)^2 ≧ 3(xxxy + yyyz + zzzx)
また,等号成立は(厳密に)いつか。
【VASILE CIRTOAJE】
0695◆2VB8wsVUoo 垢版2016/08/02(火) 11:58:10.59ID:PwIO2J7h


>1 :名無しさん :2006/04/30(日) 01:41:01 ID:KPnB.CH2
> 迷惑かしらん
>
>5392 :kmath1107★ :2016/07/31(日) 11:53:29 ID:???
> 人への念の盗み見による介入を阻め。
>
>5393 :kmath1107★ :2016/07/31(日) 11:58:25 ID:???
> 人への念の盗み見による介入を阻め。
>
>5394 :¥ ◆2VB8wsVUoo :2016/07/31(日) 12:06:23 ID:???
> ¥
>
>5395 :kmath1107★ :2016/07/31(日) 13:24:11 ID:???
> 人への念の盗み見による介入を阻め。
>
>5396 :¥ ◆2VB8wsVUoo :2016/07/31(日) 17:23:53 ID:???
> ¥
>
>5397 :kmath1107★ :2016/08/01(月) 15:59:13 ID:???
> 人への念の盗み見による介入を阻め。
>
>5398 :¥ ◆2VB8wsVUoo :2016/08/01(月) 16:06:01 ID:???
> ¥
>
0696◆2VB8wsVUoo 垢版2016/08/02(火) 12:50:30.83ID:PwIO2J7h
日本人ってホンマに『人を舐めてる』よね。こういう糞みたいな奴ばっか
りだから国家も腐るし、そして学問もダメになる。だからとにかく馬鹿板
は焼きます。こういう場所でカキコする低能が苦悩する様に。



>331 :132人目の素数さん:2016/07/31(日) 12:42:35.54 ID:eoIQzfwB
>  反論できないってことは読んでないんだなw
>  なのに数学談義大好きw馬鹿の考えてることはよくわからんw
>
0699132人目の素数さん垢版2016/08/06(土) 09:35:04.11ID:iu7LMiQH
a、b、c > 0 のとき、
(a + bc/a)*(b + ca/b)*(c + ab/c) ≧ a(b-c)^2 + b(c-a)^2 + c(a-b)^2 + 8abc

     ///////
    ///////____________
    ///////  ̄ ̄ ̄ ̄ ̄ ̄ ̄ ̄ ̄| ̄ ̄
   ///////      ___    (~) チリンチリン
   ///////     /  ≧ \  ノ,,
  ///////     |::::: (● (● |    
  ///////      ヽ::::... .ワ.....ノ    日本の夏
 ///////      (つ へへ つ      不等式の夏
///////   //△ ヽλ  ) ) 旦
//////  l ̄ ̄ ̄ ̄ ̄ ̄ ̄ ̄ ̄ ̄ ̄l
/////    ̄| .| ̄ ̄ ̄ ̄ ̄ ̄ ̄| .| ̄
////     ^^^          ^^^
0700132人目の素数さん垢版2016/08/06(土) 22:49:34.50ID:S/4UAYpT
xyz=1を満たす、x,y,zに対し、
s=(x/y,y/z,z/x,y/x,z/y,x/z), t=(z,x,y,1/z,1/x,1/y) を考え、|s|^2*|t|^2≧(s,t)^2 より
(x/y)^2+(y/x)^2+(y/z)^2+(z/y)^2+(z/x)^2+(x/z)^2 ≧ x^2 + 1/x^2 + y^2 + 1/y^2 +z^2 + 1/z^2
が得られる。x^2=a/b,y^2=b/c,z^2=c/a と置くと、
a^2/(bc) + bc/a^2 + b^2/(ca) + ca/b^2 + c^2/(ab) + ab/c^2 ≧ a/b + b/a + b/c + c/b + c/a + a/c
(1+bc/a^2)(1+ca/b^2)(1+ab/c^2) - 2
≧ (a^2+b^2)/(ab) + (b^2+c^2)/(bc) + (c^2+a^2)/(ca) = (a-b)^2/(ab) + (b-c)^2/(bc) + (c-a)^2/(ca) + 6
となり、以下、>>699の式が得られる。
0701132人目の素数さん垢版2016/08/07(日) 15:53:44.74ID:2i9EL15m
>>699
σ=a+b+c,
τ=ab+bc+ca,
υ=abc,
とおくと

(左辺)
=(a+bc/a)(b+ca/b)(c+ab/c)
=(a^3+b^3+c^3)+{(ab)^3+(bc)^3+(ca)^3}/(abc)+2abc
=σ^3+τ^3/υ−6στ+8υ,

(右辺)
=3{a(b-c)^2+b(c-a)^2+c(a-b)^2}+8abc
=3(στ-9υ)+8υ,

(左辺)−(右辺)=(σ^3+τ^3/υ+27υ)−9στ
≧0,    (←相加・相乗平均)

ぬるぽ
0702132人目の素数さん垢版2016/08/09(火) 13:17:59.74ID:NFl45Mcl
>>699を改良…

a,b,c,μ>0 のとき、
(μa+bc/a)(μb+ca/b)(μc+ab/c)−(μ+1)^3・abc ≧ K{a(b-c)^2+b(c-a)^2+c(a-b)^2},
が成立つような最大の正数Kは
K=μ(μ+√μ+1),
等号成立は(1,1,1)または(1,1,√μ)の入替え。
0703132人目の素数さん垢版2016/08/09(火) 16:21:29.05ID:00WyQw5W
>>700-701
すんばらしい!私も自分で証明できれば楽しいんだけどね…

>>702
キタ━━━┌(_Д_┌ )┐━━━!!

どうやって拡張するんだ? 頭ン中どうなってるんだろ?
0704132人目の素数さん垢版2016/08/09(火) 21:59:41.16ID:4cBgrI4L
これって新手の荒らし?
0705132人目の素数さん垢版2016/08/09(火) 22:00:48.01ID:I4hvVtKK
ここではキモAAは定番
0708◆2VB8wsVUoo 垢版2016/08/10(水) 00:18:02.71ID:1YZWefPk


>1 :名無しさん :2006/04/30(日) 01:41:01 ID:KPnB.CH2
> 迷惑かしらん
>
>5470 :¥ ◆2VB8wsVUoo :2016/08/06(土) 17:43:47 ID:???
> ¥
>
>5471 :kmath1107★ :2016/08/07(日) 06:25:49 ID:???
> 人への念の盗み見による介入を阻め。
>
>5472 :¥ ◆2VB8wsVUoo :2016/08/07(日) 08:32:51 ID:???
> ¥
>
>5473 :kmath1107★ :2016/08/07(日) 17:43:49 ID:???
> 人への念の盗み見による介入を阻め。
>
>5474 :¥ ◆2VB8wsVUoo :2016/08/07(日) 17:54:48 ID:???
> ¥
>
>5475 :名無しさん :2016/08/08(月) 04:30:59 ID:C9rjCaNs
> 人への念の盗み見による介入を阻むことができれば、多くの人に明るい未来が来る?
>
>5476 :kmath1107★ :2016/08/08(月) 10:05:45 ID:???
> 人への念の盗み見による介入を阻め。
>
> Re:>>5475 人への念の盗み見による介入が無くなれば世の不和が無くなるだろう.
>
0720◆2VB8wsVUoo 垢版2016/08/10(水) 13:05:01.06ID:1YZWefPk


>1 :名無しさん :2006/04/30(日) 01:41:01 ID:KPnB.CH2
> 迷惑かしらん
>
>5470 :¥ ◆2VB8wsVUoo :2016/08/06(土) 17:43:47 ID:???
> ¥
>
>5471 :kmath1107★ :2016/08/07(日) 06:25:49 ID:???
> 人への念の盗み見による介入を阻め。
>
>5472 :¥ ◆2VB8wsVUoo :2016/08/07(日) 08:32:51 ID:???
> ¥
>
>5473 :kmath1107★ :2016/08/07(日) 17:43:49 ID:???
> 人への念の盗み見による介入を阻め。
>
>5474 :¥ ◆2VB8wsVUoo :2016/08/07(日) 17:54:48 ID:???
> ¥
>
>5475 :名無しさん :2016/08/08(月) 04:30:59 ID:C9rjCaNs
> 人への念の盗み見による介入を阻むことができれば、多くの人に明るい未来が来る?
>
>5476 :kmath1107★ :2016/08/08(月) 10:05:45 ID:???
> 人への念の盗み見による介入を阻め。
>
> Re:>>5475 人への念の盗み見による介入が無くなれば世の不和が無くなるだろう.
>
0735132人目の素数さん垢版2016/08/11(木) 16:57:08.62ID:ejgalUa6
>>701

(左辺)−(右辺)
=(σ^3+τ^3/υ+27υ)−9στ
=(1/a){(a-b)(a-c)}^2+(1/b){(b-c)(b-a)}^2+(1/c){(c-a)(c-b)}^2
≧0,
でもいい…
0738132人目の素数さん垢版2016/08/17(水) 05:53:18.97ID:r0IaSjg0
複素数 a、b、c に対して、|sqrt(a^2 + b^2 + c^2)| ≦ max{ |a|+|b|、|b|+|c|、|c|+|a| } を示せ。

                    /´`ー─:;:;:;:;:;:/
                   /`ンー-、:;;:;:;::;:|    ト、      ,イ|
                  ,イ/  q q`ヾh    | LY´ ̄ ̄`7/ !
                      ,jイリ' rェzjゞ _,,,,jカ}    | | ``ー‐'"´; |  麻呂のスタンド
                 リ^j゙',  "´V'''''~jミi!    l |        ', |   inequalityは
                 トレ' ,'  ‐-、,, /ソ'     V       _} !   スデに不等式を
                 ヾト、   `` ,゙/       {「::i!;ヽ ,ィ;;i!:7 |    証明している…
                 /j  ヾ:;;;;;;:.:,イ´        l ゞ=''゙; ;ゞ='',' |
                , イ〈    .:;;;;;:.:/     _ _」 ',     ,' i!
               ,イ  ヽ ー-─/´ト、  ,r'´子タ ト、 ‐=- /:.!L,.-──- 、
         _,.-‐''´ \ヽー -- 、_/ `ヽ/。 。 。  _」;;:ヽ __,ィ ,r'´  ,r''"´ ̄`フ\
     ,r─''"´       `──‖‖─''´ゝノ   ,rュ_」7,イj!;:;:;:;:;::.:.:,ノ, イ7′  , '´/7rェヽ
    / i;;;;;;;|               / ̄,/q /ハ レ'ヾヾ\;;:;:.:.:: .: . ./´ー==イ、, 弋夲/i
    ! |;;;;;;|              /  ,イl   L_」l  レ'´     _ノ二ニヽ  ヾヽ  `¨7
    | ゙i;;;;;|              ハ__ノ^|」 o o oト、/  ,.-‐'"´\  _」」_」_」、  /
    ,〉、 ゙i;;;;、           ノ-、/, イ:; ̄ ̄`¨`Y;:,r'"´二ニヽ,.-‐''"´`ヽ   ヽi| /
   ,ハ \ト;;、          /ー-/  / .:.:    _,リ´   ̄`\\   ヽ  \  ,j!,イ
   / iヽ  ハ        ;r'ー=,/  /  .:.:  _/        / ゙i\   `,   !/.:;:;!
0739132人目の素数さん垢版2016/08/17(水) 13:26:18.80ID:oj9b67dk
>>738
a^2 + b^2 + c^2 = r^2e^(iθ) とおくと
LHS = |rcosφ| ≦ r = sqrt(|a|^2 + |b|^2 + |c|^2) ≦ 2/3 (|a| + |b| + |c|) ≦ RHS


どや
0740◆2VB8wsVUoo 垢版2016/08/17(水) 13:59:42.68ID:z7oUOJDv


>1 :名無しさん :2006/04/30(日) 01:41:01 ID:KPnB.CH2
> 迷惑かしらん
>
>5535 :¥ ◆2VB8wsVUoo :2016/08/13(土) 18:53:14 ID:???
> ¥
>
>5536 :kmath1107★ :2016/08/13(土) 21:08:24 ID:???
> 人への念の盗み見による介入を阻め。
>
>5537 :¥ ◆2VB8wsVUoo :2016/08/13(土) 21:25:44 ID:???
> ¥
>
>5538 :kmath1107★ :2016/08/13(土) 23:23:07 ID:???
> 人への念の盗み見による介入を阻め。
>
>5539 :kmath1107★ :2016/08/13(土) 23:41:45 ID:???
> 人への念の盗み見による介入を阻め。
>
>5540 :名無しさん :2016/08/14(日) 00:05:47 ID:???
> かつて僕(増田哲也、痴漢で逮捕)が大阪大学基礎工学部の学部学生であった時、大学院の指導教官となってくれそうな先生を
> 探して各地のいろんな先生方を訪ねて回った時の事である。理論物理学を大学院で専攻しようとして理
> 論物理学の初歩をかじっていた僕(増田哲也、痴漢で逮捕)は、当時京都大学数理解析研究所に居られた超一流の理論物理学者で
> おられる中西先生にこんな質問をした事がある。
>
>  僕(増田哲也、痴漢で逮捕) : 「理論物理学では円周率が様々な所に出てきますが、それには何か深い物理的な理由があるの
> でしょうか?」
>
> 中西先生: 「そういう事を何時も頭の片隅に置いておくのはとても大切な事です。でもそんな事ばかり
> 考えていたら研究論文が書けなくなります。研究者とはそんな甘いものではありません。」
>
>  僕(増田哲也、痴漢で逮捕) : 「は−そうなんですか−」
>
> 結局僕(増田哲也、痴漢で逮捕)は京都大学の中西先生ではなく、別の先生に大学院の指導教官になって頂き、理論物理学ではな
> く純粋数学を専攻した。しかしこの時の中西先生のお言葉は今でも何となく「気になって」いる
>
>5541 :名無しさん :2016/08/14(日) 00:22:38 ID:???
> いい加減、芳雄に謝罪しろ
>
0752132人目の素数さん垢版2016/08/17(水) 14:43:40.65ID:87S21yCr
小沢の不等式はノーベル賞級
0753◆2VB8wsVUoo 垢版2016/08/17(水) 15:55:39.46ID:z7oUOJDv


>1 :名無しさん :2006/04/30(日) 01:41:01 ID:KPnB.CH2
> 迷惑かしらん
>
>5535 :¥ ◆2VB8wsVUoo :2016/08/13(土) 18:53:14 ID:???
> ¥
>
>5536 :kmath1107★ :2016/08/13(土) 21:08:24 ID:???
> 人への念の盗み見による介入を阻め。
>
>5537 :¥ ◆2VB8wsVUoo :2016/08/13(土) 21:25:44 ID:???
> ¥
>
>5538 :kmath1107★ :2016/08/13(土) 23:23:07 ID:???
> 人への念の盗み見による介入を阻め。
>
>5539 :kmath1107★ :2016/08/13(土) 23:41:45 ID:???
> 人への念の盗み見による介入を阻め。
>
>5540 :名無しさん :2016/08/14(日) 00:05:47 ID:???
> かつて僕(増田哲也、痴漢で逮捕)が大阪大学基礎工学部の学部学生であった時、大学院の指導教官となってくれそうな先生を
> 探して各地のいろんな先生方を訪ねて回った時の事である。理論物理学を大学院で専攻しようとして理
> 論物理学の初歩をかじっていた僕(増田哲也、痴漢で逮捕)は、当時京都大学数理解析研究所に居られた超一流の理論物理学者で
> おられる中西先生にこんな質問をした事がある。
>
>  僕(増田哲也、痴漢で逮捕) : 「理論物理学では円周率が様々な所に出てきますが、それには何か深い物理的な理由があるの
> でしょうか?」
>
> 中西先生: 「そういう事を何時も頭の片隅に置いておくのはとても大切な事です。でもそんな事ばかり
> 考えていたら研究論文が書けなくなります。研究者とはそんな甘いものではありません。」
>
>  僕(増田哲也、痴漢で逮捕) : 「は−そうなんですか−」
>
> 結局僕(増田哲也、痴漢で逮捕)は京都大学の中西先生ではなく、別の先生に大学院の指導教官になって頂き、理論物理学ではな
> く純粋数学を専攻した。しかしこの時の中西先生のお言葉は今でも何となく「気になって」いる
>
>5541 :名無しさん :2016/08/14(日) 00:22:38 ID:???
> いい加減、芳雄に謝罪しろ
>
0754132人目の素数さん垢版2016/08/17(水) 16:24:03.04ID:oj9b67dk
>>751
LHS = |re^(iφ)| = r = sqrt(|a^2 + b^2 + c^2|) ≦ sqrt(|a|^2 + |b|^2 + |c|^2) ≦ 2/3 (|a| + |b| + |c|) ≦ RHS

あとから細かな間違い見つかって嫌だわ


【質問】
m を正の整数とし,a[1], …, a[m] を区間 [1/2, 1] 上の数列,s[1], …, s[m] をその各次数に対する基本対称式とする。いま
  f(x) := (x-a[1])…(x-a[m]) = x^m - s[1]x + s[2]x^2 -+ …
とし,条件
・m-1/2 ≦ s[1]
・s[m] = 1/2
・f(1/2) ≧(≦) 0
・min[x≧1] f'(x) ≧ 0
・max(min)[x≦1/2] f'(x) ≦(≧) 0
を定めます。≧(≦) は m が偶数なら≧,奇数なら≦を表します。max(min) も同様。
このとき,s[1], … s[m] ならびに a[1], … a[m] を決定することは可能ですか?
m が小さい場合はできたのですが…
0755◆2VB8wsVUoo 垢版2016/08/17(水) 17:42:31.10ID:z7oUOJDv


>1 :名無しさん :2006/04/30(日) 01:41:01 ID:KPnB.CH2
> 迷惑かしらん
>
>5535 :¥ ◆2VB8wsVUoo :2016/08/13(土) 18:53:14 ID:???
> ¥
>
>5536 :kmath1107★ :2016/08/13(土) 21:08:24 ID:???
> 人への念の盗み見による介入を阻め。
>
>5537 :¥ ◆2VB8wsVUoo :2016/08/13(土) 21:25:44 ID:???
> ¥
>
>5538 :kmath1107★ :2016/08/13(土) 23:23:07 ID:???
> 人への念の盗み見による介入を阻め。
>
>5539 :kmath1107★ :2016/08/13(土) 23:41:45 ID:???
> 人への念の盗み見による介入を阻め。
>
>5540 :名無しさん :2016/08/14(日) 00:05:47 ID:???
> かつて僕(増田哲也、痴漢で逮捕)が大阪大学基礎工学部の学部学生であった時、大学院の指導教官となってくれそうな先生を
> 探して各地のいろんな先生方を訪ねて回った時の事である。理論物理学を大学院で専攻しようとして理
> 論物理学の初歩をかじっていた僕(増田哲也、痴漢で逮捕)は、当時京都大学数理解析研究所に居られた超一流の理論物理学者で
> おられる中西先生にこんな質問をした事がある。
>
>  僕(増田哲也、痴漢で逮捕) : 「理論物理学では円周率が様々な所に出てきますが、それには何か深い物理的な理由があるの
> でしょうか?」
>
> 中西先生: 「そういう事を何時も頭の片隅に置いておくのはとても大切な事です。でもそんな事ばかり
> 考えていたら研究論文が書けなくなります。研究者とはそんな甘いものではありません。」
>
>  僕(増田哲也、痴漢で逮捕) : 「は−そうなんですか−」
>
> 結局僕(増田哲也、痴漢で逮捕)は京都大学の中西先生ではなく、別の先生に大学院の指導教官になって頂き、理論物理学ではな
> く純粋数学を専攻した。しかしこの時の中西先生のお言葉は今でも何となく「気になって」いる
>
>5541 :名無しさん :2016/08/14(日) 00:22:38 ID:???
> いい加減、芳雄に謝罪しろ
>
0757132人目の素数さん垢版2016/08/22(月) 09:25:35.49ID:CXTBAMOO
a≧b>0、nは自然数のとき、
(1/n)(a^n-b^n)≦(1/2)(a-b){a^(n-1)+b^(n-1)}
0758◆2VB8wsVUoo 垢版2016/08/22(月) 09:42:21.75ID:q01q4Ck8


>1 :名無しさん :2006/04/30(日) 01:41:01 ID:KPnB.CH2
> 迷惑かしらん
>
>5568 :名無しさん:2016/08/17(水) 18:26:13 ID:???
> うるさい
>
>5569 :kmath1107★:2016/08/17(水) 21:46:32 ID:???
> 人への念の盗み見による介入を阻め。
>
>5571 :名無しさん:2016/08/17(水) 23:39:07 ID:???
> うるさい
>
>5576 :kmath1107★ :2016/08/18(木) 20:58:14 ID:???
> 人への念の盗み見による介入を阻め。
>
>5577 :名無しさん :2016/08/18(木) 21:05:02 ID:???
> >>5575
> うるさい
>
> >>5576
> 賛同致します
>
>5578 :kmath1107★ :2016/08/19(金) 08:46:22 ID:???
> 人への念の盗み見による介入を阻め。
>
> Re:>>5577 人への念の盗み見による介入が無くなれば世の不和が無くなるだろう.
>
>5582 :¥ ◆2VB8wsVUoo :2016/08/19(金) 08:53:36 ID:???
> 芳雄が理想とし、自ら体現する大学教授とは?
> 0.自分が『お教授である』という利点を徹底活用して、偉そうに振舞う。
> 1.年寄りや権威には擦り寄って顔色を窺い、ラクして損しない様にスル。
> 2.難しい分野や困難な研究テーマは徹底して避けて、努力を最小化する。
> 3.高い学歴とか権威を効率的に利用して、自分を飾って偉く見せ掛ける。
> 4.他人に見える部分だけを巧みに繕ってメッキし、人格者のフリをする。
> 5.相手のオツムの質を窺い、シッタカだけで見識がある様に見せ掛ける。
> 6.自分よりも優秀な人間は絶対に敵に回さないでヘラヘラと仲良くする。
> 7.自分から見てダメオツムな野郎は、上から目線で威圧して屈服させる。
> 8.大して中身が無いカラッポ知識を針小棒大に騒ぎ立て、蘊蓄を傾ける。
> 9.自分の大脳が働いてない低能ぶりは、口先で適当に誤魔化して逃げる。
>
> ¥
>
0769132人目の素数さん垢版2016/08/22(月) 11:34:33.16ID:ZJVUMCJi
ageると荒らされるぞ。
このスレを見ている奴はageなくても見ているのだから、書き込みの度にageるなよ >>757
0770132人目の素数さん垢版2016/08/22(月) 11:36:27.52ID:Qopa6DmG
>>757
n{a^(n-1)+b^(n-1)}−2(a^n−b^n)/(a-b)
=n{a^(n-1)+b^(n-1)}−Σ[k=0,n-1] {a^k・b^(n-1-k)+a^(n-1-k)・b^k}
=Σ[k=0,n-1] (a^k−b^k){a^(n-1-k)−b^(n-1-k)}
≧0,
0771132人目の素数さん垢版2016/08/22(月) 11:47:45.37ID:ZJVUMCJi
上がっているスレに対して、自動で荒らし書込をしているんだろうな。

>>704-705 がageた2時間後に 708-734 の荒らし
>>739 がageた後に 740-750 の荒らし
>>752 がageて荒らした後に 754、756 の荒らし
>>757 がageた後に 758-768 の荒らし

常連はageずに出題&解答しているから、他の人もageぬようお願いします。
0773132人目の素数さん垢版2016/08/22(月) 12:51:21.49ID:Qopa6DmG
>>757
加法公式から
sinh(nt)/sinh(t)=cosh((n-1)t)+sinh((n-1)t)/tanh(t)
=cosh((n-1)t){1+tanh((n-1)t)/tanh(t)}
≦n・cosh((n-1)t),

ageさんは分かってないね。
0774132人目の素数さん垢版2016/08/22(月) 15:48:52.26ID:Qopa6DmG
>>770
n{a^(n-1)+b^(n-1)}−2(a^n−b^n)/(a-b)
=Σ[k=1,n-2] (a^k−b^k){a^(n-1-k)−b^(n-1-k)}
=(a-b)^2・Σ[k=1,n-2] k(n-1-k)・a^(k-1)・b^(n-2-k)
≧0,
0786132人目の素数さん垢版2016/08/23(火) 01:38:48.93ID:MlD6b3PU
>>770
n{a^(n-1)+b^(n-1)}−2(a^n−b^n)/(a-b)
=Σ[k=1,n-2] (a^k−b^k){a^(n-1-k)−b^(n-1-k)}
=(a-b)^2・Σ[k=1,n-2] k(n-1-k)・a^(k-1)・b^(n-2-k)
=(a-b)^2・C[n,3]・δ,
とおくと
{(a+b)/2}^(n-3)≦δ≦{a^(n-2)−b^(n-2)}/{(n-2)(a-b)},

ageさんはだめだね。
0787132人目の素数さん垢版2016/08/25(木) 02:15:08.67ID:gDSxL4jk
[第6章.908]
a,b,c>0のとき、{(a+b+c)(ab+bc+ca)}^2≧27abc(a^3+b^3+c^3),
(Inequalitybot[186])☆9


[問題787]
a,b,c>0のとき、(a+b)(b+c)(c+a)(a+b+c)^2≧24abc(aa+bb+cc)
(じゅー、Inequalitybot[196])☆7
0788132人目の素数さん垢版2016/09/02(金) 01:25:56.28ID:8nXURLtK
正の数 x に対して、log(cosh x) > (x・tanh x)/2

            ファサァ
       ∧_∧
       ( ・∀・)       ))
       /つ( ̄`ヽO_ノ⌒ヽ
      ノ   )        \ ))
     (__丿\ヽ ::    ノ:::: )
         丿        ,:'  ))
       (( (___,,.;:-−''"´``'‐'
                 もう寝まつ。
          ∧_∧
          ( ・∀・ )
         /  _ノ⌒⌒⌒`〜、_
      ( ̄⊂人 //⌒   ノ  ヽ)
     ⊂ニニニニニニニニニニニニニニ⊃
0789132人目の素数さん垢版2016/09/04(日) 02:05:19.01ID:kABoMFjs
>>788
x=0のとき等号成立。
(左辺−右辺)’=(1/2)tanh(x)−x/{2cosh(x)^2}
=(1/2)tanh(x){1−(2x)/sinh(2x)}
>0,
0790132人目の素数さん垢版2016/09/05(月) 01:30:20.77ID:k5MxNxjW
>>788
t=tanh(x) のべき級数に展開する。(McLaurin展開)

2*(左辺)=2*log(cosh(x))
=−log(1-tt)
=t^2+(1/2)t^4+(1/3)t^6+(1/4)t^8+…,

2*(右辺)=x*t
=(t/2)log{(1+t)/(1-t)}
=(t/2)log(1+t)−(t/2)log(1-t)
=t^2+(1/3)t^4+(1/5)t^6+(1/7)t^8+…,
0793132人目の素数さん垢版2016/09/05(月) 23:34:31.26ID:HWkA5iDY
>>792
t=tanh(x)とおくと、
(左辺−中辺)’={(3-tt)x+t}/4−t
=(3/4){(1-tt/3)xーt}
≧0,

〔補題〕
x>0 のとき、x>t/(1-tt/3),
0794132人目の素数さん垢版2016/09/05(月) 23:42:53.55ID:HWkA5iDY
>>793
〔補題〕
x>0 のとき、x>t/(1-tt/3),

調和数列>等比数列で、
x=t+(1/3)t^3+(1/5)t^5+(1/7)t^7+…
>t+(1/3)t^3+(1/9)t^5+(1/27)t^7+…
=t/1-tt/3),

x=(1/2)log{(1+t)/(1-t)}
=∫[0〜t] 1/(1-uu) du
>∫[0〜t] (1+uu/3)/(1-uu/3)^2 du
=t/(1-tt/3),

∵相乗-相加平均で
(1-uu)(1+uu/3)=(1-uu/3)^2−(2u/3)^2≦(1-uu/3)^2,
0795132人目の素数さん垢版2016/09/11(日) 02:04:22.52ID:41nsXD0z
〔問題〕
任意の自然数n、および任意の正の実数a_0,a_1,a_2,…,a_nに対して
 1/(a_0+a_1)+1/(a_0+a_1+a_2)+…+1/(a_0+a_1+…+a_n)<k(1/a_0+1/a_1+1/a_2+…+1/a_n)
が成り立つような実定数kの最小値を求めよ。
(JMO-2016春合宿)
0797132人目の素数さん垢版2016/09/12(月) 12:52:02.08ID:7LUxH/Az
k ≦ π^2/6 - 1 ≒ 0.644934 しか分からん
まだ評価甘そう
0798◆2VB8wsVUoo 垢版2016/09/12(月) 13:06:12.78ID:wdbNdCQa


>1 :名無しさん :2006/04/30(日) 01:41:01 ID:KPnB.CH2
> 迷惑かしらん
>
>5714 :名無しさん:2016/09/01(木) 22:40:59 ID:???
> >>5711
> 黙ってろカスが。お前こんなことずっと続けてて父親に申し訳ないと思わないのか。
>
>5718 :¥ ◆2VB8wsVUoo :2016/09/02(金) 07:47:45 ID:???
> >>5714
> マジレスしておくが、芳雄にはきちんと罰だけは受けて貰う。あんな酷い事をし
> ておきながら、無傷であの世に逃亡というのは絶対に許されない。死ぬ前に充分
> な精神的苦痛をタップリと味わうべき。あの糞野郎だけは絶対に許されないので。
> 芳雄に対する怒りと憎しみは、馬鹿板に対する怒りとは比べ物にはならんわ。
>
> ¥
>
>5720 :¥ ◆2VB8wsVUoo :2016/09/02(金) 08:54:09 ID:???
> >>5714
> 言って置くが、被害を受け始めた高校生の頃から私は芳雄を論理分析し、その欠
> 陥や弱点を精密に理解し、そしてその横暴極まりない無責任な態度に対抗しなが
> ら狙い撃ちにして来た。私は芳雄のせいで甚大な被害を被ったのであり、それを
> 「親が責任を取る」という様ないい加減な逃げ口上で逃亡し、無責任を通す卑怯
> な行為は到底許されない。なのでその報いだけでもきちんと受けさせてやるだけ。
>
> 糞芳雄の野郎、このまま逃げ切りは許さない。自分から言い放った『親としての
> 責任』というものが微塵でも残ってるのであれば、それ相当の行為が自らなされ
> て当然というものだろう。手を切り落とすもよし、足を切り落とすもよし。或い
> は自分で主張した釜ヶ崎に自分で行って、そして労務者にでも殴られて撲殺され
> るのもいいだろう。
>
> とにかく自分で言った事だけは、きちんと自分から実行するべき。知らぬ存ぜぬ
> で、無責任な逃げ切りだけは許されない。ソレこそが芳雄が言う所の卑怯者だか
>  らだ。糞芳雄は恥を知るべき。今からでもいいから、尊厳の意味を理解するべき。
>
>  ¥
>
0809132人目の素数さん垢版2016/09/15(木) 10:32:49.10ID:glMCJfdL
どうせ期限切れになるのを見越してるのかもしれないが、
債務者に無断で過払い金の返還請求してネコババする(弁)は辞めてもらいたいね。→
0810132人目の素数さん垢版2016/09/18(日) 10:38:59.46ID:KcEt3KAv
>>795
n=1 で k=0.25    (a0=a1=1)
n=2 で k=0.3009441531(a0=a1=1 a2=2.4305)   a2=2(1+√7)/3
n=3 で k=0.3190867373(a0=a1=1 a2=2.1491 a3=5.3864)
n=4 で k=0.3266922362(a0=a1=1 a2=2.0639 a3=4.4987 a4=11.4234)
だから、もうチョト大きそう…
0811132人目の素数さん垢版2016/09/20(火) 15:01:00.56ID:znrqlsji
>>795
等比数列
a_0=a_1=1、a_k=r^(k-1) (公比r>1)
の場合を考える。

(右辺)=k{1+1+1/r+…+1/r^(n-1)}
=k{2r-1-(1/r)^(n-1)}/(r-1)
→k(2r-1)/(r-1),  (n→∞)
(左辺)はチト面倒だが…

r≒2の辺りでkは最大になる。

r=2の場合はk→1/3 (n→∞)
0812811垢版2016/09/20(火) 15:33:02.10ID:znrqlsji
>>795
等比数列
a_0=a_1=1、a_j=r^(j-1) (公比r>1)
の場合を考える。

訂正スマソ
0813132人目の素数さん垢版2016/09/20(火) 20:49:49.22ID:YmQwqlus
参考文献に挙げられていた論文を国会図書館から取り寄せたら、ドイツ語でした… Σ(゚Д゚ )!
0815132人目の素数さん垢版2016/10/03(月) 04:50:15.28ID:hlTdGaar
正の実数 x、y、z が xyz=1 をみたすとき、
√{(x+1)/(x^2-x+1)} + √{(yx+1)/(y^2-y+1)} + √{(z+1)/(z^2-z+1)} ≦ 3√2


 "; ;ヾ; ;ヾ; ;メヾ "ゞ ;ヾ ;ゞ ;" "ゞ ; ; ; ゞ ;" "ゞ";ヾ ; ヾ ;ゞ; ;ゞ ;ゞ ;" "ゞ        /.             ヽ
 ;" "ゞ ; ; ; ゞ ; ;ヾ ; ; ヾ ;ゞ;ヾ ; ;";ヾ; ;"/" ; ;ヾ ;ヾ; ヾ ; ヾ ;ゞ; ;ゞ ;" ";ゞ ; ;ヾ      l             l
 " ;ヾ ; ;";ヾ; ;"/" ; ;ヾ ;ヾ;ヾ ; ;ヾ ; ; ヾ ;ゞ  " ;ヾ ; ;";ヾゝゝ" ;ヾゞ           ヽ            /
,." ;ヾ ; ;";ヾ; ;"/" ; ;ヾ ;ヾ;ヾ ; ;ヾ ; ; ヾ ;ゞ  " ;ヾ ; ;";ヾゝゝ" ;ヾ ; ; ヾ ;ゞ;        \        /
 ゞヾ ; ;" ; ; ;; ;"iiiiii;;;;;::::: :)_/ヽ,.ゞ:,,ヾゞヾゞ__;::/        `      `        `   ー ─ ' `
   ゞヾゞ;\\iiiiii;;;;::::: :|;:/ヾ; ;ゞ "ゝゞ ; ;`
 " ;゛ ; ;" ; ;ゞ "|iiiiii;;;;::: : |:/ ヾゞ        `         `      ` `
  `      ,|i;iiiiiii;;;;;;::: :| `    `         `     `      ` `   `
        ,|iiii;iiii;;;;:;_ _: :|  ___  秋の夜長に不等式    `        `        `,
   `    |iiiiiii;;;;;;((,,,)::.:|/  ≧ \                    ヾ从//"
    `   |iiiiiiii;;ii;;;;;;~~~:|:::: (● (●|           `  ゙  `    ヾ'./"
,         |iiiiii;iii;;;;i;;:: :: ::|ヽ::::......ワ...ノ                ○     .||.       ,
    `   |iii;;iiiii;::;:;;;;::: :::| ( つ且 ~      `             ○○   | |
  , , .,.. ,..M|M|iMii;;ii:i;;i:i;:; ゝ つつ.,.. ,...... ,.... ,,,.,.. ,.... ,,,.,.. ,..,,,,.,...,..,.,| ̄ ̄|,.,..(  ).. ,,,..,,.. ,.... ,,,.,...,.. .. ,.... ,,,.,.. ,.... ,,,
0816132人目の素数さん垢版2016/10/08(土) 16:41:27.99ID:VzLBekw5
>>815

凸性より
(左辺)≦(√3)√{(x+1)/(xx-x+1)+(y+1)/(yy-y+1)+(z+1)/(zz-z+1)},
ゆえ、
 (x+1)/(x^2-x+1)+(y+1)/(y^2-y+1)+(z+1)/(z^2-z+1)≦6,  …(*)
を示そう。

(i) x,y,z≦2 のとき
 (a+1)/(aa-a+1)=(3-a)−(2-a)(1-a)^2/(aa-aa+1)≦3-a,
 (*)≦9-(x+y+z)≦6,

(ii) x≧2 のとき
 (x+1)/(xx-x+1)=1−x(x-2)/(xx-x+1)≦1,
 (b+1)/(bb-b+1)=M{1−(b+1-√3)^2/(bb-b+1)}≦M,
 ここで M=1+(2/√3)=2.1547
 (*)≦1+M+M=3+(4/√3)=5.3094<6,
0817132人目の素数さん垢版2016/10/09(日) 15:18:04.31ID:Dmd9ztww
>>816
凸関数じゃないんだけど


>>815
f(x) = (x+1)/sqrt(x^3+1) とおく。x ≧ y ≧ z と仮定してよい

・x ≦ 32.82951185 のとき
f(x) ≦ -log(x)/2sqrt(2) + sqrt(2) からこれを巡回的に足して主張を得る

・x ≧ 32.82951185 のとき
z ≦ 1/sqrt(32....) = 0.174529 である。よって
 f(x) ≦ f(32...) ≦ 0.179843
 f(y) ≦ f(0.73...) ≦ 1.46788
 f(z) ≦ f(0.17...) ≦ 1.17142
となる。したがって
 LHS ≦ 0.179843 + 1.46788 + 1.17142 ≦ RHS
が得られる


3f(x) ≦ 4.40366..., RHS = 4.24264 だから値域を少し厳密に評価するだけで解ける
0818816垢版2016/10/10(月) 04:11:35.08ID:noR8aJyR
>>817
凸性と言ったのは
 √a + √b + √c ≦ √(1+1+1)・√{a+b+c},
という意味です。

コーシーを持ち出すまでもないと思ったので…
0819132人目の素数さん垢版2016/10/10(月) 05:41:25.02ID:noR8aJyR
>>815 を改造してみる…

実数 x、y、z(≧-1)が x+y+z≧3 をみたすとき、
√{(x+1)/(xx-x+1)} + √{(y+1)/(yy-y+1)} + √{(z+1)/(zz-z+1)} ≦ 3√2,

実数 x、y、z(≧-1)が x+y+z≦3/2 をみたすとき、
√{(x+1)/(xx-x+1)} + √{(y+1)/(yy-y+1)} + √{(z+1)/(zz-z+1)} ≦ 3√2,
0820132人目の素数さん垢版2016/10/14(金) 16:19:00.33ID:s2NnBcrE
正の実数 a, b, c に対して次の不等式を示せ
 Σ[cyc] (a+2c)/(a+2b) ≧ sqrt((5(a^2+b^2+c^2)/(ab+bc+ca)) + 4)
0821132人目の素数さん垢版2016/10/15(土) 04:34:28.61ID:htY30JEj
>>820
s=a+b+c, t=ab+bc+ca, u=abc, =(a-b)(b-c)(c-a) とおくと、
(左辺) = 2s{1/(a+2b)+1/(b+2c)+1/(c+2a)}−3 = 2s{(2ss+3t)/(3st-)}−3,
(右辺) = √(5ss/t−6),
さて、どうする?
0822132人目の素数さん垢版2016/11/08(火) 04:15:31.61ID:LIWaiFBV
〔相加-相乗平均〕

(a_1)^n+(a_2)^n+……+(a_n)^n− n・a_1・a_2……a_n
=Σ[i<j] (a_i-a_j)^2 P_(i,j)

P_(i,j)={1/(n-1)}Σ[k=0,n-2] {ai^(k+1)−aj^(k+1)}/(ai-aj)・Q_(n-2-k)(i,j)

Q_L(i,j)は、aiとajを除く(n-2)文字によるL次の基本対称式を、その項数C[n-2,L]で割ったもの。
Q_0=1,

(注)
{ai^(k+1)−aj^(k+1)}/(ai−aj)=(ai)^k+(ai)^(k-1)・aj+……+ai・(aj)^(k-1)+(aj)^k,
なので、正係数の多項式である。

「フルヴィッツ・ムーアヘッドの等式」と云うらしい。
0823132人目の素数さん垢版2016/11/16(水) 13:25:30.54ID:f4/M3jIZ
任意の非負実数 x, y, z に対して次の不等式が成り立つ最良の正の定数 k は?
   |(x-y)(y-z)(z-x)| ≦ k(x+y+z)((x-y)^2 + (y-z)^2 + (z-x)^2)
0824132人目の素数さん垢版2016/11/23(水) 09:39:23.50ID:KCIuqlXC
>>823
x,y,zを一斉に増すと、右辺は増加、左辺は不変。

∴{x,y,z}={x,1,0} としてもよい。

(3k)^4+3(3k)^2−(3/16)=0,

k=(-1+√3)/{12^(3/4)}=0.1135416731
0827132人目の素数さん垢版2016/11/23(水) 15:54:15.99ID:n5BWZq4/
(1) a+b+c=3 をみたす任意の正の数 a, b, c に対して次の不等式が成り立つ最良の正の定数は?
  (a-b)(b-c)(c-a) ≦ 4/√(27abc)

(2) a+b+c=1 をみたす正の数に対して次の最大値は?
  (abc)^(1/3)(a-b)(b-c)(c-a)


両方共ワカラン
0829132人目の素数さん垢版2016/12/01(木) 01:08:06.58ID:laYsAhNA
hlawka って何て読むんだすか? らうか?
何人?
0840132人目の素数さん垢版2016/12/02(金) 08:07:58.05ID:7xVhMtBb
>>827 (1)

題意より
 a+b+c = 3,
また、等号成立条件より
 ab+bc+ca = 2,
 abc = 2/9,
が出るので、(a,b,c)は
 t^3 -3t^2 +2t -2/9 = 0,
の3実根。
すなわち
a = 1 + (2/√3)cos(θ/6) = 2.096648361
b = 1 + (2/√3)cos(θ/6 - 2π/3) = 0.764760120
c = 1 + (2/√3)cos(θ/6 + 2π/3) = 0.138591519
θ = arccos(-1/3) = 109゚ 28' 16.4”(四面体角)
0861132人目の素数さん垢版2016/12/03(土) 01:04:48.92ID:JV/Azs1X
実数 a, b, c が a^2+b^2+c^2=3 をみたすとき,a+b+c-abc の最大値は?
0862◆2VB8wsVUoo 垢版2016/12/03(土) 01:07:58.16ID:gn3EMfBZ


>前科持ち変質者と絶対出会える掲示板 [無断転載禁止]
>
>1 名前:132人目の素数さん 2016/11/16(水) 21:02:24.40 ID:8UX5OsVV
> 変質者前科持ちと気が触れ合える掲示板
>
>11 名前:132人目の素数さん :2016/11/19(土) 08:36:12.59 ID:6KwDBI7h
> 変質者前科持ち=増田哲也
>
>12 名前:132人目の素数さん :2016/11/19(土) 09:04:39.15 ID:AZB04dZ8
> わざわざ言わんでもええ
>
>13 名前:出会える掲示板 ◆2VB8wsVUoo :2016/11/19(土) 15:58:01.20 ID:21LrO2+x
> 絶対に…
>
> ケケケ¥
>
>14 名前:132人目の素数さん :2016/11/19(土) 16:31:33.55 ID:6KwDBI7h
> 六十目前で父親逆恨みしたり掲示板逆恨みする根性の腐れっぷりは凄くて困る
>
0873132人目の素数さん垢版2016/12/04(日) 03:23:03.70ID:igohLjS8
>>861

a = b = √{(5+√13)/6} = 1.197605338
c = -√{(4-√13)/3} = -0.36260572
のとき
a+b+c - abc = 2a +(aa-1)(-c)
= 2a + ((√13 -1)/6)(-c)
= 2.552675308961574826258
かな。
0886132人目の素数さん垢版2016/12/04(日) 07:47:28.84ID:igohLjS8
>>861
ついでに…
a=b=c=1 で 2(鞍点?)
a=b=0.4820872, c=1.5922260 で 2.1863542858636(極大)
a=b=0, c=√3 で √3(極小?)
0897132人目の素数さん垢版2016/12/05(月) 11:42:11.58ID:9Kv+bmZv
xi > 0,
A(n) = (x1+x2+...+xn)/n,   相加平均
G(n) = (x1・x2・...・xn)^(1/n),  相乗平均
H(n) = n/(1/x1+1/x2+...+1/xn),  調和平均
とおく。

[1] 略

[2]
A(2) + m・H(2) ≧ (1+m)G(2), m=1.0

[3]
A(3) + m・H(3) ≧ (1+m)G(3), m=0.90096030150908885
 
(1,1,x3) x3=0.396257004730747667698678 は 64x^3 +87x^2 -42x -1 =0 の根

[4]
A(4) + m・H(4) ≧ (1+m)G(4), m=0.7761577683742073233
 
(1,1,1,x4) x4=0.229929540827345357763 は 6561x^5 +18299x^4 +11210x^3 -3210x^2 -91x-1=0 の根

が成り立つか?
0899132人目の素数さん垢版2016/12/05(月) 15:21:43.32ID:SKxowvFC
>>897
(3)
A[3] + m*H[3] ≧ n*G[3]
ここで m=0.90096, n=m+1 が最適な係数
等号成立は (1, 1, 0.39625)

(4)
A[4] + m*H[4] ≧ n*G[4]
ここで m=0.77615, n=m+1 が最適な係数
等号成立は (1, 1, 1, 0.39625) または (1, 1, 4.32911, 4.32911)
0902132人目の素数さん垢版2016/12/05(月) 23:41:34.31ID:SKxowvFC
>>897
[4]
p=1.444113430416044 は x^5+3*x^4+6*x^3-6*x^2-11*x-9=0 の解
q=0.692466380367298 は 9*x^5+11*x^4+6*x^3-6*x^2-3*x-1=0 の解
等号成立は (p, p, p, 1), (q, 1, 1, 1) のとき

等号を成立させる方程式の係数と符号が反転してて面白いので載せてみた
0904132人目の素数さん垢版2016/12/06(火) 06:14:40.46ID:61lM6Ipy
>>898-903
thx.


>>897
x_3 = t^3 とおくと、
 4t^3 + 3t^2 - 3t - 1 = 0,
 t = {2(√5)cosθ -1}/4 = 0.734500874964259
 ただし θ = (1/3)arccos[1/(5√5)] = 0.49374463978515
0905132人目の素数さん垢版2016/12/06(火) 07:59:56.74ID:61lM6Ipy
>>898
n-5 は

[5]
A(5) + m・H(5) ≧ (1+m)G(5), m=0.676175

(1,1,1,1,r^5) r = 0.6897105532534071796 は 16r^7 + 23r^6 +21r^5 +10r^4 -10r^3 -6r^2 -3r -1 =0 の正根。

と予想するが...
0907132人目の素数さん垢版2016/12/09(金) 00:40:57.61ID:sgxdKdhy
最近の不等式の証明技法をまとめようかなと思ってるけど面倒でやる気が起きない
0908132人目の素数さん垢版2016/12/11(日) 09:01:03.18ID:688mVHLv
>>906 のとき
x = {1,1,・・・・,t^n}

A(n) = [t^n + (n-1)] / n,
G(n) = t,
H(n) = n・t^n / [(n-1)t^n + 1]

A-G = (t-1)^2 (1/n) f(t),
G-H = (t-1)^2 {t/[(n-1)t^n + 1]} g(t),
(A-G)/(G-H) = [(n-1)t^n +1]f(t) / {nt・g(t)} ≧ m,

ここで
f(t) = [t^n -nt +(n-1)] / (t-1)^2 = t^(n-2) + 2t^(n-3) + ・・・・ + (n-2)t + (n-1),
g(t) = [(n-1)t^n -nt^(n-1) +1] / (t-1)^2 = (n-1)t^(n-2) + (n-2)t^(n-3) + ・・・・ +2t +1,

(A-G)/(G-H) が極小のとき、

[(n-1)^2・t^n -1]{f(t)/g(t)} + [(n-1)t^n +1]t{f(t)/g(t)} ' = 0,
[(n-1)^2・t^n -1]f(t)g(t) - [(n-1)t^n +1]t{f(t)g '(t) - f'(t)g(t)} = 0,
ここで
f(t)g(t) = Σ[k=0〜n-3] ((k+1)(k+2)(3n-3-k)/6){t^k + t^(2n-4-k)} + ((n-1)n(2n-1)/6)t^(n-2),
f(t)g '(t) - f '(t)g(t) = n・Σ[k=0〜n-4] ((k+1)(k+2)(k+3)/6){t^k + t^(2n-6-k)} + n((n-2)(n-1)n/6)t^(n-3),
0909132人目の素数さん垢版2016/12/19(月) 03:17:02.97ID:4qCEI1DC
>>897 >>905

Sierpinskiの不等式
 A(n)^(n-1)・H(n)≧G(n)^n
を使えば
 A(n) + (1/(n-1))H(n) ≧ (n/(n-1)){A(n)^(n-1)・H(n)}^(1/n) ≧ (n/(n-1))G(n),
 m ≧ 1/(n-1),
は簡単に出ます。

しかし掛け算すると、x→(1,1,・・・・,1,0)のとき下限値1/(n-1)に近づくので、これ以上改良できそうにない…
というワケで加減で比べてみました。
0910132人目の素数さん垢版2016/12/20(火) 06:49:30.48ID:9UZFmJjk
>>909
〔Sierpinskiの不等式〕

A(n+1)^n・H(n+1)/G(n+1)^(n+1) ≧ A(n)^(n-1)・H(n)/G(n)^n ≧ ・・・ ≧ A(2)H(2)/G(2)^2 = 1,

(略証)
nについての帰納法で。
n=2のとき、等号成立。
x_{n+1} = x,
A(n)=Ao, G(n)=Go, H(n)=Ho,
A(n+1)=A, G(n+1)=G, H(n+1)=H,
と略記する。
A = (n・Ao + x)/(n+1)
G^(n+1) = x・Go^n,
1/H = (n/Ho + 1/x)/(n+1),

(A^n・H)/G^(n+1) ÷ {Ao^(n-1)・Ho}/Go^n
= {A^n/Ao^(n-1)} H/(Ho・x)
≧{n・A -(n-1)Ao} H/(Ho・x)
= (Ao + nx)/(Ho + nx)
≧ 1,   (← Ao≧Ho)
0911132人目の素数さん垢版2016/12/20(火) 14:43:09.84ID:9UZFmJjk
〔問題〕
A, B が実対称行列のとき、次を示せ。
 tr{exp(A+B)}≦ tr{exp(A)exp(B)},
等号成立は AB=BA のとき。
 
(京大RIMS元所長)荒木教授ご提出らしい。

数セミ増刊「数学の問題」第2集、日本評論社(1978) No.96
0913132人目の素数さん垢版2016/12/23(金) 22:29:02.46ID:N1oFke4u
>>909
〔Jacobsthalの不等式〕

 (n+1)(A-G) ≧ n(Ao - Go),

(略証)
(左辺)= (n・Ao +x) -(n+1)(Go^n・x)^{1/(n+1)}
 ≧(n・Ao +x) - (n・Go +x)
 = n(Ao - Go)
 =(右辺),
0914132人目の素数さん垢版2016/12/25(日) 03:08:58.00ID:HgkzhkFu
>>913
(n+1)A - nAo = x = G^(n+1)/Go^n ≧ (n+1)G - nGo,

∴ (n+1)(A-G) ≧ n(Ao-Go),

同様にして
 A^(n+1)/Ao^n ≧ (n+1)A - nAo = x = G^(n+1)/Go^n,

∴ (A/G)^(n+1) ≧ (Ao/Go)^n,
0915132人目の素数さん垢版2016/12/25(日) 05:29:38.31ID:HgkzhkFu
>>912

nが10〜1000 の辺りでは

m 〜 {1.1287*log(n) + 1.2272}/(n-1),

1/H 〜 1.153*log(n),

1/x 〜 1.153n*log(n) - (n-1),

A(n) = (n-1+x)/n,

らしい。
0917132人目の素数さん垢版2017/01/08(日) 06:07:07.63ID:rl9rb1ia
>>916 (注意)

1/(aa+3) + 1/(bb+3) + 1/(cc+3) ≦ 3/4,
は成り立ちません。

a = b = 0.29712745268     (*)
c = 2.40574509464
のとき、
0.761405273304


(*) 2a^3 -7a^2 +12a -3 = 0 の根
(1/6){7 + (36√58 -251)^(1/3) - (36√58 +251)^(1/3)},
0918132人目の素数さん垢版2017/01/10(火) 04:33:57.70ID:FU/1ZKud
>>916 (注意)

(2√b)/(a+3) + (2√c)/(b+3) + (2√a)/(c+3) ≦ 3/2,
も成り立ちません。

a = 0.818145
b = 0.823310
c = 1.358545
のとき
1.500059562452
0919132人目の素数さん垢版2017/01/11(水) 12:38:59.99ID:o5/kKbcv
〔問題〕
a,b,cを正の実数とするとき、次を示せ。

[2] a + √(ab) ≦ {(1+√2)/2}(a+b),

[3] a + √(ab) + (abc)^(1/3) ≦ (4/3)(a+b+c),
0920132人目の素数さん垢版2017/01/12(木) 09:21:49.92ID:OCuLi6LZ
a,b,c,dを正の実数とするとき、

[4] a + √(ab) + (abc)^(1/3) + (abcd)^(1/4) ≦ K(4)(a+b+c+d),

K(4) = 1.4208443854096138127
0921132人目の素数さん垢版2017/01/12(木) 11:12:31.15ID:OCuLi6LZ
>>919-920

〔Carlemanの不等式〕(有限版)

相加-相乗平均をたした形であるが、そのままでは係数が合わない。
そこで正の係数 c_1〜c_n を掛けて

K(n)・(a1+a2+・・・・+an) − {a1 + √(a1・a2) + ・・・・・ + (a1・a2・・・・an)^(1/n)}

= Σ[L=2〜n] {(c1・a1+c2・a2+・・・・・+cL・aL)/(L・d_L) - (a1・a2・・・aL)^(1/L)},

とおく。ここに、d_L = (c1・c2・・・・・cL)^(1/L),

a_L の係数を比べて
1/(L・dL) + 1/((L+1)d_(L+1)) + ・・・・ + 1/(n・dn) = K/c_L,
      1/((L+1)d_(L+1)) + ・・・・ + 1/(n・dn) = K/c_(L+1),
辺々引いて
1/(L・dL) = K/c_L − K/c_(L+1),
∴ 1/c_(L+1) = 1/c_L − 1/(K・L・d_L),
により c_Lが定まる。
c_1 = 2 とおくと、
c_2 = 2K/(K-1),
c_3 = 2K/{K -1 -√((K-1)/4K)},
・・・・
また、K(n) は 1/c_(n+1)=0 から定まる。
0923132人目の素数さん垢版2017/01/17(火) 10:39:38.76ID:Qggnth+1
〔Stirlingの公式〕
正の整数nについて
log(n!) > (n+1/2)log(n)−n+0.8918
を示せ。
0925132人目の素数さん垢版2017/01/22(日) 07:55:35.43ID:j1H92TDS
>>924

・0<a≦1/(2√5)のとき[3a+1/(4a),1/a]
  最小:(x,y)=(1/2,2a)
  最大:(x,y)=(1,a)

・1/(2√5)≦a≦1/(2√2)のとき[2√{1-(2a)^2},1/a]
  最小:(x,y)=(2a/√{1-(2a)^2},2a)
  最大:(x,y)=(1,a)

・1/(2√2)≦a≦1/2 のとき[1/(2a),1/a]
  最小:(x,y)=(1,2a)
  最大:(x,y)=(1,a)

・1/2≦a のとき[1/(2a),3a+1/(4a)]
  最小:(x,y)=(1,2a)
  最大:(x,y)=(1/2,2a)
0928132人目の素数さん垢版2017/02/14(火) 01:57:11.20ID:U44OFY/t
単位円(原点Oを中心とする半径1の円)の周上に2点 A,B がある。
∠AOB = ω の二等分線を OM とすると
∠AOM = ∠MOB = ω/2,
また、OMと反対の方向に点Cをとる。
∠OCA = θ,OC=k とおくと、
tanθ = sin(ω/2)/{k+cos(ω/2)},
とくに k=2 のとき
tanθ = sin(ω/2)/{2+cos(ω/2)}< ω/6,(仁平氏による)

数セミ '17年3月号 p.44 NOTE
0929132人目の素数さん垢版2017/02/14(火) 02:35:05.63ID:U44OFY/t
>>928

〔補題〕
0<t<π のとき
 sin(t) < 3sin(t)/{2+cos(t)} < t,
 {sin(t),sin(t),tan(t)}の調和平均はtより小さい。(B.C.Carlson)

(略証)
左側は明らか。
右側はtで微分して
3cos(t)/{2+cos(t)}+3{sin(t)}^2/{2+cos(t)}^2
=1−3{[1-cos(t)]/[2+cos(t)]}^2
< 1,

不等式の和書[3] p.45 の式でxをcos(2t)とおく。

なお、相加平均はtより大きい。(Snellius-Huygens)
0931132人目の素数さん垢版2017/02/22(水) 16:20:23.47ID:zQPH35Dc
>>930

a,b≧c≧0 としてもよい。

(左辺) = (a-c)(a-2b+c)^2 + b(a-b)^2 + c(b-c)^2 + c(c-a)^2 ≧ 0,

対称式ぢゃないからチョト面倒...
0933132人目の素数さん垢版2017/02/26(日) 23:13:36.91ID:DmeGzA4L
>>929 〔応用問題〕

1周の長さが 2π である正n角形において、外接円の半径をR、内接円の半径をrとするとき、

(1) r < 1 < R,

(2) 3/(2/R + 1/r) < (RRr)^(1/3) < 1 < (2R+r)/3,

を示せ。
0934132人目の素数さん垢版2017/02/27(月) 17:58:31.54ID:tcvWjEXJ
>>932

p≦3/2 のとき
{ab(1-c)}^(1/p)≦{ab(1-c)}^(2/3)
≦{ab+b(1-c)+(1-c)a}/3  (←相乗・相加平均)
={1-(1-a)(1-b)+(2ab-bc-ca)}/3
≦{1 + (2ab-bc-ca)}/3,
巡回的にたす。

p>3 - log(4)/log(3)= 1.7381405 のとき不成立
反例 (a,b,c)=(2/3,2/3,2/3)
0936132人目の素数さん垢版2017/03/03(金) 07:01:45.16ID:Fl6w77Qk
△ABCについて次を示せ。
(tan(A/2)+tan(B/2))^(-1/2)
+(tan(B/2)+tan(C/2))^(-1/2)
+(tan(C/2)+tan(A/2))^(-1/2)
≧2+2^(-1/2)
0938132人目の素数さん垢版2017/03/05(日) 22:51:16.40ID:rca0XhBC
〔問題2714〕

a,b,c,p,q,r は正の実数で、abc=1, p≧2, q≧2, r≧2 をみたすとする。

(a^p +p)(b^q +q)(c^r +r)
 ≧ (2+aa)(2+bb)(2+cc)
 ≧ (2+1/a)(2+1/b)(2+1/c)
 ≧ (2+√a)(2+√b)(2+√c)
 ≧ {2 + 1/a^(1/4)}{2 + 1/b^(1/4)}{2 + 1/c^(1/4)}
 ≧ {2 + a^(1/8)}{2 + b^(1/8)}{2 + c^(1/8)}
 ≧ ・・・・・
 ≧ 27,
を示せ。(「すうじあむ」の問題を元に改作)

http://suseum.jp/gq/question/2714
0939132人目の素数さん垢版2017/03/08(水) 00:01:03.85ID:hSPnYqZt
正の数nと、正の実数a_1、…a_nに対し、次式をみたす実数Mの最大値を求めよ。

n・Σ[1≦k≦n] (a_1 + … + a_k)・(a_k)^2 ≧ M・(a_1 + … + a_n)^3
0940132人目の素数さん垢版2017/03/08(水) 07:51:24.09ID:hSPnYqZt
The positive numbers x, y satisfy the equation x^3 + y^3 = x?y. Prove that x^2 + y^2 < 1.

The positive numbers a_1,…,a_n satisfy √a_1 + … + √a_n = 1. Show that (a_1)^(a_1)・…・(a_n)^(a_n) ≧ (a_1 + … + a_n)^2.

Let 0 < x_1 < x_2 <…< x_n < 2π. Show that 納i,j=1;i≠j to n] 1/|xi?xj| + 1/{2π?|xi?xj|} ≧ (n^2/π)納k=1 to n?1] 1/k.
0941132人目の素数さん垢版2017/03/09(木) 09:14:49.54ID:3QaTlDUD
-が?に文字化けしているな

Let 0 < x_1 < x_2 <…< x_n < 2π. Show that 納i,j=1;i≠j to n] (1/|x_i-x_j| + 1/{2π-|x_i-x_j|} ) ≧ (n^2/π)納k=1 to n-1] 1/k.
0942132人目の素数さん垢版2017/03/10(金) 00:46:09.66ID:VVolMD9v
>>939
n=1 のとき M_1 = 1,
n=2 のとき M_2 = 2(47-14√7)/27 = 0.73773938
 2{a^3 + (a+b)bb} - M_2・(a+b)^3 = (2-M_2)・(a-tb)^2・(a+b/tt) ≧0,
 t = (1+√7)/3 = 1.215250437


>>940
題意より xy ≧ 0,
(x-y)y ≧ 0,
xx+xy+yy = (x^3-y^3)/(x-y) = 1 - 2(y^3)/(x-y) ≦ 1,
0943132人目の素数さん垢版2017/03/11(土) 03:43:57.83ID:S/5xVczT
>>939

M_n は既知とし、
a_{n+1} = x,
a_1 + a_2 + ・・・・+ a_n + x = S,
とおく。

a_1 + a_2 + ・・・・+ a_n = S-x,

(左辺) = {M(n)/n}(S-x)^3 + Sxx = f(x),
f '(x) = -3{(M_n)/n}(S-x)^2 + 2Sx
 = -3{(M_n)/n}{xx - 2(coshθ)Sx + SS}  {coshθ=1+n/(3M_n) とおいた}
 = -3{(M_n)/n}{x - S・e^(-θ)}(x - S・e^θ),
左辺は x = S・e^(-θ)で最小となる。このとき
S - x = S{1 - e^(-θ)},
f(S・e^(-θ)) / S^3 = {(M_n)/n}{1 - e^(-θ)}^3 + e^(-2θ)
 = M_(n+1)/(n+1),
ここに、coshθ = 1 + n/(3M_n),
これにより M_{n+1} が定まる。

M = lim[n→∞] M_n = 4/9.
0944132人目の素数さん垢版2017/03/11(土) 10:10:59.71ID:S/5xVczT
>>939(補足)

e^(-θ) ≒ 1/(2coshθ) = 1/{2(n/3M_n + 1) = (3/2)M_n/(n+3M_n) を使って

f(S・e^(-θ)) / S^3 = (1/3)e^(-θ){2+e^(-θ)} ≒ M_n{n+(15/4)M_n}/(n+3M_n)^2

∴ M_{n+1} - M_n = M_n*(n+1){n+(15/4)M_n}/(n+3M_n)^2 - M_n = M_n*{1-(9/4)M_n}/n + O(1/nn),

1/n の係数が0に収束しないと M_n が発散してしまうから、
M = lim[n→∞] M_n = 4/9,
0945132人目の素数さん垢版2017/03/13(月) 20:43:05.22ID:UB++6Hh4
>>940 下
k=1,2,…,n-1 とする。
 Σ[i-j=k] 1/(x_i - x_j) および Σ[i-j=n-k] 1/{2π - (x_i-x_j)}
のn項について、相加-調和平均(コーシー)すると、
 Σ[i-j=k] 1/(x_i - x_j) + Σ[i-j=n-k] 1/{2π - (x_i-x_j)}
 ≧ nn/{Σ[i-j=k] (x_i - x_j) + Σ[i-j=n-k] {2π - (x_i-x_j)}}
 =(nn/2π)(1/k),
k=1,2,…,n-1 でたす。
等号成立は x_i - x_j = (2π/n)(i-j)のとき。
0946132人目の素数さん垢版2017/03/15(水) 22:14:08.94ID:Oh51f5Dy
>>940 下(続き)

・・・ さらに、0<x<2π で f(x)が下に凸のとき

Σ[1≦j<i≦n] {f(x_i-x_j) + f(2π-x_i+x_j)} ≧ n Σ[k=1,n-1] f(2kπ/n),

f(x)が上に凸のときは、不等号が逆向き。
0947132人目の素数さん垢版2017/03/16(木) 18:00:56.77ID:/k5pY9BZ
>>940

0 < a_k < 1 より
 (左辺) > a_1 + a_2 + … + a_n,
 1 = √a_1 + √a_2 + … + √a_n > a_1 + a_2 + … + a_n,
辺々掛ける。
0949132人目の素数さん垢版2017/03/20(月) 18:16:48.99ID:ZS4SrzTA
For n≧2, let a_1, …, a_n be posithive real numbers. Prove
 { Π[i=1 to n] (1+a_i) }^{n-1} ≧ { Π[1≦i<j≦n] [1 + (a_i・a_j)/(a_i + a_j)] }^2
0950132人目の素数さん垢版2017/03/20(月) 22:19:37.49ID:yRo4d+xZ
>>949
a,b>0のとき, a>ab/(a+b)だから
(1+a)(1+b)>(1+ab/(a+b))^2
これを使って終わり.

>>936も等号が成立しない気がする.
0951132人目の素数さん垢版2017/03/21(火) 06:34:34.56ID:lHafklKO
>>949
a,b>0のとき、√ab ≧ 2ab/(a+b) だから
(1+a)(1+b) ≧ (1+√ab)^2≧ {1+2ab/(a+b)}^2
これを使って終わり.
等号成立は a_i = 一定 のとき。
0952132人目の素数さん垢版2017/03/23(木) 23:56:17.13ID:foDcA2jG
>>948
1 - x^n < 1/(1+x^n) < 1 (0<x<1)
0 < 1/(1+x^n) < 1/x^n (1<x)
より、
1 - 1/(n+1) < ∫[0,1] 1/(1+x^n) dx < 1,
0 < ∫[1,∞) 1/(1+x^n) dx < 1/(n-1),
0954132人目の素数さん垢版2017/03/29(水) 06:28:52.30ID:qmY7hsva
Prove that the inequality
   1/√(2x) + 1/√(2y) + 2/√(x+y) + 2 ≧ 4/√(x+2) + 4/√(y+2)
holds for all pairs (x,y) of positive real numbers.
0955132人目の素数さん垢版2017/04/04(火) 08:50:30.08ID:h0o7Rnkh
>>954
f(x,y) = 1/√(2x) + 1/√(2y) + 2/√(x+y) +2 -4/√(x+2) -4/√(y+2) とおく。
x=y のときは凸性から、
f((x+y)/2, (x+y)/2) = 4/√(2x) + 4/√(2+2) - 8/√(x+2) ≧0,
となる。
f(x、y) - f((x+y)/2, (x+y)/2) ≧ 0 を示さねば...
0956132人目の素数さん垢版2017/04/12(水) 09:05:18.35ID:kkWXQg4L
(1)
Let a, b anc c be the lengths of the sides of a triangle with inradius r.
Prove a^6 + b^6 + c^6 ≧ 5184*r^6.

(2)
Suppose that f : [0,1] → R is a differentiable function with continuous derivative and with
   ∫[0,1] f(x)dx = ∫[0,1] xf(x)dx = 1.
Prove that
   ∫[0,1] |f'(x)|^3 dx ≧ {128/(3π)}^2.

(3)
Calclate lim[x→∞] (Σ[n=1 to ∞] (x/n)^n )^(1/x).

(4)
Evaluate ∫[0, π/2] (sin x)/(1 + sqrt{sin 2x}) dx.

(5)
Calclate ∫[0,∞]∫[0,∞] (sin x * sin y * sin(x+y))/{xy(x+y)} dx dy.

(6)
Calclate Σ[n=1 to ∞] {2^(2n-1)/(2n+1)}*{(n-1)!/(2n-1)!!}^2 = π-2.

( ゚∀゚) ウヒョッ!
0958132人目の素数さん垢版2017/04/12(水) 22:09:51.74ID:kkWXQg4L
(7)
Find the greatest real number M such that the inequality
a^2 + b^2 + c^2 + 3abc ≧ M(ab + bc + ca)
holds for all nonnegative real numbers a, b, c satisfying a + b + c = 4.

(8)
Find the greatest real number M such that
(x^2 + y^2)^3 ≧ M(x^3 + y^3)(xy - x - y)
for all real numbers x, y satisfying x + y ≧ 0.

(9)
Let a, b, c be nonnegative real numbers satisfying a^2 + b^2 + c^2 = 1. Prove that
sqrt(a + b) + sqrt(b + c) + sqrt(c + a) ≧ sqrt{ 7(a + b + c) - 3}

(10)
Prove that for all positive real numbers a, b, c satisfying a^2 + b^2 + c^2 + 2abc ≧1,
the following inequality holds:
1/a + 1/b + 1/c ≧ a/b + b/c + c/a + 2(a + b + c).

(11)
Find the greatest real number T satisfying
(x^2 + y)(x + y^2)/(x+y-1)^2 + (y^2 + z)(y + z^2)/(y+z-1)^2 + (z^2 + x)(z + x^2)/(z+x-1)^2 -2(x+y+z) ≧ T
for all real numbers x, y and z such that x+y≠1, y+z≠1, z+x≠1.

(12)
Show that for all nonnegative real numbers a, b, c satisfying a^2 +b^2 +c^2 ≦ 3 the following inequality holds:
(a + b + c)(a + b + c - abc) ≧ 2(a^2・b + b^2・c + c^2・a)

(*゚∀゚)=3ハァハァ
0960132人目の素数さん垢版2017/05/01(月) 16:09:03.66ID:Gg+cOD9T
>>959
(左辺)>e^e/2>(e+1)/2>e^(1/2)>(右辺)
0961132人目の素数さん垢版2017/05/01(月) 16:11:04.11ID:Gg+cOD9T
>>958
(10)反例 a=b=c=1
0962132人目の素数さん垢版2017/05/01(月) 16:11:58.65ID:Gg+cOD9T
萎える
0975132人目の素数さん垢版2017/06/10(土) 18:52:40.27ID:3dLjunNb
>>956 (1) コーシーで
 (1+1+1)(1+1+1)(a^6 + b^6 + c^6) ≧ (aa+bb+cc)^3,
 aa+bb+cc ≧ 36rr を示す。

>>956 (4)   (π-2)/2,


>>959
 1/π + 1/π + 1/e ≧ 1,
相加-相乗 または 凸性から
 e^(1/π) + e^(1/π) + e^(1/e) ≧ 3e^(1/3),
0977132人目の素数さん垢版2017/06/11(日) 16:28:29.36ID:JurbFnaF
>>956 (1) コーシーで
 (1+1+1)^5 (a^6 + b^6 + c^6) ≧ (a+b+c)^6,
一方、
 a = r {cot(B/2) + cot(C/2)},
 b = r {cot(C/2) + cot(A/2)},
 c = r {cot(A/2) + cot(B/2)},
∴ a+b+c = 2r {cot(A/2) + cot(B/2) + cot(C/2)} ≧ 6r cot((A+B+C)/6) = 6r cot(π/6) =(6√3)r
0978132人目の素数さん垢版2017/06/11(日) 16:52:20.52ID:JurbFnaF
>>958 (7)
(a+b+c) {aa+bb+cc - M(ab+bc+ca)} + 12abc
= s(ss-2t) - Mst + 12u
= F_1(a,b,c) + (2-M)st + 3u (← Schur)
≧0,
∴ M=2
0979132人目の素数さん垢版2017/06/16(金) 12:24:58.16ID:LCy4Y8vy
>>956 (1) >>977
(a+b+c)/2 = s とおく。
相乗-相加平均で
(s-a)(s-b)(s-c) ≦ (s/3)^3,

r = /s
= √{(s-a)(s-b)(s-c)/s}  (Heron)
≦ s/(3√3)
= (a+b+c)/(6√3),
0982132人目の素数さん垢版2017/06/18(日) 16:20:22.03ID:tfNCpQJl
>>981

【Flanders' inequality】A+B+C=π, 0<A,B,C<π のとき、
 0 < sin(A)sin(B)sin(C) ≦ {(3√3)/2π}^3 ABC ≦ (3√3)/8,
 (初代スレ.668)

 g(x) = log{sin(x)/x},
 g '(x) = cot(x) - 1/x,
 g "(x) = 1/x^2 - 1/sin(x)^2 < 0,
ゆえ、g(x) は上に凸。


【類題】A+B+C=π, 0<A,B,C<π のとき、
 -1 < cos(A)cos(B)cos(C) ≦ [1-cos(A)][1-cos(B)][1-cos(C)] ≦ 1/8,

 (初代スレを参照、右:557-558,566、中:580-587)
0984132人目の素数さん垢版2017/06/23(金) 00:32:52.83ID:P0aRc9y/
実数 a,b,c,d が a+b+c+d=0, a^2+b^2+c^2+d^2=100 をみたすとき、a^3+b^3+c^3+d^3 のとりうる値の範囲を求めよ。
0988132人目の素数さん垢版2017/06/23(金) 03:23:02.02ID:P0aRc9y/
ノート整理中に見つけたが出典不明。正の数a,b,cに対して (a^b)(b^c)(c^a)≦(a^a)(b^b)(c^c) を示せ。

改造しようと思ったが、すぐには思いつかんかった。
0990132人目の素数さん垢版2017/06/23(金) 12:58:33.41ID:vMBsUZ6Z
>>988
 対数とってチェビシェフ


>>989
さだまさし

(a, b, c) = (1/8, 8, 64) のとき
b log(a) + c log(b) + a log(c) > 100 > (a+b+c)/3 log(abc),
0992132人目の素数さん垢版2017/06/24(土) 01:17:12.91ID:u2QpKHjV
(不等式への招待 第5章 698、708より)
> a,b,c>0→a^{b+c}+b^{c+a}+c^{a+b}≧1
>
> (1) a,b,c の中に1以上のものがあるときは明らか。
>
> 次に M = Max{b+c,c+a,a+b} とおく。
>
> (2) a,b,c ≦ 1 かつ M ≦ 1 のとき
>  b+c≦1, …, …
>  y=x^(b+c) は xについて上に凸だから(x=1での)接線の下側にある。
>  x^(b+c) ≦ 1 +(b+c)(x-1) ≦ 1 + (b+c)x,
>  (1/x)^(b+c) ≧ 1/{1 + (b+c)x},  (ベルヌーイの式)
> x=1/a とおいて
>  a^(b+c) ≧ a/(a+b+c),
>  循環的にたす。
>
> (3) a,b,c ≦ 1 かつ M ≧ 1 のとき
>  0 < a ≦ b,c ≦ 1 としても一般性を失わない。
>  a+b, a+c ≦ b+c = M,
>  (与式) ≧ b^(c+a) + c^(a+b)
>    ≧ b^M + c^M
>    ≧ 2・(M/2)^M   (← 下に凸)
>    ≧ 2(1/2)    (← *)
>    = 1,
>
> *) {M・log(M/2)} ' = 1 + log(M/2),
> ∴ (M/2)^M は M>2/e  で単調増加。
> ∴ (M/2)^M ≧ 1/2,   (M≧1)
>
>  casphy - 高校数学 - 不等式 - 710〜713


等号成立条件が分かりませんぬ。
0994132人目の素数さん垢版2017/06/25(日) 02:29:19.46ID:dLSgUfzK
そろそろ次スレ建てようと思うが、数学板はスレ落ち対策(スレが立ってすぐの時期に、一定時間書き込みが無かったら落ちる)しなくて大丈夫だっけ?
0996132人目の素数さん垢版2017/06/25(日) 08:35:04.48ID:dLSgUfzK
>>995
さんくす。専ブラ使っていて、不等式スレ、面白スレ以外はあぼーんしているので分からなかったぜ。
0997132人目の素数さん垢版2017/06/25(日) 17:20:11.69ID:dLSgUfzK
a,b,c を正の定数、
x,y,z は ax+by+cz=1 をみたす実数、
min{ x/a, y/b, z/c } の最大値を求めよ。
(出典不明)
0998132人目の素数さん垢版2017/06/26(月) 01:13:09.35ID:vrMzbwMW
>>997
x/a=X, y/b=Y, z/c=Z とおく。
X,Y,Z は aaX + bbY + ccZ = 1 をみたす実数。
(aa+bb+cc)*min{X,Y,Z} ≦ aaX + bbY + ccZ = 1,
∴ min{X,Y,Z} ≦ 1/(aa+bb+cc),
10011001垢版Over 1000Thread
このスレッドは1000を超えました。
もう書けないので、新しいスレッドを立ててくださいです。。。
life time: 1569日 3時間 9分 28秒
10021002垢版Over 1000Thread
2ちゃんねるの運営はプレミアム会員の皆さまに支えられています。
運営にご協力お願いいたします。


───────────────────
《プレミアム会員の主な特典》
★ 2ちゃんねる専用ブラウザからの広告除去
★ 2ちゃんねるの過去ログを取得
★ 書き込み規制の緩和
───────────────────

会員登録には個人情報は一切必要ありません。
月300円から匿名でご購入いただけます。

▼ プレミアム会員登録はこちら ▼
https://premium.2ch.net/

▼ 浪人ログインはこちら ▼
https://login.2ch.net/login.php
レス数が1000を超えています。これ以上書き込みはできません。

ニューススポーツなんでも実況